Sie sind auf Seite 1von 225

UNIVERSIDADNACIONALABIERTAYADISTANCIA UNAD

ESCUELADECIENCIASBSICAS,TECNOLOGAEINGENIERA
CONTENIDODIDCTICODELCURSO:100411ClculoIntegral

UNIVERSIDAD NACIONAL ABIERTA Y A DISTANCIA


ESCUELA DE CIENCIAS BSICAS, TECNOLOGA E INGENIERA
PROGRAMA CIENCIAS BSICAS






100411 Clculo Integral
JORGE ELICER RONDON DURAN
Autor
JOS PEDRO BLANCO ROMEERO
Director Nacional
MARTIN GOMEZ ORDUZ
Acreditador


Bogot, D. C Agosto de 2010

UNIVERSIDADNACIONALABIERTAYADISTANCIA UNAD
ESCUELADECIENCIASBSICAS,TECNOLOGAEINGENIERA
CONTENIDODIDCTICODELCURSO:100411ClculoIntegral


ASPECTOS DE PROPIEDAD INTELECTUAL Y VERSIONAMIENTO

El presente mdulo fue diseado en el ao 2007 por el Ing. JORGE ELIECER
RONDON DURAN docente de la UNAD, ubicado en el CEAD de JOSE
CELESTINO MUTIS, el Autor es de profesin ingeniero. Se ha desempeado
como tutor de la UNAD desde hace varios aos, empezando como tutor hasta el
cargo que ocupa en la actualidad de coordinador nacional de Ciencias Bsicas.
Como novedades se presentan otros aspectos didcticos que facilitan el estudio
autnomo del clculo integral, as como la estructura y contenidos solicitados por
la VIMMEP y la ECBTI.


MARTIN GOMEZ, licenciado en fsica y matemticas de la UPTC, tutor de tiempo
completo de Yopal Casanare, apoy el proceso de revisin de estilo del mdulo
y dio aportes disciplinares, didcticos y pedaggicos en el proceso de acreditacin
del material didctico, este trabajo se llevo a cabo en los meses de Julio y Agosto
de 2009.

Este documento se puede copiar, distribuir y comunicar pblicamente bajo las
condiciones siguientes:

Reconocimiento. Debe reconocer los crditos de la obra de la manera
especificada por el autor o el licenciador (pero no de una manera que
sugiera que tiene su apoyo o apoyan el uso que hace de su obra).

No comercial. No puede utilizar esta obra para fines comerciales.

Sin obras derivadas. No se puede alterar, transformar o generar una obra
derivada a partir de esta obra.

Al reutilizar o distribuir la obra, tiene que dejar bien claro los trminos de la
licencia de esta obra.

Alguna de estas condiciones puede no aplicarse si se obtiene el permiso del
titular de los derechos de autor

Nada en esta menoscaba o restringe los derechos morales del autor.

UNIVERSIDADNACIONALABIERTAYADISTANCIA UNAD
ESCUELADECIENCIASBSICAS,TECNOLOGAEINGENIERA
CONTENIDODIDCTICODELCURSO:100411ClculoIntegral


INTRODUCCIN

La matemtica es una ciencia eminentemente terica, debido a que parte de
teoras y definiciones, cuyas demostraciones se soportan en el principio de la
lgica, los axiomas y postulados, que permiten el desarrollo de habilidades de
pensamiento de orden superior, especialmente la Deduccin, Induccin y la
Abstraccin, pero a su vez presenta dificultades para poder desplegar dichas
habilidades, ya que se requiere trabajar el sentido de anlisis, desarrollo del
raciocinio, aspectos no fciles de activar en la mente humana.

El Clculo Integral es el rea de las matemticas, que pertenece al campo de
formacin disciplinar y tiene carcter bsico en cualquier rea del saber, debido a
que los Ingenieros, Administradores, Economistas, Fsicos, Qumicos, por
supuesto los Matemticos y dems profesionales requieren de esta rea del
saber.

Un buen conocimiento del clculo diferencial, permite y facilita trabajar el curso de
clculo integral, en donde se desarrollan teoras, principios y definiciones
matemticas propias del clculo infinitesimal. El objetivo fundamental es que los
estudiantes puedan identificar, comprender e interiorizar las temticas que cubren
el curso, con el fin de adquirir conocimientos matemticos que le den capacidad
de resolver problemas donde el clculo Univariado es protagonista.

El Clculo Integral es la rama de las Matemticas muy utilizadas en Ciencias,
Tecnologa, Ingeniera e Investigacin, que requiere un trabajo sistemtico y
planificado, para poder cumplir el propsito fundamental que es saber integrar,
tcnica que permite solucionar problemas de estos campos. Por otro lado, la
integracin es necesaria para otros escenarios como las Ecuaciones
Diferenciales, los Mtodos Numricos, la geometra diferencial, la Probabilidad, la
Estadstica Avanzada y otras reas del conocimiento.

UNIVERSIDADNACIONALABIERTAYADISTANCIA UNAD
ESCUELADECIENCIASBSICAS,TECNOLOGAEINGENIERA
CONTENIDODIDCTICODELCURSO:100411ClculoIntegral

Las Unidades Didcticas que conforman el curso son: La Integracin, Los Mtodos
de Integracin y Las Aplicaciones de las integrales. En la primera unidad se
desarrolla lo referente a la antiderivada o primitiva, la integral indefinida, la integral
definida, el teorema fundamental del clculo y las integrales impropias. La
segunda unidad presenta lo relacionado con las tcnicas de integracin, iniciando
con las integrales inmediatas producto de la definicin de antiderivada, la
integracin por cambio de variable o tambin llamada sustitucin, integracin por
partes, integracin por fracciones parciales, integracin de funciones
trascendentales; tales como, exponencial, logartmica, trigonomtricas e
hiperblicas. La tercera unidad presenta las aplicaciones de la integracin, tales
como reas bajo curvas, longitud de una curva, volmenes de slidos de
revolucin, la integracin en la fsica, en la estadstica y en la economa.
En los ejercicios propuestos, para las primeras temticas, no se dan las
respuestas ya que stas son muy obvias, pero para las dems temticas, se
ofrecen las respuestas, con el fin de motivar el procedimiento de los mismos. Es
pertinente desarrollarlos de manera metdica y cuidadosa; adems, confrontar la
respuesta obtenida con la dada en el mdulo, cualquier aclaracin compartirla con
el tutor o el autor a travs del correo jorge.rondon@unad.edu.co
Como el conocimiento se va renovando y actualizando, los aportes que se hagan
al presente material sern bien venidos, esperando as una actividad continua de
mejoramiento en beneficio de todos los usuarios del material. Como el material
presenta las temticas fundamentales, es pertinente complementar con otras
fuentes como libros, de los cuales se presentan en la bibliografa, Internet y otros.
Es recomendable desarrollar el trabajo acadmico de manera adecuada, como se
explicita en el modelo acadmico pedaggico que la UNAD tiene, para obtener
los mejores resultados del curso. El estudio independiente, como primer
escenario, es fundamental para la exploracin, anlisis y comprensin de las
temticas. El Acompaamiento Tutorial, debe permitir complementar el trabajo
realizado en el escenario anterior, especialmente en la aclaracin de dudas,
complementacin y profundizacin pertinente. En este aspecto, se deben explorar
las herramientas que estn a la mano para aprovechar de la mejor manera dichos
recursos, as el grado de aprendizaje es ms amplio y se ver mejor reflejado el
aprendizaje autnomo.

El autor.

UNIVERSIDADNACIONALABIERTAYADISTANCIA UNAD
ESCUELADECIENCIASBSICAS,TECNOLOGAEINGENIERA
CONTENIDODIDCTICODELCURSO:100411ClculoIntegral


INDICE DE CONTENIDO
UNIDAD UNO: LA INTEGRACION
CAPTULO 1: LA INTEGRAL INDEFINIDA
Leccin 1: La integracin
Leccin 2: La Antiderivada
Leccin 3: Integral indefinida
Leccin 4: Propiedades de las Integrales indefinidas.
Leccin 5: La constante de integracin

CAPTULO 2: LA INTEGRAL DEFINIDA
Leccin 6: Sumas De RIEMANN
Leccin 7: rea bajo la curva
Leccin 8: Estimacin por sumas finitas.
Leccin 9: Definicin
Leccin 10: Integral definida

CAPTULO 3: TEOREMAS
Leccin 11: Teorema de integrabilidad
Leccin 12: Valor medio de una funcin
Leccin 13: Primer teorema fundamental del clculo
Leccin 14: Segundo teorema fundamental del clculo
Leccin 15: Teorema de simetra
Actividades de autoevaluacin de la Unidad 1
Fuentes documentales de la Unidad 1

UNIVERSIDADNACIONALABIERTAYADISTANCIA UNAD
ESCUELADECIENCIASBSICAS,TECNOLOGAEINGENIERA
CONTENIDODIDCTICODELCURSO:100411ClculoIntegral


UNIDAD DOS: TCNICAS DE INTEGRACIN
CAPTULO 4: MTODOS DE INTEGRACIN I
Leccin 16: Integrales Impropias con integrando discontinuo
Leccin 17: Integrales impropias con lmites de integracin infinitos
Leccin 18: Integrales Inmediatas
Leccin 19: Integrales inmediatas con sustitucin
Leccin 20: Integracin por cambio de variable

CAPTULO 5: MTODOS DE INTEGRACIN II
Leccin 21: Integracin por racionalizacin
Leccin 22: Integracin por sustitucin trigonomtrica caso I
Leccin 23: Integracin por sustitucin trigonomtrica caso II
Leccin 24: Integracin por sustitucin trigonomtrica caso III
Leccin 25: Integracin por partes

CAPTULO 6: MTODOS DE INTEGRACIN III
Leccin 26: Integracin por fracciones parciales.
Leccin 27: Integracin de funcin exponencial
Leccin 28: Integracin de funcin logartmica
Leccin 29: Integracin de la funcin trigonomtrica
Leccin 30: Integracin de la funcin hiperblica
Actividades de autoevaluacin de la Unidad 2
Fuentes documentales de la Unidad 2

UNIVERSIDADNACIONALABIERTAYADISTANCIA UNAD
ESCUELADECIENCIASBSICAS,TECNOLOGAEINGENIERA
CONTENIDODIDCTICODELCURSO:100411ClculoIntegral


UNIDAD TRES: APLICACIONES DE LAS INTEGRALES
CAPTULO 7: ANLISIS DE GRAFICAS
Leccin 31: rea de regiones planas
Leccin 32: rea entre curvas
Leccin 33: rea de superficies de revolucin
Leccin 34: Longitud de una curva
Leccin 35: Longitud de un arco en forma paramtrica.

CAPTULO 8: VOLUMEN DE SUPERFICIE DE REVOLUCION.

Leccin 36: Volumen de slidos de revolucin: Mtodo de arandelas
Leccin 37: Volumen de slidos de revolucin: Mtodo de casquetes cilndricos
Leccin 38: Volumen de slidos de revolucin: Mtodo de rebanadas o discos.
Leccin 39: Momentos y centros de masa.
Leccin 40: Volumen.

CAPTULO 9: EN LAS CIENCIAS
Leccin 41: Integrales en la fsica: trabajo y movimiento.
Leccin 42: Integrales en la hidrulica: bombeo de lquidos.
Leccin 43: Integrales en la estadstica: Funcin de distribucin
Leccin 44: Integrales en la economa.
Leccin 45: Integrales en las ciencias sociales.
Actividades de autoevaluacin de la Unidad 3
Fuentes documentales de la Unidad 3

UNIVERSIDADNACIONALABIERTAYADISTANCIA UNAD
ESCUELADECIENCIASBSICAS,TECNOLOGAEINGENIERA
CONTENIDODIDCTICODELCURSO:100411ClculoIntegral


LISTADO DE TABLAS

Tabla No. 1 Listado de integrales inmediatas.

UNIVERSIDADNACIONALABIERTAYADISTANCIA UNAD
ESCUELADECIENCIASBSICAS,TECNOLOGAEINGENIERA
CONTENIDODIDCTICODELCURSO:100411ClculoIntegral


LISTADO DE GRFICOS Y FIGURAS

Figura No. 1 Polgonos circunscritos
Figura No. 2 Polgonos inscritos
Figura No. 3 Particin
Figura No. 4 rea
Figura No. 5 Integral impropia
Figura No. 6 Convergencia
Figura No. 7 Sustitucin trigonomtrica caso 1
Figura No. 8 Sustitucin trigonomtrica caso 2
Figura No. 9 Sustitucin trigonomtrica caso 3
Figura No. 10 Aplicacin
Figura No. 11 rea bajo la curva
Figura No. 12 Particiones
Figura No. 13 Grafica de 2x
Figura No. 14 Grafica de x
3
Figura No. 15 Grafica de y=3-x
2
Figura No. 16 rea entre curvas
Figura No. 17 Grafica solucin problema No. 1
Figura No. 18 Solucin rea bajo curvas
Figura No. 19 Solucin problema No. 3
Figura No. 20 Superficie de revolucin
Figura No. 21 Superficie de revolucin de
2
x y =

UNIVERSIDADNACIONALABIERTAYADISTANCIA UNAD
ESCUELADECIENCIASBSICAS,TECNOLOGAEINGENIERA
CONTENIDODIDCTICODELCURSO:100411ClculoIntegral

Figura No. 22 Superficie de revolucin de x y =
Figura No. 23 Longitud de curva
Figura No. 24 Demostracin longitud de curva
Figura No. 25 Longitud de curva paramtrica.
Figura No. 26 Arandelas
Figura No. 27 Solucin volumen ejemplo No. 1
Figura No. 28 Solucin volumen ejemplo No. 3
Figura No. 29 Casquetes
Figura No. 30 Desarrollo slidos de revolucin
Figura No. 31 Solucin ejemplo No. 1
Figura No. 32 Solucin ejemplo No. 2
Figura No. 33 Demostracin casquetes
Figura No. 34 Rebanadas
Figura No. 35 Discos
Figura No. 36 Solucin problema No. 1
Figura No. 37 Solucin problema No. 2
Figura No. 38 Centro de masa
Figura No. 39 Centroide
Figura No. 40 Teorema de Pappus
Figura No. 41 Bombeo
Figura No. 42 Bombeo circular
Figura No. 43 Curva oferta - demanda
Figura No. 44 Excedente del consumidor
Figura No. 45 Excedente del productor

UNIVERSIDADNACIONALABIERTAYADISTANCIA UNAD
ESCUELADECIENCIASBSICAS,TECNOLOGAEINGENIERA
CONTENIDODIDCTICODELCURSO:100411ClculoIntegral


UNIDAD 1: LA INTEGRACION

Introduccin:


Una dificultad que enfrento a la humanidad desde hace muchos siglos fue el
clculo de reas y volmenes de cuerpos conocidos, quien enfrento primero este
problema al parecer fue Eudoxo de Cnido por all por el siglo IV antes de nuestra
era. Eudoxo ideo el mtodo de exhaucion el cual consista en descomponer en
partes muy pequeas las areas y los volmenes para luego componerlas y de
esta manera obtener las superficies y los grosores de los cuerpos.

La Geometra griega se interes pronto por las reas de figuras en el plano y los
volmenes de cuerpos geomtricos. Tambin tempranamente descubrieron que el
tratamiento de las figuras de contornos curvilneos no era sencillo de abordar.
Algunos estudiosos de la antigedad que se interesaron por el tema fueron:

KEPLER
1
Estaba interesado en las cnicas para su aplicacin en la astronoma,
por lo tanto, plantea el clculo del rea de una rbita considerndola que esta
formada por tringulos infinitamente pequeos con un vrtice en el Sol; esto da
origen a un clculo integral rudimentario. El estudio de los volmenes lo retomo
para el clculo del vino al ver la inexactitud de la capacidad de los toneles.


GALILEO
2
Se interesa por la parbola, al estudiar la trayectoria de un proyectil y
hallar la integral que expresa el espacio recorrido en un movimiento
uniformemente acelerado.

LEIBNIZ
3
Sistematizo y logro un desarrollo eficiente.


Mayor informacin en el siguiente link:

http://www.matematicas.profes.net/archivo2.asp?id_contenido=45558


___________________
1
Naci en 1571 en WEIL DER STADT y muri en RATISBONA en 1630 (Alemania).
2
Naci en 1564 en PIZA y muri en FLORENCIA 1642 (Italia).
3
Por primera vez utilizo el smbolo

que aparece de estilizar la S de las sumatorias.


UNIVERSIDADNACIONALABIERTAYADISTANCIA UNAD
ESCUELADECIENCIASBSICAS,TECNOLOGAEINGENIERA
CONTENIDODIDCTICODELCURSO:100411ClculoIntegral



Justificacin:


Tanto la integral como la derivada son herramientas importantes que ayudan a
resolver problemas en la fsica, la estadstica, la probabilidad, la hidrulica y otros
campos de las ciencias; es por eso que temas tan importantes son abordados en
esta unidad.


En esta primera unidad presentamos tres captulos en los cuales tratamos las
bases de la integracin empezando por la integral indefinida, la integral definida y
en el tercer captulo retomamos el tema de los teoremas claves para comprender
mejor el estudio de las integrales.


Intencionalidades formativas:


Para esta unidad podemos enumerar como intencionalidades formativas las
siguientes:

Que los estudiantes identifiquen los principios del clculo integral para
asimilar la teora de las integrales.
Los estudiantes interpreten las diferentes teoras, definiciones y teoremas
del clculo integral para poder comprender en diversos escenarios su mejor
manera de utilizarlos.
Manejar de manera apropiada las integrales indefinidas, las integrales
definidas y los teoremas en los cuales se basan.














UNIVERSIDADNACIONALABIERTAYADISTANCIA UNAD
ESCUELADECIENCIASBSICAS,TECNOLOGAEINGENIERA
CONTENIDODIDCTICODELCURSO:100411ClculoIntegral

Presentamos un cuadro con el resumen del contexto terico de esta unidad


Denominacin de
los captulos
CAPITULO 1: La integral indefinida

CAPITULO 2 La integral definida

CAPITULO 3 Teoremas que la sustentan
Asimilacin de
conceptos

Los lectores de la primera unidad la integracin, estarn en
capacidad de comprender los conceptos fundamentales del
clculo integral en cuanto a sus orgenes, diferentes clases
de integracin, la apropiacin de la simbologa empleada,
los teoremas que la sustentan y tienen una visin general
del curso.
Conceptos

Esta Unidad parte de conceptos elementales para ir
adentrando al estudiante en conceptos ms amplios y
complejos empleados en el Clculo Integral.
Competencias


De conocimientos
Adquirir las tcnicas propias del clculo integral.
El conocimiento en matemticas se adquiere con
papel y lpiz en la realizacin de ejercicios que estn
propuestos en esta unidad o en la bibliografa y
cibergrafia sugeridas.

Contextuales:
Adquirir los conocimientos propios del curso
acadmico con el fin de aplicarlos en la solucin de
problemas de su carrera y de esta manera poner el
prctico el aprendizaje significativo.
Los estudiantes deben desarrollar habilidades para
aplicar los conocimientos adquiridos en la solucin de
problemas prcticos.

Comunicativas:
Adquirir la jerga propia del lenguaje utilizado en el
clculo integral.
Interpretar y entenderlos la diferente simbologa y su
aplicacin.
Adquirir facilidad de expresin y vencer el miedo en
la interaccin con las NTIC

UNIVERSIDADNACIONALABIERTAYADISTANCIA UNAD
ESCUELADECIENCIASBSICAS,TECNOLOGAEINGENIERA
CONTENIDODIDCTICODELCURSO:100411ClculoIntegral
Valorativas:
Adoptar, identificar y practicar lo valores de la UNAD.
Adquirir capacidad de valoracin y tolerancia con
nuestros compaeros virtuales o presenciales.


UNIVERSIDADNACIONALABIERTAYADISTANCIA UNAD
ESCUELADECIENCIASBSICAS,TECNOLOGAEINGENIERA
CONTENIDODIDCTICODELCURSO:100411ClculoIntegral

CAPITULO 1: La integral indefinida
Introduccin
La derivada corresponde a la nocin geomtrica de tangente y a la idea fsica de
velocidad, es decir dada una curva calcular su pendiente o dado el recorrido de un
mvil calcular su velocidad, mientras que la idea de integral est relacionada con
la nocin geomtrica de rea y la idea fsica de trabajo, por lo tanto, dada una
funcin se halla el rea comprendida bajo la curva o dada una fuerza variable, se
calcula el trabajo realizado por dicha fuerza.
Partiendo de este ltimo concepto este captulo pretender ilustrar el concepto de la
integral indefinida, en el cual tenemos que si nos dan la derivada de una funcin
nosotros debemos hallar dicha funcin.
Leccin 1: La integracin
En el mundo de las Matemticas encontramos que existen operaciones opuestas,
como la suma y la resta, el producto y el cociente, donde una deshace o anula la
otra. De la misma manera la Integracin es una operacin opuesta a la
Diferenciacin. La relacin Diferenciacin Integracin es una de los
conocimientos ms importantes en el mundo de las Matemticas. Ideas
descubiertas en forma independiente por los grandes Matemticos Leibniz y
Newton. Inicialmente Leibniz al proceso de integracin lo llamo: Calculus
Summatorius pero en 1.696 influenciado por Johann Bernoulli, de la dinasta
Bernoulli, le cambio el nombre a Calculus Integrelis.






Gottfried Wilhelm von Leibniz
1


___________________
1
Julio de 1646 Noviembre de 1716 HANNOVER Alemania.
Gran Filosofo, politlogo y matemtico.
Precursor de la Lgica Matemtica, desarrollo el Clculo,
independiente de Newton, publicando su trabajo en 1.684,
su notacin es la que se utiliza actualmente. Descubri el
sistema binario, muy utilizado en los sistemas
informticos. Contribuyo a la creacin de la Real
Academia de Ciencias en Berln en 1.670, siendo su
primer presidente.

UNIVERSIDADNACIONALABIERTAYADISTANCIA UNAD
ESCUELADECIENCIASBSICAS,TECNOLOGAEINGENIERA
CONTENIDODIDCTICODELCURSO:100411ClculoIntegral

El clculo ha sido una secuencia de reas matemticas entrelazadas, donde se
utilizan principios de lgebra, Geometra, Trigonometra, se debe destacar que
para desarrollar el curso de Clculo Integral, es pertinente tener claros los
principios de las reas nombradas y adems los de Clculo Diferencial, ya que
como se dijo en el prrafo anterior, la integracin es la opuesta a la diferenciacin.
Leccin 2: La Antiderivada
Para conceptuar la Antiderivada, comencemos por pensar que se tiene una
funcin, digamos ) (x f , el trabajo consiste en encontrar otra funcin, digamos
) (x D tal que: ) ( ) ( ' x f x D = . As D(x) es una antiderivada de f(x). Identificar una
funcin a partir de su derivada, consiste en hallar un dispositivo (tcnica) que nos
de todas las funciones posibles, donde f(x) es su derivada, a dichas funciones se
les llama Antiderivada de f(x). El dispositivo para ste proceso es llamado La
Integracin.
Veamos un ejemplo sencillo: Sea f(x) = 2x, cual ser una funcin D(x) cuya
derivada es 2x? Con algo se astucia y conocimientos slidos en diferenciacin
podemos identificar que D(x) = x
2.
Veamos: Si derivamos D(x) = x
2
obtenemos
f(x) = 2x.
Otro ejemplo: f(x) = cos(x), cual ser un D(x)? Debemos buscar una funcin
cuya derivada es cos(x), evidentemente es sen(x), luego D(x) = sen(x).
Para la notacin de antiderivada hubo diversas propuestas, pero la del gran
Matemtico Leibniz es la ms utilizada universalmente.

dx ... . Posteriormente se
analizar esta notacin.
Para los ejemplos anteriores con la notacin de Leibniz se tiene:
c x dx x + =

2
) 2 ( Para el otro: c x sen dx x + =

) ( ) cos(
Posteriormente se aclara el concepto de la c




DEFINICINNo1:
UnafuncinD(x)esunaantiderivadadelafuncinf(x),si:
D(x)=f(x).Paratodoxeneldominiodef(x).

UNIVERSIDADNACIONALABIERTAYADISTANCIA UNAD
ESCUELADECIENCIASBSICAS,TECNOLOGAEINGENIERA
CONTENIDODIDCTICODELCURSO:100411ClculoIntegral

El conjunto de todas las antiderivadas de f(x) se le llama la Integral Indefinida de
f(x) y se puede escribir: c x D dx x f + =

) ( ) (





Demostracin:
Como G(x) y F(x) son antiderivadas de f(x), entonces tenemos que: G(x) = F(x),
por una definicin previa que dice: si g(x) = f(x) entonces: g(x) = f(x) + c para
todo x en el intervalo I abierto. Por consiguiente: G(x) = F(x) + c, para alguna
constante c.
Ejemplo No 1:
Encontrar todas las funciones cuya derivada es f(x) = 4x
3
+ 2.
Solucin:
Una funcin puede ser x
4
+ 2x + 5, ya que al derivarla obtenemos 4x
3
+ 2. Luego:
Si
f(x) = 4x
3
+ 2, entonces D(x) = x
4
+ 2x + 5, pero tambin puede ser D(x) = x
4
+ 2x +
12. En general cualquier funcin de la forma D(x) = x
4
+ 2x + C, es antiderivada de
la funcin f(x), siendo C una constante.
Ejemplo No 2:
Encontrar todas las funciones cuya derivada es: f(x) = sec
2
(x).
Solucin:
Si recordamos sobre derivadas de funciones trigonomtricas, podemos saber que
la funcin cuya derivada corresponde a sec
2
(x), es tan(x), luego:
Si f(x) = sec
2
(x), entonces D(x) = tan(x) + C

TEOREMA:
SeanF(x)yG(x)antiderivadasdef(x)enunintervalocerradoI,
entonces:
G(x)=F(x)+cparaalgunaconstantec.

UNIVERSIDADNACIONALABIERTAYADISTANCIA UNAD
ESCUELADECIENCIASBSICAS,TECNOLOGAEINGENIERA
CONTENIDODIDCTICODELCURSO:100411ClculoIntegral

Por consiguiente, la forma de las funciones cuya derivada corresponde a sec
2
(x)
es: D(x) = tan(x) + c
Ejemplo No 3:
Hallar algunas funciones cuya derivada es g(x) = 12
Solucin:
Cualquier funcin de la forma 12x + C es antiderivada de g(x), luego algunas de
estas puede ser:
G(x) = 12x + 5, G(x) = 12x + 10, G(x) = 12x + 25
En general: G(x) = 12x + C
Los ejercicios propuestos, se deben desarrollar, utilizando las definiciones y
teoremas, analizados en este aparte.
EJERCICIOS:
Encontrar la antiderivada F(x) + C de las siguientes funciones:

1. f(x) = 8

2. f(x) = 3x
2
+ 4

3. f(x) = x
21
x
10


4. f(x) = 3/x
4
6/x
5


5. f(x) = (3x
2
5x
6
) / x
8


Desarrollar la operacin propuesta:
6.

dx x ) 6 (
5

7. ( )

+ dx x
2
2
7 3

UNIVERSIDADNACIONALABIERTAYADISTANCIA UNAD
ESCUELADECIENCIASBSICAS,TECNOLOGAEINGENIERA
CONTENIDODIDCTICODELCURSO:100411ClculoIntegral

8.
( )
dy
y
y y

+
2
3
4

9. [ ]dx x x sen

) ( csc ) (
2

10.

dx

UNIVERSIDADNACIONALABIERTAYADISTANCIA UNAD
ESCUELADECIENCIASBSICAS,TECNOLOGAEINGENIERA
CONTENIDODIDCTICODELCURSO:100411ClculoIntegral

Leccin 3: Integral indefinida.
Conociendo el concepto de Antiderivada, podemos formalizar desde el punto de
vista matemtico la integral indefinida. Leibniz (1.646 1.716) a la Antiderivada
la llamo Integral Indefinida, quizs pensando que este tipo de integrales
incluye una constante arbitraria.
Luego podemos definir la integral indefinida de la siguiente manera:


( ) ( )

+ = c x D dx x f


Donde:

Smbolo de integracin.
f(x) = Integrando
dx = diferencial de la variable,
D(x) = La integral de f(x)
c = constante de integracin.
Veamos un poco esta nomenclatura matemtica: Por definicin de derivada
tenemos:
[ ] dx x f x D x f x D
dx
d
) ( ) ( ' ) ( ) ( = =

La operacin opuesta:
dx x f x D d x f
dx
x D d
) ( )) ( ( ) (
)) ( (
= =

c dx x f x D dx x f x D d + = =

) ( ) ( ) ( )) ( (

UNIVERSIDADNACIONALABIERTAYADISTANCIA UNAD
ESCUELADECIENCIASBSICAS,TECNOLOGAEINGENIERA
CONTENIDODIDCTICODELCURSO:100411ClculoIntegral

No debemos olvidar la constante de integracin.
Con base en las definiciones anteriores y los conceptos analizados, se puede
obtener algunas integrales, basado en la teora de la antiderivada.

INTEGRALES INMEDIATAS:
INTEGRAL DERIVADA

+ = C x dx
1 ) ( = + c x
dx
d

+
+
=
+
c
n
x
dx x
n
n
1
1
para n -1
n
n
x c
n
x
dx
d
=

+
+
+
1
1

+ = c
n
e
dx e
nx
nx
para n 0
nx
nx
e c
n
e
dx
d
=

+ = c
a Log
a
dx a
x
x
) (
para a > 0
x
x
a c
a Log
a
dx
d
=

+
) (

+ = c
k
kx
dx kx sen
) cos(
) ( para k 0
) (
) cos(
kx sen c
k
kx
dx
d
=

+ =

c x Ln dx
x
) (
1
[ ]
x
c x Ln
dx
d 1
) ( = +
c x Sen x d
x
+ =

) (
1
1
1
2

[ ]
2
1
1
1
) (
x
x Sen
dx
d

+ = c x dx x ) tan( ) ( sec
2

[ ] ) ( sec ) tan(
2
x c x
dx
d
= +

Tabla No. 1
___________________
1
Listado de integrales inmediatas.

UNIVERSIDADNACIONALABIERTAYADISTANCIA UNAD
ESCUELADECIENCIASBSICAS,TECNOLOGAEINGENIERA
CONTENIDODIDCTICODELCURSO:100411ClculoIntegral

Leccin 4: Propiedades de las integrales.
Para las propiedades indefinidas, podemos destacar las siguientes propiedades,
consecuencia de las aplicadas en la diferenciacin.
1.

= dx x f dx x f ) ( ) (

2.

= dx x f k dx x kf ) ( ) (

3.

+ = c kx kdx

4.
[ ]

= dx x kg dx x kf dx x kg x kf ) ( ) ( ) ( ) (

5.
c x f Ln dx
x f
x f
+ =

) (
) (
) ( '

La demostracin se pude hacer por medio de sustitucin.
6.
[ ]
[ ]
c
p
x f
dx x f x f
p
p
+
+
=

+
1
) (
) ( ' ) (
1

La demostracin se puede hacer por medio de la tcnica de sustitucin.

Veamos algunos ejemplos:
1.

+ = = c x dx dx 4 4 4
Aplicando las propiedades 1 y 2.
2.

+ = = c e dx e dx e
x x x 2 2 2
2
5
5 5
Aplicando propiedad 3 e integrales
inmediatas.
3.
( )

+ = + dx x sen dx x dx x dx x sen x x ) ( 2 4 3 ) ( 2 4 3
3 2 3 2

UNIVERSIDADNACIONALABIERTAYADISTANCIA UNAD
ESCUELADECIENCIASBSICAS,TECNOLOGAEINGENIERA
CONTENIDODIDCTICODELCURSO:100411ClculoIntegral

Aplicamos las propiedades 3 y 4, luego:


+ + + = + c x x x dx x sen dx x dx x ) cos( 2 ) ( 2 4 3
4 3 3 2

4.
c x Ln dx
x
x
+ + =

4 3
4 3
6
2
2
Aplicamos la propiedad 5.
5.
( ) ( ) ( ) c x sen x dx x x x sen x + = =

5
2
4
2
) 2 ( 5
5
1
) 2 cos( 2 10 ) 2 ( 5

Aplicamos la propiedad 6.

Leccin 5: La constante de integracin.
Retomando lo manifestado en el Teorema No 1, podemos observar que las
antiderivadas de una funcin slo se diferencian por una constante C dada. Si
recordamos el ejemplo

+ = c x dx x ) tan( ) ( sec
2
, podemos especificar algunas
antiderivadas.
D(x) = 2 ) tan( + x , D(x) = 2 ) tan( + x , D(x) = 5 ) tan( + x , D(x) = 100 ) tan( + x , .
A partir de lo anterior, se afirma que la constante de integracin es propia de las
integrales indefinidas, ya que son muchas las antiderivadas de una funcin que
contiene el integrando.
Por otro lado, cuando estamos integrando donde hay suma o resta, cada trmino
tendr su constante de integracin, pero todas las constantes obtenidas se
pueden agrupar en una sola.
Ejemplo No 1.
Desarrollar: dx x e x
x

+ )) cos( 2 7 (
4

Solucin:
Aplicando las propiedades de suma y resta tenemos:

UNIVERSIDADNACIONALABIERTAYADISTANCIA UNAD
ESCUELADECIENCIASBSICAS,TECNOLOGAEINGENIERA
CONTENIDODIDCTICODELCURSO:100411ClculoIntegral

dx x e x
x

+ )) cos( 2 7 (
4
=

+ dx x dx e dx x
x
) cos( 2 7
4
desarrollando cada integral.
3 2 1
5
) ( 2
5
7
c x sen c e c x
x
+ + + + , luego las constantes las podemos agrupar en una
sola:
3 2 1
5
) ( 2
5
7
c x sen c e c x
x
+ + + + = C x sen e x
x
+ + ) ( 2
5
7
5


Ejemplo No 2.
Hallar: ( )dx e
x x

+
4
2
Solucin:
Aplicando las propiedades y las integrales inmediatas:
( )

+ + + = + = +
2
4
1
4 4
4
1
) 2 (
2
2 2 c e c
Ln
dx e dx dx e
x
x
x x x x
Agrupado las constantes:
( ) c e
Ln
dx e
x
x
x x
+ + = +

4 4
4
1
) 2 (
2
2


EJERCICIOS:
Hallar las antiderivadas de las funciones dadas:
1. 20 ) ( = x f
2. 2 ) (
4
+ = x x f
3.
x
x
x f

=
1
) (
4.
x
e x sen x f
2
2 ) 2 ( 3 ) ( + =

UNIVERSIDADNACIONALABIERTAYADISTANCIA UNAD
ESCUELADECIENCIASBSICAS,TECNOLOGAEINGENIERA
CONTENIDODIDCTICODELCURSO:100411ClculoIntegral

Aplicando las propiedades, resolver las siguientes integrales.
5.

dx 6
6. ( )dx x x

+ ) 3 ( sec 2 25
2 3

7. ( )dx x sen e
t

7 ) 5 ( 2
8. dx
x
x

) tan(
) ( sec
2

9. ( )dx t t t

+ + 3 8 ) 2 3 4 (
2

10. dx
e
e
x
x

+ 5


UNIVERSIDADNACIONALABIERTAYADISTANCIA UNAD
ESCUELADECIENCIASBSICAS,TECNOLOGAEINGENIERA
CONTENIDODIDCTICODELCURSO:100411ClculoIntegral

CAPITULO 2: La integral definida

Introduccin
( ) ( ) ( ) a F b F dx x f
b
a
=


Para analizar las integrales definidas es necesario el estudio de los conceptos de
Sumatorias, Sumas de Riemman y reas bajo la curva. Cada una se irn
desarrollando de manera secuencial, para poder interiorizarlas adecuadamente.
El tema de Sumatorias, se desarroll en el curso de lgebra, Trigonometra y
Geometra Analtica, sin embargo para cualquier duda o aclaracin es pertinente
consultarlo en dicho curso.

Leccin 6: Sumas de Riemann
Comencemos por definir una funcin f(x) en el intervalo cerrado I = [a, b], en dicho
intervalo puede haber valores positivos y negativos; incluso, podra ser no
continua. Hacemos una particin P del intervalo I en n subintervalos, para
facilidad se hace una particin regular, pero no necesariamente debe ser regular,
dicha particin debe tener la condicin que:
X
0
< X
1
< X
2
< < X
n-1
< X
n
, donde a = X
0
y b = X
n

Ahora sea X
i
= X
i
X
i-1
El tamao del subintervalo. En cada subintervalo se
escoge un punto muestra, puede ser un punto frontera.
i
x
~
.
X
1
= X
1
X
0.

X
2
= X
2
X
1



As para los dems intervalos.
Como la particin se hizo sobre la funcin f(x), entonces:

UNIVERSIDADNACIONALABIERTAYADISTANCIA UNAD
ESCUELADECIENCIASBSICAS,TECNOLOGAEINGENIERA
CONTENIDODIDCTICODELCURSO:100411ClculoIntegral



Suma de Riemman.

Aqu R
p
es la suma de Riemman para f(x) en la particin P.






Georg Friedrich Bernhard Riemann
1
Fig. No. 1 Polgonos circunscritos.


Ejemplo No 1:
Evaluar la suma de Riemman para la funcin f(x) = x
2
+2 en el intervalo [-2, 2], la
particin es regular, tomando P = 8

Solucin:

Tomemos X
0
= -2 y X
n
= 2. Se toma
i
x
~
como el punto medio del i-simo
intervalo. Tambin:
2
1
8
) 2 ( 2
=

=
i
x X
i
= 0,5; con esto se obtienen 8
subintervalos, cuyos puntos medios son:



___________________
1
1826 Alemania 1866 Suiza.

=
=
n
i
i i p
x x f R
1
)
~
(

UNIVERSIDADNACIONALABIERTAYADISTANCIA UNAD
ESCUELADECIENCIASBSICAS,TECNOLOGAEINGENIERA
CONTENIDODIDCTICODELCURSO:100411ClculoIntegral


-1.75, -1.25, -0.75, -0.25, 0.25, 0.75, 1.25, 1.75.

Apliquemos la frmula de sumas de Riemman:

=
=
8
1
)
~
(
i
i i p
x x f R Entonces:

R
p
= [f(-1.75)+f(-1.25)+f(-0.75)+f(-0.25)+f(0.25)+f(0.75)+f(1.25)+f(1.75)] * 0.5

En la funcin se reemplaza: 0625 , 5 2 ) 75 , 1 ( ) 75 , 1 (
2
= + = = x f y as para los
dems.

R
p
= [5.0625 + 3.5625 + 2.5625 + 2.0625 + 2.0625 + 2.5625 + 3.5625 + 5.0625] *
0.5

R
p
= [25.50] * 0.5 = 13.25

Ejemplo No 2:

Evaluar la suma de Riemman para la funcin h(t) = t
3
2t, en el intervalo [1, 2]. La
particin es regular y los puntos muestra definidos son: 20 , 1
~
1
= x , 38 , 1
~
2
= x ,
68 , 1
~
3
= x , 92 , 1
~
4
= x

Solucin

Tenemos todos los insumos para hacer la suma correspondiente:

=
=
4
1
)
~
(
i
i i p
x x f R Entonces:
R
p
= [f(1.20) + f(1.38) + f(1.68) + f(1.92)] * 0.25

R
p
= [-0.672 0.131928 + 1.3816 + 3.2779] * 0.25
R
p
= [3.855] * 0.25 = 0.9637

Resolver el ejemplo anterior utilizando 8 subintervalos P = 8, definiendo el tamao
de cada subintervalo y el punto muestra de cada uno.



UNIVERSIDADNACIONALABIERTAYADISTANCIA UNAD
ESCUELADECIENCIASBSICAS,TECNOLOGAEINGENIERA
CONTENIDODIDCTICODELCURSO:100411ClculoIntegral

Leccin 7: rea bajo la curva
Concepto Intuitivo:
Para hallar el rea de una figura con lados rectos, la geometra plana (estudiada
en matemtica bsica) permite calcular dichas reas, por ejemplo rectngulos,
tringulos, paralelogramos, otros. Cuando la frontera de una figura es curva la
situacin es de un anlisis ms profundo, ya que se requiere mayor trabajo
matemtico. El gran matemtico de la antigedad ARQUIMEDES, propuso una
solucin consistente en que al considerar una sucesin de polgonos inscritos que
aproximen la regin curva, que puede ser ms y ms precisa, a medida que el
polgono aumenta el nmero de lados.




Cuando P tiende a infinito
( P ), el rea del polgono se
hace semejante a la del crculo.



Fig. No. 2 Polgonos inscritos.

Pero la genialidad de Arqumedes, tambin lo llevo a demostrar que con polgonos


circunscritos, se llegaba al mismo resultado.










UNIVERSIDADNACIONALABIERTAYADISTANCIA UNAD
ESCUELADECIENCIASBSICAS,TECNOLOGAEINGENIERA
CONTENIDODIDCTICODELCURSO:100411ClculoIntegral

Leccin 8: Estimacin por sumas finitas.

Para determinar cmo se halla el rea bajo la curva, utilizaremos el principio de los
polgonos inscritos y adems una de las funciones ms conocidas: f(x) = x
2
. El
proceso consiste en hallar el rea de la regin A ( R ) acotada por el intervalo [a,
b], para nuestro caso tomemos: [0, 2]

La particin P del intervalo [0, 2] en n subintervalos, cuya longitud x es:
n n n
x x
x
n
2 0 2
0
=

= Particin
regular.
Comencemos:
X
0
= 0
X
1
= X
0
+ x = x
X
2
= X
1
+ x = x + x = 2x
X
3
= X
2
+ x = 2x + x = 3x
M
X
i
= X
i-1
+ x = (i 1) x + x = ix
M
X
n-1
= (n-1) x
X
n
= nx Fig. No. 3 Particin.

Pero x = 2/n, entonces:

X
0
= 0, X
1
= 2/n, X
2
= 4/n, , X
i
= 2i/n,
, , X
n
= n(2/n) = 2

El rea de la regin R
i
es f(x
i-1
) x .

El rea total de la regin R
n
ser la suma de las reas de todos los rectngulos
inscritos en la curva.

x x f x x f x x f R A
n n
+ + + =

) ( ) ( ) ( ) (
1 1 0
L
Para la funcin que estamos analizando tenemos:

2
3 3
2
2
2
8 8 2
*
2
) ( i
n n
i
n n
i
x x x x f
i i
= =

= =

UNIVERSIDADNACIONALABIERTAYADISTANCIA UNAD
ESCUELADECIENCIASBSICAS,TECNOLOGAEINGENIERA
CONTENIDODIDCTICODELCURSO:100411ClculoIntegral


Luego:
[ ]


= + + + + =
6
) 1 2 )( 1 ( 8
) 1 ( 2 1 0
8
) (
3
2 2 2 2
3
n n n
n
n
n
R A
n
L

Revisar las propiedades de las sumatorias en el modulo de lgebra, Trigonometra
y Geometra analtica, unidad tres, donde puedes reforzar estos conceptos.
Luego:

+ =

+
=
2 3
2 3
1 3
2
3
4 3 2
6
8
) (
n n n
n n n
R A
n
Entonces:
2
3
4 4
3
8
) (
n n
R A
n
+ =


A medida que n se hace ms grande, entonces el rea de la suma de los
rectngulos inscritos es ms y ms aproximado al rea de la curva. Por
consiguiente:
3
8
3
4 4
3
8
) ( ) (
2
=

+ = =

n n
Lim R A Lim R A
n
n
n

NOTA: Realice la misma demostracin pero usando rectngulos circunscritos.


Leccin 9: Definicin
Sea f(x) una funcin definida en el intervalo cerrado [a, b] y continua en el intervalo
abierto (a, b). Si f(x) 0 en [a, b], el rea bajo la curva de f(x) en el intervalo
definido esta dado por:

( )

=

=
n
i
i
n
x x f Lim A
1

Ejemplo 1:
Calcular el rea bajo la curva de f(x) = 3x
2
x en el intervalo [1, 3].

UNIVERSIDADNACIONALABIERTAYADISTANCIA UNAD
ESCUELADECIENCIASBSICAS,TECNOLOGAEINGENIERA
CONTENIDODIDCTICODELCURSO:100411ClculoIntegral

Solucin:
Comencemos el proceso hallando
n n
x
2 1 3
=

=
1
0
= x
n
n
n
x x x
2 2
1
0 1
+
= + = + =
n
n
n n n
x x x
4 4
1
2
)
2
1 (
1 2
+
= + = + + = + =
n
n
n n n
n
x x x
6 6
1
2 4
2 3
+
= + = +

+
= + =
n
i n
n
i
x x x
i i
2 2
1
1
+
= + = + =


Ahora por la definicin:
[ ]



=

= =
n
i
i
i
n
n
i
i
n
x x x Lim x x f Lim A
1
2
1
3 ) (

+
=
n
i
n
n n
i n
n
i n
Lim A
1
2
2 2 2
3


Desarrollando las potencias y multiplicando, obtenemos:

+ +
=
n
i
n
n
i n
n
i ni n
n
Lim A
1
2
2 2
2 12 12 3 2



UNIVERSIDADNACIONALABIERTAYADISTANCIA UNAD
ESCUELADECIENCIASBSICAS,TECNOLOGAEINGENIERA
CONTENIDODIDCTICODELCURSO:100411ClculoIntegral

Aplicando las propiedades de las sumatorias, tenemos:

=

+ +
=
n
i
n
i
n
n
i n
n n
i ni n
n
Lim A
1 1
2
2 2
2 2 12 12 3 2



=

+ + =
n
i
n
i
n
i
n n
i
n
i
n n
Lim A
1 1
2
2
2
1
2 12 12
3
2



= = =

+ + =
n
i
n
i
n
i
n
i
n
n
n
i
n
i
n
n
n
Lim A
1 1
2
2
1
2
*
2 12 12
3
2

Recordemos las propiedades de las sumatorias.

( ) ( )

+ +
+

+
+ =

6
1 2 12
2
12
3
2
2
2
2
n n n
n
n n
n
n
n
Lim A
n

+
+
2
2
*
2
2
n n
n
n
n

+
+

+ +
+
+
+ =

n
n n
n
n n
n n n
n
n n
n
n
Lim A
n
2
2
2 3 2
2 2 6 4 6 6
3
2


UNIVERSIDADNACIONALABIERTAYADISTANCIA UNAD
ESCUELADECIENCIASBSICAS,TECNOLOGAEINGENIERA
CONTENIDODIDCTICODELCURSO:100411ClculoIntegral

+ + + + + =

n n n n
Lim A
n
2
2 2
4 12
8
12
12 6
2

+ + =

2
4 2 24
4 26
n n n
Lim A
n

+ + =

2
4 22
22
n n
Lim A
n


Aplicando lmite:
22 0 0 22 = + + = A
Unidades cuadradas.

UNIVERSIDADNACIONALABIERTAYADISTANCIA UNAD
ESCUELADECIENCIASBSICAS,TECNOLOGAEINGENIERA
CONTENIDODIDCTICODELCURSO:100411ClculoIntegral


EJERCICIOS LECCION No. 4:


1. Demostrar que el rea bajo la curva para la funcin
2
2 2 x x y = en el intervalo
[0, 1] es 1/3.

SUGERENICA: Siga el procedimiento anterior, teniendo en cuenta las
propiedades de las sumatorias.


Hallar el rea del polgono circunscrito para la funcin propuesta:


2. f(x) = x + 1 donde a = -1 y b = 2 Con particin regular.


3. f(x) = x
2
+ 4 donde a = 2 y b = 4 Con particin regular.


4. g(x) = x
3
donde a = 0 y b = 2 Con particin regular.


Para las funciones dadas:
Determinar los puntos de evaluacin, correspondientes a los puntos medios
de cada subintervalo dado segn el valor de n.
Graficar la funcin de los rectngulos que la aproximan.
Calcular la suma de Riemman

5. f(x) = sex(x) [0, ] y n = 4

6. g(x) = x
3
1 [1, 2] y n = 4

7. 2 ) ( + = x x h [1, 4] y n = 6

8.
x
x
x P
1 2
) (

= [2, 4] y n = 10




UNIVERSIDADNACIONALABIERTAYADISTANCIA UNAD
ESCUELADECIENCIASBSICAS,TECNOLOGAEINGENIERA
CONTENIDODIDCTICODELCURSO:100411ClculoIntegral

Leccin 10: Integral definida.
Conocidos y estudiados los conocimientos sobre Sumas de Riemman y reas
bajo la curva, podemos hacer una definicin formal sobre la integral definida.
DEFINICIN:
Sea f(x) una funcin definida en el intervalo cerrado [a, b], la integral definida de
f(x) que va de a hasta b se define como:

( ) ( )

=

=
n
i
i
n
b
a
x x f Lim dx x f
1
Llamada tambin la Integral de Riemman
Donde:
a = Lmite Inferior
b = Lmite Superior
f(x) = El integrando; o sea, la funcin que se va a integrar.
dx = Diferencial de la variable.
Analizando un poco el lmite de la sumatoria, igual que en el caso de la derivacin.

=
n
i
i
p
L x x f Lim
1
0
) (

Esto significa que dado un > 0, tan pequeo como se quiera, existe un > 0 tal
que:
=

=
n
i
i
L x x f
1
) (

Para todas las sumas de Riemman

x x f
i
) ( de la funcin definida en el
intervalo dado, si la norma p de la particin asociada, es menor que , se dice
que el lmite dado existe y es L.
Surge la pregunta: Qu funciones son integrables? La respuesta es que NO
todas las funciones son integrables en un intervalo cerrado I.

UNIVERSIDADNACIONALABIERTAYADISTANCIA UNAD
ESCUELADECIENCIASBSICAS,TECNOLOGAEINGENIERA
CONTENIDODIDCTICODELCURSO:100411ClculoIntegral

Asociado al caso de lmite, se requiere que la suma de Riemman tenga lmite, ya
que hay casos donde esta suma se puede hacer muy grande, como es el caso de:

n
i i
n
x
Lim
1
2
1


Existen adems funciones acotadas que pueden no ser integrables, por el grado
de complejidad de la misma, como es el caso de:


2
0
2
dx e
x

Para esto existe un teorema de integrabilidad que nos garantiza las funciones
integrables en un intervalo cerrado I, su demostracin NO est al alcance de este
nivel ya que requiere clculo avanzado.

UNIVERSIDADNACIONALABIERTAYADISTANCIA UNAD
ESCUELADECIENCIASBSICAS,TECNOLOGAEINGENIERA
CONTENIDODIDCTICODELCURSO:100411ClculoIntegral


CAPITULO 3: Teoremas

Introduccin
En este captulo se presentan tres teoremas o afirmaciones que se pueden
demostrar como verdaderas dentro un contexto lgico, esos tres teoremas nos
ayudan a comprender los conceptos empleados en el clculo integral.
El teorema del valor medio, de la integrabilidad, primer y segundo teorema
fundamental del clculo y el teorema de simetra, los cuales pasamos a
comprender en el siguiente espacio.

Leccin 11: Teorema de integrabilidad.
Si f(x) es acotada en el intervalo cerrado [a, b] y si f(x) es continua excepto en un
nmero finito de puntos, entonces f(x) es integrable en [a, b]. En particular si f(x)
es continua en todo el intervalo, entonces es integrable en [a, b].
Consecuencia de este teorema podemos ver que las funciones polinmicas, seno
y coseno, son integrables en todo el intervalo cerrado I. Las funciones racionales
lo son en I siempre y cuando dicho intervalo no contenga puntos en donde el
denominador es cero.
Ahora podemos hacer la siguiente relacin como conclusin de lo que venimos
analizando:
rea bajo la curva de y = f(x) en el intervalo cerrado [a, b] es equivalente a

b
a
dx x f ) (

PROPIEDADES DE LA INTEGRAL DEFINIDA:
Las propiedades aplicadas a la integral indefinida, tambin son aplicables a las
integrales definidas. Veamos algunas.

UNIVERSIDADNACIONALABIERTAYADISTANCIA UNAD
ESCUELADECIENCIASBSICAS,TECNOLOGAEINGENIERA
CONTENIDODIDCTICODELCURSO:100411ClculoIntegral

1.

=
b
a
dx x f 0 ) ( Para a = b
2.

=
b
a
a
b
dx x f dx x f ) ( ) ( Para a < b
3.

+ =
b
a
c
a
b
c
dx x f dx x f dx x f ) ( ) ( ) ( Para a < c < b
4. [ ]

=
b
a
b
a
b
a
dx x g dx x f dx x g x f ) ( ) ( ( ) (
5.

=
b
a
b
a
dx x f K dx x Kf ) ( ) (
6.

=
b
a
a b K Kdx ) (

7. Si f(x) y g(x) son funciones integrables en el intervalo I = [a, b] y si f(x) g(x)
Para todo x en [a, b], entonces:

b
a
b
a
dx x g dx x f ) ( ) (
Leccin 12: Valor medio de una funcin.
El concepto de valor medio lo conocemos muy bien, por los principios de
Estadstica, pero en este caso vamos a calcular el valor promedio de una funcin
f(x) en un intervalo cerrado I. Para este caso escogemos una muestra de puntos
en el intervalo I, construyendo la Particin correspondiente, donde: x
0
< x
1
< x
2

< x
n
; adems, x
0
= a y x
n
= b. La diferencia entre los puntos es:
n
a b
x

=
El valor promedio de la funcin f(x) est dado por el promedio de los valores de la
funcin en x
1
, x
2
, x
n
:
[ ]

=
= + + + + =
n
i
i n
x f
n
x f x f x f x f
n
x f
1
3 2 1
) (
1
) ( ... ) ( ) ( ) (
1
) (

UNIVERSIDADNACIONALABIERTAYADISTANCIA UNAD
ESCUELADECIENCIASBSICAS,TECNOLOGAEINGENIERA
CONTENIDODIDCTICODELCURSO:100411ClculoIntegral

Si multiplicamos y dividimos por b a tenemos:

=

=
n
a b
x f
a b
x f
n
i
i
1
) (
1
) (
Recordemos que:
n
a b
x

= , luego:
x x f
a b
x f
n
i
i

=

=1
) (
1
) (
Corresponde a la suma de Riemman.
DEFINICIN:
Para la funcin f(x) integrable en [a, b] y sabiendo que la suma de Riemman tiene
lmite:
( ) ( ) ( )


=


=

=
n
i
b
a
i
n
dx x f
a b
x x f
a b
Lim x f
1
1 1

Ejemplo 1:
Hallar el valor promedio de la funcin sen(x) en [0, ]
Solucin:
Aplicando la definicin tenemos:


=

0
) (
0
1
) (
1
) ( dx x sen dx x f
a b
x f
b
a

( ) [ ] ) 0 cos( ( ) cos(
1
) cos(
1
) (
1
) (
0
0
= = =



x dx x sen x f

[ ]

2
1 1
1
) ( = + = x f

El proceso requiere la aplicacin del teorema fundamental del clculo, el cual
estudiaremos en seguida.
Ejemplo 2:
Cual ser el valor promedio de la funcin f(x) = x
2
2 en el intervalo [0, 4]

UNIVERSIDADNACIONALABIERTAYADISTANCIA UNAD
ESCUELADECIENCIASBSICAS,TECNOLOGAEINGENIERA
CONTENIDODIDCTICODELCURSO:100411ClculoIntegral

Solucin:
Al igual que en el caso anterior, con la aplicacin de la frmula para valor
promedio de la funcin:
( )
4
0
3
4
0
2
2
3
1
4
1
2
0 4
1
) (
1
) (

=

x x dx x dx x f
a b
x f
b
a

3
10
3
40
4
1
0 8
3
64
4
1
2
3
1
4
1
) (
4
0
3
=

= x x x f
3
10
) ( = x f


EJERCICIOS:
1. Hallar el valor promedio para la funcin f(x) = 4x
3
en el intervalo [1, 3]
2. Cual ser el valor promedio de la funcin
16
) (
2
+
=
x
x
x g en el intervalo [0, 3]
3. Determinar el valor medio de la funcin: g(x) = sen
2
(x) cos(x) para el intervalo
[0, /2]
4. Cual ser el valor promedio de la funcin f(x) = cos(x) en el intervalo [0, /2]

Leccin 13: Primer teorema fundamental del clculo
Para enunciar el teorema, analicemos la siguiente situacin: Sea A(x) el rea bajo
la curva de la funcin f(t) a dicha funcin se le llama funcin acumulada, ya que va
acumulando el rea bajo la curva dada t = a hasta t = x. donde x > 1.
Sabemos que:

=
x
a
dt t f x A ) ( ) (

UNIVERSIDADNACIONALABIERTAYADISTANCIA UNAD
ESCUELADECIENCIASBSICAS,TECNOLOGAEINGENIERA
CONTENIDODIDCTICODELCURSO:100411ClculoIntegral

Por otro lado, sabemos por definicin de reas bajo la curva que:

=

=
n
i
i
n
x x f Lim x A
1
) ( ) (

Al relacionar las ecuaciones anteriores:

=
=

x
a
n
i
i
n
dt t f x x f Lim ) ( ) (
1


Ahora definamos a B(x) como el lmite de la sumatoria, de tal manera que
) (x f
dx
dB
= Luego: ) ( ) ( x f dt t f
dx
d
x
a
=



TEOREMA: Sea f(x) una funcin continua en el intervalo cerrado [a, b] y sea x un
punto en (a, b), entonces:

Se debe anotar que x es variable y que la tasa de acumulacin en t = x es igual al
valor de la funcin f(x) que se est acumulando en t = x.

Demostracin:

Por la definicin de derivada:

+
=

+

x
a
x x
a
x x
dt t f dt t f
x
Lim
x
x F x x F
Lim x F ) ( ) (
1 ) ( ) (
) ( '
0 0



) ( ) ( x f dt t f
dx
d
x
a

UNIVERSIDADNACIONALABIERTAYADISTANCIA UNAD
ESCUELADECIENCIASBSICAS,TECNOLOGAEINGENIERA
CONTENIDODIDCTICODELCURSO:100411ClculoIntegral


+

+


=

x x
x
x
x
a
x x
a
x
dt t f
x
Lim dt t f dt t f
x
Lim ) (
1
) ( ) (
1
0 0


Si observamos cuidadosamente la ltima expresin, podemos deducir que
corresponde a lmite del valor promedio de f(x) en el intervalo [x, x + x]. Como
x > 0, por teorema de valor medio:

+
=

x x
x
c f dt t f
x
) ( ) (
1
Donde x < c < x + x

Pero cuando x tiende a cero, entonces c tiende a x; adems, f(x) es continua.

) ( ) ( ) (
1
) ( '
0 0
x f c f Lim dt t f
x
Lim x F
x
x x
a
x
= =

=

+




Este teorema en su concepto expresa que toda funcin f(x) continua en un
intervalo cerrado, tiene antiderivada.

Ejemplo 1:
Desarrollar:

x
dt t
dx
d
1
4

Solucin:

Por la definicin del teorema:
4
1
4
x dt t
dx
d
x
=

UNIVERSIDADNACIONALABIERTAYADISTANCIA UNAD
ESCUELADECIENCIASBSICAS,TECNOLOGAEINGENIERA
CONTENIDODIDCTICODELCURSO:100411ClculoIntegral


Ejemplo 2:

Dado: ( )

+ =
x
dt t t x F
1
2
2 4 ) ( Hallar F(x).

Solucin:

El integrado por definicin es F(x) = f(x) entonces: F(x) = x
2
+ 4x 2
Si lo resolvemos por otro lado, tenemos: ( )

+ =
x
dt t t
dx
d
dx
dF
1
2
2 4 por definicin del
teorema: 2 4
2
+ = x x
dx
dF

Ejemplo 3:
Si

=
2
1
) cos( ) (
x
dt t x P Calcular P(x).

Solucin:
Como el lmite superior tiene potencia, hacemos cambio de variable. U = x
2
, luego:

=
u
dt t x P
1
. ) cos( ) ( Por la regla de la cadena:
dx
du
dt t
du
d
dx
dP
dx
du
du
dP
dx
dP
u
* ) cos( *
1

= =

Desarrollando:
x u
dx
du
u
dx
dP
2 * ) cos( * ) cos( = = recordemos que u = x
2
en este contexto.
) cos( 2 ) ( '
2
x x x P =


UNIVERSIDADNACIONALABIERTAYADISTANCIA UNAD
ESCUELADECIENCIASBSICAS,TECNOLOGAEINGENIERA
CONTENIDODIDCTICODELCURSO:100411ClculoIntegral

Ejemplo 4
Sea ( )

=
2
1
4 2 ) (
x
dt t x H Hallar H(x).
Solucin:
Hacemos cambio de variable as: u = x
2
ahora:
( ) ( ) x u
dx
du
dt t
dx
d
dx
dH
u
2 * 4 2 * ) 4 2 (
1
=

=

Reemplazando u tenemos
( ) ( ) x x x x
dx
dH
8 4 2 * 4 2
3 2
= =
Por consiguiente:

x x
dx
dH
8 4
3
=


Leccin 14: Segundo teorema fundamental del clculo.

En clculo el estudio de los lmites es fundamental, dos lmites muy importantes en
clculo son:

+
=

x
x f x x f
Lim x f
x
) ( ) (
) ( '
0
y
x x f Lim
i
n


) (

Por medio del teorema fundamental nmero uno, se estudio la relacin que tienen
estos dos lmites, fundamental para resolver integrales definidas.
La existencia de la antiderivada, lo garantiza el primer teorema fundamental del
clculo, la evaluacin de dichas integrales se garantiza por medio del segundo
teorema fundamental.

UNIVERSIDADNACIONALABIERTAYADISTANCIA UNAD
ESCUELADECIENCIASBSICAS,TECNOLOGAEINGENIERA
CONTENIDODIDCTICODELCURSO:100411ClculoIntegral



TEOREMA:

Sea f(x) una funcin contina en un intervalo definido, por consiguiente es
integrable en el intervalo cerrado [a, b], sea P(x) una antiderivada de f(x) en el
intervalo dado, entonces:



Demostracin:
La demostracin requiere los conocimientos de teoremas y definiciones
estudiadas anteriormente, por lo cual se debe tener presente estos aspectos.
Sea la funcin

=
x
a
dt t f x G ) ( ) ( para x en el intervalo [a, b], sabemos que
) ( ) ( ' x f x G =
Para todo x en [a, b], luego G(x) es una antiderivada de f(x), pero P(x) es tambin
antiderivada de f(x). Por el teorema de antiderivada, sabemos:
P(x) = G(x), donde P(x) y G(x) solo difieren por una constante, luego para todo x
en [a, b]: P(x) = G(x) + C, para P(x) y G(x) continuas en el intervalo dado, luego:
P(a) = G(a) + C y P(b) = G(b) + C en el intervalo cerrado definido.
Para

=
= =
a x
a
dt t f a G 0 ) ( ) ( Recuerdas?
P(a) = G(a) + C saber porque verdad!

P(a) = 0 + C entonces: P(a) = C, por lo tanto:
P(b) P(a) = [G(b) + C] C = G(b). Luego al igual que G(a), podemos decir:

=
b
a
a P b P dx x f ) ( ) ( ) (

UNIVERSIDADNACIONALABIERTAYADISTANCIA UNAD
ESCUELADECIENCIASBSICAS,TECNOLOGAEINGENIERA
CONTENIDODIDCTICODELCURSO:100411ClculoIntegral

=
=
b x
a
dt t f b G ) ( ) ( Por consiguiente:

=
b
a
dx x f a P b P ) ( ) ( ) (
As queda demostrado el teorema.
Esta misma demostracin se puede hacer por las sumas de Riemman, veamos:
Primero participamos el intervalo [a, b] en: x
o
, x
1
, x
2
, , x
n


donde x
o
= a y x
n
= b,
adems: x = x
i
x
i-1
, como x es el tamao de cada subintervalo, entonces:
n
a b
x

= para i = 1, 2, 3, , n Ahora:
P(b) P(a) = [P(x
1
) P(x
o
)] + [P(x
2
) P(x
1
)] + + [P(x
n
P(x
n-1
)] resumiendo:
[ ]

=

=
n
i
i i
x P x P a P b P
1
1
) ( ) ( ) ( ) (
Como P(x) es una antiderivada de f(x) derivable en (a, b) y continua en [a, b], por
el teorema del valor medio
x c f x x c P x P x P
i i i i i i
= =

) ( ) )( ( ' ) ( ) (
1 1
para c
i
(x
i-1
, x
i
) donde i = 1,
2, 3, Por asociacin de las dos ecuaciones anteriores:
[ ]

= =

= =
n
i
i
n
i
i i
x c f x P x P a P b P
1 1
1
) ( ) ( ) ( ) ( ) ( Si tomamos limite a ambos lados de
la ecuacin cuando n tiende a infinito, obtenemos:

= =

=

b
a
n
n
i
i
n
dx x f a P b P Lim x c f Lim ) ( )) ( ) ( ( ) (
1

Por consiguiente:

=
b
a
dx x f a P b P ) ( ) ( ) (

UNIVERSIDADNACIONALABIERTAYADISTANCIA UNAD
ESCUELADECIENCIASBSICAS,TECNOLOGAEINGENIERA
CONTENIDODIDCTICODELCURSO:100411ClculoIntegral

Ejemplo 1:
Aplicar el segundo teorema fundamental del clculo para resolver:

b
a
xdx
Solucin:
( )

= = =
=
=
b
a
b x
a x
a b
a b x
xdx
2 2
2 2 2
2
1
2 2 2

Ejemplo 2:
Resolver la integral: ( )dx x x


2
0
3
4
Solucin:
( ) ( ) ( ) ( ) ( )

2 4 2 4
2
0
2 4
2
0
3
0 2 0
4
1
2 2 2
4
1
2
4
1
4 x x dx x x

( ) 4 8 4 8
4
16
4
2
0
3
= = =

dx x x


Ejemplo 3:
Demostrar que:
12
47 1
4
1
2
=

dx
x
x

Solucin:

Como
2
1
x
x es continua en [1, 4], se puede aplicar el teorema fundamental,
luego:

UNIVERSIDADNACIONALABIERTAYADISTANCIA UNAD
ESCUELADECIENCIASBSICAS,TECNOLOGAEINGENIERA
CONTENIDODIDCTICODELCURSO:100411ClculoIntegral


+ = + =



4
1
4
1
1
4
1
2
2
2
3
2
1
3
2
) (
1
x x dx x x dx
x
x
Evaluando:
( ) ( ) ( ) ( )
4
1
3
11
3
5
4
1
3
16
1 1
3
2
4 4
3
2 1
4
1
1 1
2
2
3
2
3
+ = + =

+ =


dx
x
x

12
47
4
1
3
11 1
4
1
2
= + = =

dx
x
x


Ejemplo 4:
Hallar el valor de:

2
0
) (

dx x sen

Solucin:
La funcin seno es continua en el intervalo propuesto, luego se puede integral, por
medio del teorema fundamental.
)) 0 cos( ( ) cos( ) cos( ) (
2
0
0
2
2
= =

x dx x sen

1 1 0 ) (
2
0
= + =

dx x sen






UNIVERSIDADNACIONALABIERTAYADISTANCIA UNAD
ESCUELADECIENCIASBSICAS,TECNOLOGAEINGENIERA
CONTENIDODIDCTICODELCURSO:100411ClculoIntegral

Leccin 15: Teorema de simetra.
Si f(x) es una funcin par, entonces:

=

a a
a
dx x f dx x f
0
) ( 2 ) (

Si f(x) es una funcin impar, entonces:
0 ) ( =

a
a
dx x f

Demostracin:
Vamos a demostrar la primera parte del teorema, el segundo se deja como
ejercicio.


+ =

a
a
a
a
dx x f dx x f dx x f
0
0
) ( ) ( ) (
Ahora hacemos una sustitucin u = -x, luego
du = -dx. Por definicin, si f(x) es par. Se cumple: f(-x) = f(x), entonces:



= =
0 0 0
) ( ) )( ( ) (
a a a
du u f dx x f dx x f
Luego:

=
a a
dx x f du u f
0 0
) ( ) (
Por lo tanto:

= + =

a a a a
a
dx x f dx x f dx x f dx x f
0 0 0
) ( 2 ) ( ) ( ) (

EJERCICIOS:
1. Escribir las siguientes integrales como una sola:
a-)

+
3
2
2
0
) ( ) ( dx x f dx x f
b-)

+
1
2
2
0
) ( ) ( dx x f dx x f

UNIVERSIDADNACIONALABIERTAYADISTANCIA UNAD
ESCUELADECIENCIASBSICAS,TECNOLOGAEINGENIERA
CONTENIDODIDCTICODELCURSO:100411ClculoIntegral

2. Hallar

4
0
. ) ( dx x f donde:

+
<
=
1 2
1 2
) (
2
x si x
x si x
x f
3. Calcular

4
0
. ) ( dx x f donde:

+
<
=
3 1
3 2
) (
2
x si x
x si x
x f
4: Desarrollar: ( ) ( )

+
1
0
10
2
2 1 dx x x
5. Hallar
( )

dx x x sex
2
) cos( ) (

6. Para un gas ideal, la presin es inversamente proporcional al volumen, el
trabajo requerido para aumentar el volumen de un gas particular de V = 2 a V = 4
esta dado por la siguiente expresin:

2
1
) (
V
V
dV V P donde la constante de
proporcionalidad para este caso es de 12. Cual ser el valor de la integral.

7. La temperatura T en una regin particular, est dada por la funcin
T(t) = 75 20cos(/6)t donde t = tiempo en meses. Estimar la temperatura
promedio
Durante todo el ao.

UNIVERSIDADNACIONALABIERTAYADISTANCIA UNAD
ESCUELADECIENCIASBSICAS,TECNOLOGAEINGENIERA
CONTENIDODIDCTICODELCURSO:100411ClculoIntegral


ACTIVIDADES DE AUTOEVALUACIN DE LA UNIDAD
PREGUNTAS DE SELECCIN MLTIPLE CON NICA RESPUESTA
A continuacin, usted encontrar preguntas que se desarrollan en torno a un
enunciado, problema o contexto, frente al cual, usted debe seleccionar aquella que
responde correctamente a la pregunta planteada entre cuatro opciones
identificadas con las letras A, B, C, D. Una vez la seleccione, mrquela en su hoja
de respuestas rellenando el valo correspondiente.
1. En la suma de Riemman la funcin se aplica sobre los puntos muestra, ste
representa:
A. El valor representativo del subintervalo
B. El valor representativo del intervalo
C. El valor representativo del rea
D. El valor representativo de la ordenada

2. La resolucin de integrales indefinidas originan
A. Una funcin
B. Un escalar
C. Infinito
D. Cero

3. La resolucin de integrales definidas originan
A. Un escalar
B. Otra funcin
C. Cero
D. Uno

UNIVERSIDADNACIONALABIERTAYADISTANCIA UNAD
ESCUELADECIENCIASBSICAS,TECNOLOGAEINGENIERA
CONTENIDODIDCTICODELCURSO:100411ClculoIntegral

4. Sea f(x) una funcin continua en el intervalo cerrado [a, b] y sea x un punto
en (a, b), entonces:





La definicin dada corresponde a:

A. El primer teorema fundamental del clculo
B. El segundo teorema fundamental del clculo
C. El teorema del valor medio de una funcin
D. La definicin de integral impropia
5. Al resolver

2
0
) 2 tan(

dx x
Su resultado es:
A. Diverge
B. 1/2
C. 2
D. 0
6. Al resolver
dy y y

+
2
0
1
se obtiene
A. 5,276
B. 2,789
C. 1,432
D. 10,450
) ( ) ( x f dt t f
dx
d
x
a

UNIVERSIDADNACIONALABIERTAYADISTANCIA UNAD
ESCUELADECIENCIASBSICAS,TECNOLOGAEINGENIERA
CONTENIDODIDCTICODELCURSO:100411ClculoIntegral

7. Al resolver
dx e
x

+
1
0
1 3
se obtiene
A. 17,293
B. 20,287
C. 26.456
D. 10,345
8. Al resolver
dx x sen x
o

) ( ) ( cos
2 2
se obtiene
A. 0,3927
B. 2,453
C. 0,679
D. 7,895
9. Al resolver


dx
x x
1
se obtiene
A. 2Ln(3)
B. 4Ln(4)
C. 5Ln(10)
D. 7Ln(2)
10. Al resolver
dx x
x

2
3 *
se obtiene
A. 2 / 2Ln(3)
B. 5 / 3Ln(2)
C. 3 / Ln(7)
D. 7 / 3Ln(6)

UNIVERSIDADNACIONALABIERTAYADISTANCIA UNAD
ESCUELADECIENCIASBSICAS,TECNOLOGAEINGENIERA
CONTENIDODIDCTICODELCURSO:100411ClculoIntegral


HOJA DE RESPUESTAS.

A B C D
1
2
3
4
5
6
7
8
9
10

UNIVERSIDADNACIONALABIERTAYADISTANCIA UNAD
ESCUELADECIENCIASBSICAS,TECNOLOGAEINGENIERA
CONTENIDODIDCTICODELCURSO:100411ClculoIntegral

FUENTES DOCUMENTALES DE LA UNIDAD 1

RONDON, J.E (2007) Calculo Integral. Primera edicin, UNAD Ciencias bsicas
PURCELL, E (2001) Clculo, Pearson Education: Prentice hall, Octava Edicin,
Mxico.
THOMAS Y FINNEY (1987). Clculo con Geometra Analtica Vol. 1. Edicin
sexta, Addison Wesley Iberoamericana. Mxico.
STEWART, J. (2001) Clculo de una Variable. Thomsom-Learning. Cuarta edicin,
Bogot.
LARSON, R. Y HOSTETLER, R. (1998) Clculo Vol. 1, Mc Graw Hill, sexta
edicin, Mxico.
SMITH, R. Y MINTON, R. (2002) Clculo Vol. 1. Segunda Edicin, Mc Graw Hill,
Bogot.
BAUM Y MILLES. (1992). Clculo Aplicado. Limusa, Mxico.
LEYTOLD, L. (1987) El Clculo con Geometra Analtica. Harla, Mxico.
PITA, C. (1998) Clculo de una Variable. Pearson educacin, Mxico.
DE BURGOS, J. (2007) Clculo infinitesimal de una Variable. McGraw Hill, Madrid.

FUENTES DOCUMENTALES DE LA INTERNET

http://www.xtec.cat/~jlagares/integral.esp/integral.htm
http://thales.cica.es/rd/Recursos/rd97/Problemas/541pIntegral.html
http://sigma.univalle.edu.co/index_archivos/calculo1y2/formulasdecalculo1y2.pdf
http://www.matematicasbachiller.com/temario/calculin/index.html
http://es.wikipedia.org/wiki/Teorema_fundamental_del_c%C3%A1lculo
http://www.aulafacil.com/matematicasintegrales/curso/Temario.htm

UNIVERSIDADNACIONALABIERTAYADISTANCIA UNAD
ESCUELADECIENCIASBSICAS,TECNOLOGAEINGENIERA
CONTENIDODIDCTICODELCURSO:100411ClculoIntegral
http://www.monografias.com/trabajos10/historix/historix.shtml
http://www.fata.unam.mx/tecnologia/material/sem01/Calculo_I_Historia_1.pdf
http://www.uam.es/personal_pdi/ciencias/barcelo/histmatem/calculo/calculo.html
http://integrals.wolfram.com/index.jsp?expr=x%5E2*%28x4%29%5E0.5&random=false
http://www.dma.fi.upm.es/docencia/primerciclo/calculo/tutoriales/integracion/
http://www.matematicasypoesia.com.es/ProbIntegral/problema110.htm
http://usuarios.iponet.es/ddt/logica1.htm

UNIVERSIDADNACIONALABIERTAYADISTANCIA UNAD
ESCUELADECIENCIASBSICAS,TECNOLOGAEINGENIERA
CONTENIDODIDCTICODELCURSO:100411ClculoIntegral


UNIDAD 2: TCNICAS DE INTEGRACIN.

Introduccin:


La primera forma de desarrollar integrales, es por medio de las integrales
inmediatas, donde se resuelven utilizando el principio de la antiderivada. Como

+ = c x dx

+ = c kx kdx para k = constante


c
n
x
dx x
n
n
+
+
=
+

1
1

+ = c x Ln dx
x
) (
1

La tcnica de sustitucin por cambio de variable, se utiliza cuando la funcin que
conforma el integrando es tal que una parte es la derivada de la otra parte y las
dos estn en forma de producto. Las condiciones bsicas para establecer que se
puede aplicar una sustitucin es una buena observacin de la funcin a integrar y
algo de perspicacia matemtica. Cuando el integrado presenta radicales, se
puede presentar problemas para resolver la integral, la racionalizacin puede ser
un camino para superar dicho problema. En el mundo matemtico, cientfico y
otros, se presentan casos donde la integral es un Producto de Funciones, casos
donde se aplica la tcnica llamada integracin por partes. En muchas ocasiones
se ha manifestado que toda regla de derivacin da origen a una regla de
integracin. La integracin por partes esta relacionada con la regla de la cadena.
La sustitucin trigonomtrica, es una tcnica que se puede utilizar cuando en el
integrando se presentan expresiones como:
2 2
x a ,
2 2
x a + ,
2 2
a x ;
siendo a > 0. Por un teorema de lgebra avanzada se afirma que toda fraccin
racional; es decir, el cociente de dos polinomios, se puede descomponer en suma
de fracciones racionales ms simples. Para desarrollar el mtodo de fracciones
parciales, se debe tener en cuenta: Para la fraccin
) (
) (
) (
x g
x f
x p = con g(x) 0 sea
una fraccin racional propia; es decir, f(x) debe tener menor grado que g(x) y por
otro lado, que g(x) se pueda descomponer en factores primos. Tericamente
cualquier polinomio con coeficientes reales se puede escribir como producto de
factores lineales reales y / o factores cuadrticos, es posible que obtenerlos no

UNIVERSIDADNACIONALABIERTAYADISTANCIA UNAD
ESCUELADECIENCIASBSICAS,TECNOLOGAEINGENIERA
CONTENIDODIDCTICODELCURSO:100411ClculoIntegral

sea tarea fcil. Descomposicin En Factores Lineales Simples, Descomposicin
En Factores Lineales Repetidos, Descomposicin En Factores Cuadrticos.


Justificacin:


Para poder desarrollar las integrales debemos manejar bien las derivadas, realizar
muchos ejercicios sobre un papel, tener algo de fortuna, conocer algunas
integrales directas y aplicar las tcnicas de integracin que analizaremos en esta
unidad.

Al resolver una integral se pueden presentar dos casos:

Necesitamos obtener una antiderivada si la integral es indefinida, actividad
que nos ocupa en esta unidad y
Encontrar un numero (escalar) si la integral es definida

Debido a la complejidad de las aplicaciones, de los ejercicios de aplicacin y de
los mimos problemas tericos, aparecen integrales que no es posible solucionar
por los teoremas bsicos de las antiderivadas; por lo tanto requerimos de tcnicas
o metodologas apropiadas para su solucin, tal como vamos a detallarlas en las
siguientes lecciones.


Para esta SEGUNDA UNIDAD tenemos tres captulos en los cuales tratamos las
diferentes tcnicas de integracin bsicas para el entrenamiento de los
estudiantes que estn tomando el curso.


Intencionalidades formativas:

Que los estudiantes se familiaricen con los mtodos de integracin.
La idea central de la UNIDAD es que los estudiantes al enfrentarse con
cualquier tipo de integral, adquieran las habilidades necesarias para su
solucin, sin aprenderse de memoria los mtodos de integracin.
La mejor manera de solucionar integrales es realizando ejercicios, los
cuales se encuentran al final de cada leccin, al final de la unidad o en la
bibliografa y cibergrafia recomendada.


Presentamos un cuadro con el resumen del contexto terico de esta unidad

UNIVERSIDADNACIONALABIERTAYADISTANCIA UNAD
ESCUELADECIENCIASBSICAS,TECNOLOGAEINGENIERA
CONTENIDODIDCTICODELCURSO:100411ClculoIntegral

Denominacin de
los captulos
CAPITULO 1: MTODOS de integracin I.

CAPITULO 2 Mtodos de integracin II

CAPITULO 3 Mtodos de integracin III

Asimilacin de
conceptos

Apropiarse de los mtodos de integracin que estn al
alcance de este mdulo, ver sus ventajas y desventajas y
aprender a manejarlos.

Conceptos

Los mtodos de integracin se presentan de una manera
sencilla y de menor dificultad a mayor dificultad, para
facilitarle su asimilacin al lector.

Competencias

De conocimientos
Adquirir las tcnicas propias de los mtodos de
integracin de la unidad.
Adquirir conocimiento mediante la realizacin del
mayor nmero posible de ejercicios.

Contextuales:
Adquirir los conocimientos propios de los mtodos de
integracin.

Los estudiantes deben desarrollar habilidades para
aplicar los conocimientos adquiridos en la solucin de
problemas prcticos.

Comunicativas:
Adquirir el manejo de los elementos involucrados en
los diferentes mtodos de solucin de integrales.
Interpretar y entender la diferente simbologa y su
aplicacin.
Adquirir facilidad de expresin y vencer el miedo en
la interaccin con las NTIC

Valorativas:
Adoptar, identificar y practicar lo valores de la UNAD.
Adquirir capacidad de valoracin y tolerancia con
nuestros compaeros virtuales o presenciales.

UNIVERSIDADNACIONALABIERTAYADISTANCIA UNAD
ESCUELADECIENCIASBSICAS,TECNOLOGAEINGENIERA
CONTENIDODIDCTICODELCURSO:100411ClculoIntegral


CAPITULO 4: Mtodos de integracin I

Introduccin
El teorema fundamental del clculo se puede aplicar bajo la condicin de que la
funcin sea continua en el intervalo de integracin. Por lo cual, cuando vamos a
integral lo primero que debemos observar es que se verifique el teorema. Existen
casos en que el teorema NO se cumple, dichas situaciones son las que
abordaremos en este aparte del curso.

( ) ( )

=
t
a
b t
b
a
dx x f Lim dx x f


Leccin 16: Integrales impropias con integrando discontinuo.







Fig. No. 5 Integral impropia.
La funcin que observamos es dada por la ecuacin:
2
1
) (
x
x f = y deseamos
integrarla en el intervalo [1, -2].

UNIVERSIDADNACIONALABIERTAYADISTANCIA UNAD
ESCUELADECIENCIASBSICAS,TECNOLOGAEINGENIERA
CONTENIDODIDCTICODELCURSO:100411ClculoIntegral

Sin pensarlo dos veces lo que haramos es:

= = =
1
2
1
2
2
1
2
2
2
3
1
x
dx x
x
dx

Obviamente la respuesta NO es correcta. Por qu?

El problema requiere que recordemos dos trminos: Continuidad y Acotacin.
La integral que estamos analizando se le llama Integral Impropia, debido a que el
integrando es discontinuo en el intervalo propuesto.
Considere el caso de:


1
0
1 x
dx
Argumente y comparta con sus compaeros
DEFINICIN:
Sea f(x) una funcin continua en el intervalo semiabierto [a, b), entonces:



Si el lmite existe y es finito, decimos que la integral impropia es convergente,
donde el lmite es el valor de la integral. Si el lmite no existe, decimos que la
integral impropia es divergente.
Ejemplo 1:
Integral la funcin
3
1
) (
x
x f = en el intervalo (0, 8].
Solucin:
Como la funcin es discontinua en x = 0, entonces planteamos una solucin
aplicando la definicin dada anteriormente.

=
b
a
t
a
b t
dx x f Lim dx x f ) ( ) (

UNIVERSIDADNACIONALABIERTAYADISTANCIA UNAD
ESCUELADECIENCIASBSICAS,TECNOLOGAEINGENIERA
CONTENIDODIDCTICODELCURSO:100411ClculoIntegral

( ) ( )

= =

+ + +


3
2
3
2
3
2
3
1
2
3
8
2
3
2
3 1
0
8
0
8
0
8
0
3
t Lim x Lim dx x Lim dx
x
t
t
t
t
t

Evaluando obtenemos:

( ) 6 4 *
4
3
0
2
3
64
2
3 1
3
2
3
8
0
3
= = =

dx
x
Por consiguiente:
6
1
8
0
3
=

dx
x


Ejemplo 2:
Determinar la convergencia o no convergencia de la siguiente expresin:


1
0
1 x
dx

Solucin:
Como la funcin NO est definida para x = 1, debemos tomar el lmite unilateral,
luego el intervalo a tomar ser [0, 1), entonces:
( ) [ ]

=



t
t
t t
x Lim
x
dx
Lim
x
dx
0
0
1 1
1
0
1 2
1 1

Evaluando:
( ) 2 1 0 2 1 2
1
0
1
1
0
= = =

t
t
x Lim
x
dx

La integral propuesta es convergente y converge a 2.

UNIVERSIDADNACIONALABIERTAYADISTANCIA UNAD
ESCUELADECIENCIASBSICAS,TECNOLOGAEINGENIERA
CONTENIDODIDCTICODELCURSO:100411ClculoIntegral

Ejemplo 3:
Demostrar que

1
0
1
dx
x
k
es convergente si k < 1.
Solucin:
1
1 1
0 0
1
0
1
1
t
k
t
t
k
t
k
k
x
Lim dx x Lim dx
x

+
= =
+

+ +
Evaluando:

k k
t
k k
x
Lim dx
x
k k
t
k
t
k

+
=

+
=
+ + +

+
1
1
1 1
1
1
1
1 1
1
1
0
1
0
Para k < 1
Qu pasar si k 1? Hacer el anlisis con los compaeros del pequeo grupo
colaborativo.

DEFINICIN:
Sea f(x) una funcin continua en el intervalo semiabierto (a, b], entonces:
( ) ( )
+

=
b
t
a t
b
a
dx x f Lim dx x f

Al igual que en el caso anterior, si el lmite existe la integral converge y si el lmite
no existe, la integral diverge.
Con las definiciones dadas, podemos resolver integrales impropias con integrado
discontinuo.
Con el fin de fortalecer el tema, estimado estudiante demostrar que:
a-)


1
0
2
1 x
dx
Converge a
3
2
3

UNIVERSIDADNACIONALABIERTAYADISTANCIA UNAD
ESCUELADECIENCIASBSICAS,TECNOLOGAEINGENIERA
CONTENIDODIDCTICODELCURSO:100411ClculoIntegral

b-)


4
0
3
3 2 x
dx
Converge a ( )
3 3
101 4
2
1

c-)

2
0
) 2 tan(

dx x
Diverge.
Estos ejercicios deben desarrollarlos en el pequeo grupo colaborativo y
socializarlo con el tutor.


Leccin 17: Integrales impropias con lmites de integracin finitos

En el campo de las integrales impropias, tambin podemos encontrar unas
integrales impropias donde uno de los lmites es infinito, tal es el caso de:

0
2
dx e
x
muy utilizada en Probabilidad, pero tambin hay casos en Economa,
Administracin y otros.
La resolucin de este tipo de integrales, utiliza tambin lmites para eliminar una
posible indeterminacin.
DEFINICIN:
Sea f(x) una funcin continua en el intervalo semiabierto [a, b) o (-, a], entonces:

( ) ( )

=
R
a
R
a
dx x f Lim dx x f
o
( ) ( )



=
a
R
R
a
dx x f Lim dx x f

Si los lmites existen, entonces las integrales impropias son convergentes. Pero si
el lmite no existe, entonces la integral impropia diverge.

UNIVERSIDADNACIONALABIERTAYADISTANCIA UNAD
ESCUELADECIENCIASBSICAS,TECNOLOGAEINGENIERA
CONTENIDODIDCTICODELCURSO:100411ClculoIntegral

Ejemplo 1:
Determinar la convergencia o divergencia de:

1
2
1
dx
x
.
Solucin:
Observamos que el lmite superior es infinito, entonces aplicando la definicin
tenemos:

= =
x
R
R
R
R
Lim dx x Lim dx
x
1
1
1
2
1
2
1
Evaluando el lmite, tenemos:
( ) 1 1 0 1
1
1
1
2
= + = =

R Lim dx
x
R


La integral propuesta converge a 1.

Ejemplo 2:
Demostrar que:


1
x
dx
es divergente.


Fig. No. 6 Convergencia.



UNIVERSIDADNACIONALABIERTAYADISTANCIA UNAD
ESCUELADECIENCIASBSICAS,TECNOLOGAEINGENIERA
CONTENIDODIDCTICODELCURSO:100411ClculoIntegral

Solucin: Siguiendo el mismo procedimiento anterior:




=
1 1
R
R
x
dx
Lim
x
dx

[ ]
1
1

R
R
x Ln Lim
x
dx

Si evaluamos el lmite:
[ ] = =


R Ln Ln
x
dx
1
1

Como el lmite no existe, la integral diverge.
En estudios matemticos sobre fenmenos luminosos, electricidad, sonido y en
general en fenmenos ondulatorios, se puede encontrar integrales impropias,
donde los dos lmites de integracin son infinitos. Para resolver ente tipo de
integrales, hacemos uso de la siguiente definicin:

DEFINICIN:
Sea f(x) una funcin continua en el intervalo (-, ), si


a
dx x f ) ( y

a
dx x f ) ( son
convergentes, decimos que


dx x f ) ( es convergente y su valor se puede hallar
por la siguiente relacin:

( ) ( ) ( )



+ =
a
a
dx x f dx x f dx x f

Si alguna de las integrales diverge o las dos, entonces la integral total diverge.

UNIVERSIDADNACIONALABIERTAYADISTANCIA UNAD
ESCUELADECIENCIASBSICAS,TECNOLOGAEINGENIERA
CONTENIDODIDCTICODELCURSO:100411ClculoIntegral

Ejemplo 1:
Dada la integral:

+
dx
x
2
1
1
Determinar si converge o diverge.
Solucin:
Inicialmente definamos a = 0 y as aplicando la definicin:





+
+

+
=

+
a
a
dx
x
dx
x
dx
x
2 2 2
1
1
1
1
1
1


En seguida aplicamos el lmite:

+
+

+

a
b
c
a
c b
dx
x
Lim dx
x
Lim
2 2
1
1
1
1
Integrando:
[ ] [ ]
c
a
c
a
b
b
x Tan Lim x Tan Lim ) ( ) (
1 1


+

Evaluando el lmite:

[ ] [ ] ) ( ) ( ) ( ) (
1 1 1 1
a Tan c Tan Lim b Tan a Tan Lim
c b




+

Como a = 0, entonces:
[ ] [ ]

= + 0 ) ( 0
2 2 La integral converge.

Ejemplo 2:
Demostrar que:

dx e
x
Diverge.

UNIVERSIDADNACIONALABIERTAYADISTANCIA UNAD
ESCUELADECIENCIASBSICAS,TECNOLOGAEINGENIERA
CONTENIDODIDCTICODELCURSO:100411ClculoIntegral

Solucin:
Aplicando la definicin dada para estos casos tenemos:

+ =
0
0
dx e dx e dx e
x x x
Llamemos a la primea integral A y a la
segunda B
Desarrollemos la primera integral:
A=
( ) [ ] ) (
0
0 0
R
R
o
R
x
R
R
x
R
x
e e Lim e Lim dx e Lim dx e = = =


Evaluando:
A =
1 0 1
1
1
0
0
= + = + = + =


e
e e dx e
x
Converge
Ahora desarrollemos la segunda integral:
B =
( ) [ ] ) (
0
0 0
e e Lim e Lim dx e Lim dx e
R
R
R
o
x
R
R
x
R
x
= = =


Evaluando:

B =
= + = + =

1
0
0
e e dx e
x
Diverge
Vemos que la primera integral converge, pero la segunda diverge, por
consiguiente la integral original diverge.

UNIVERSIDADNACIONALABIERTAYADISTANCIA UNAD
ESCUELADECIENCIASBSICAS,TECNOLOGAEINGENIERA
CONTENIDODIDCTICODELCURSO:100411ClculoIntegral

EJERCICIOS LECCION No. 2:
Determinar SI la integral converge o diverge, en caso de que converja, hallar el
valor correspondiente:
1.
dx
x

1
0
3
1

2.

1
1
1
dx
x

3.

1
0
) ( dx x Ln

4.

0
) tan( dx x

5.

1
4
dx e
x

6.

+
10
2
1
dx
x
x

7.


1
0
2
1
2
dx
X

8.

2
0
) cot(

dx x

9.


5
0
5
4
dx x
10.


5
4
1
dx
x

UNIVERSIDADNACIONALABIERTAYADISTANCIA UNAD
ESCUELADECIENCIASBSICAS,TECNOLOGAEINGENIERA
CONTENIDODIDCTICODELCURSO:100411ClculoIntegral

Leccin 18: Integrales inmediatas.

Inicialmente vamos a hacer un recuento de las integrales que se pueden resolver
utilizando el concepto de antiderivada.
Recopilando lo estudiado en integrales indefinidas, las propiedades analizadas,
podemos exponer a continuacin las integrales obtenidas por definicin de la
antiderivada o primitiva.
1.

+ = c x dx
2.

+ = c kx kdx para k = constante


3.
c
n
x
dx x
n
n
+
+
=
+

1
1
para n -1 4.

+ = c x Ln dx
x
) (
1

5.
c
Lna
a
dx a
x
x
+ =

para a > 0 y a 1 6.
c e
n
dx e
nx nx
+ =

1
n 0
7.
c nx
n
dx nx sen + =

) cos(
1
) (
n 0 8.

+ = c nx sen
n
dx nx ) (
1
) cos(
n 0
9.

+ = c x dx x ) tan( ) ( sec
2
10.

+ = c x dx x ) cot( ) ( csc
2

11.

+ = c x dx x x ) sec( ) tan( ) sec(


12.

+ = c x dx x x ) csc( ) cot( ) csc(



13. c
a
x
Sen
x a
dx
+

1
2 2
a 0 14. c
a
x
Tan
a x a
dx
+

=
+

1
2 2
1
a 0
15. c
a
x
Sec
a
a x x
dx
+

1
2 2
1
16. c
x
a
Cos
a
a x x
dx
+

1
2 2
1

17.

+ = c x dx x senh ) cosh( ) ( 18.

+ = c x senh dx x ) ( ) cosh(

La idea no es memorizar estas frmulas, solo que con un buen anlisis se pueden
utilizar en muchas situaciones.

UNIVERSIDADNACIONALABIERTAYADISTANCIA UNAD
ESCUELADECIENCIASBSICAS,TECNOLOGAEINGENIERA
CONTENIDODIDCTICODELCURSO:100411ClculoIntegral

Ejemplo No 1:
Resolver:

dx
x
4

Solucin:
Como podemos ver se trata de una funcin exponencial, luego con la frmula
numero 5 se puede resolver esta integral.
c
Ln
dx
x
x
+ =

4
4
4

Ejemplo No 2:
Resolver:

dx 5

Solucin:
En este caso se trata de una constante, luego con la frmula numero 2 se puede
resolver esta integral
c x dx + =

5 5

Ejemplo No 3:
Hallar la siguiente integral:

dx e
x 4
30
Solucin:
Tenemos la integral de una constante por una funcin, por las propiedades
estudiadas, podemos sacar la constante de la integral y luego operar la funcin,
veamos:

= dx e dx e
x x 4 4
30 30
Por la frmula 6, desarrollamos la integral.
c e c e dx e
x x x
+ = + =

4 4 4
2
15
)
4
1
( 30 30
Por consiguiente:

UNIVERSIDADNACIONALABIERTAYADISTANCIA UNAD
ESCUELADECIENCIASBSICAS,TECNOLOGAEINGENIERA
CONTENIDODIDCTICODELCURSO:100411ClculoIntegral

c e dx e
x x
+ =

4 4
2
15
30

Ejemplo No 4:
Resolver:

dx x) 5 cos( 20

Solucin:
Se trata de la integral de una constante por una funcin trigonomtrica, la solucin
es de la siguiente manera:
c x sen dx x dx x +

= =

) 5 (
5
1
20 ) 5 cos( 20 ) 5 cos( 20

c c x sen dx x + + =

) 5 ( 4 ) 5 cos( 20


Ejemplo No 5:
Resolver la siguiente integral:
( )

dx x
2
2
4

Solucin:
Vemos que e integrado es un producto notable, es conveniente resolverlo primero
paras poder luego hacer la integracin.
( )

+ = + = dx dx x dx x dx x x dx x 16 8 ) 16 8 ( 4
2 4 2 4
2
2

Se aplico la linealidad para las integrales, ahora resolvemos cada integral.
3 2
3
1
5 2 4
16
3
8
5
1
16 8 c x c x c x dx dx x dx x + + + + =


Sumando las constantes en una sola, obtenemos:

UNIVERSIDADNACIONALABIERTAYADISTANCIA UNAD
ESCUELADECIENCIASBSICAS,TECNOLOGAEINGENIERA
CONTENIDODIDCTICODELCURSO:100411ClculoIntegral

c x x x c x c x c x + + = + + + + 16
3
8
5
1
16
3
8
5
1
3 5
3 2
3
1
5
Por consiguiente:
( ) c x x x dx x + + =

16
3
8
5
1
4
3 5
2
2


Leccin 19: Integrales inmediatas con sustitucin

En algunos casos la funcin NO tiene la forma directa para resolverla como
integral inmediata, pero haciendo una pequea transformacin, se puede llevar la
funcin dada a una forma tal que se pueda aplicar alguna de las funciones
inmediatas para resolverla.
Ejemplo No 6:
Resolver la integral: dx
x x


2
2 3
1

Solucin:
Si observamos detalladamente, esta funcin no tiene una forma conocida de las
integrales inmediatas, sin embargo por la forma de la funcin se puede inferir que
podemos llevarla a la forma dx
x a


2 2
1
Para esto debemos transformar el
trinomio
a la forma a
2
x
2
, entonces: 1 ) 1 2 ( 3 2 3
2 2
+ + + = x x x x organizando:
2
) 1 ( 4 + x ahora incluymoslo en la integral:
( )
dx
x
dx
x
dx
x x

+
=
+
=

2 2 2 2
) 1 ( 2
1
1 4
1
2 3
1
Ya lo tenemos de la forma
de una integral inmediata, observemos la frmula 13, luego:

UNIVERSIDADNACIONALABIERTAYADISTANCIA UNAD
ESCUELADECIENCIASBSICAS,TECNOLOGAEINGENIERA
CONTENIDODIDCTICODELCURSO:100411ClculoIntegral

c
x
Sen dx
x
+

+
=
+

2
1
) 1 ( 2
1
1
2 2
Por consiguiente:
c
x
Sen dx
x x
+

+
=

2
1
2 3
1
1
2


Ejemplo No 7:
Resolver la integral:
dx
x
x sen

+
2
0
2
) ( cos 16
) (


Solucin:
La forma de la funcin no es conocida, pero se puede transformar a la forma

=
+

a
x
Tan dx
x a
1
2 2
1
segn la forma propuesta. Entonces:
Sea U = cos(x) luego: dU/dx = -sen(x) entonces: dU = -sen(x)dx. Apliquemos la
integral sin lmites, para facilitar el proceso, al final se evalan los lmites.

=
+

=
+

4 4
1
16 ) ( cos 16
) (
1
2 2
U
Tan
U
dU
dx
x
x sen
Pero U = cos(x),
Entonces reemplazamos:


4
) cos(
4
1
4 4
1
1 1
x
Tan
U
Tan Evaluando en los lmites propuestos:


4
) 0 cos(
4
) 2 / cos(
4
1
4
) cos(
4
1
4 4
1
1 1
0
1 1
2
Tan Tan
x
Tan
U
Tan


Resolviendo y simplificando:
[ ]

= =


4
1
4
1
) 4 / 1 ( ) 0 (
4
1
4
) 0 cos(
4
) 2 / cos(
4
1
1 1 1 1 1
Tan Tan Tan Tan Tan

UNIVERSIDADNACIONALABIERTAYADISTANCIA UNAD
ESCUELADECIENCIASBSICAS,TECNOLOGAEINGENIERA
CONTENIDODIDCTICODELCURSO:100411ClculoIntegral

Luego:

=
+

4
1
4
1
) ( cos 16
) (
1
0
2
2
Tan dx
x
x sen



Ejemplo No 8:
Hallar la solucin de la integral propuesta:
dx
x
x x

+
+
1
2 3
2

Solucin:
La funcin del integrado No tiene forma conocida, pero es un polinomio que
podemos dividir para reducirlo lo ms que se pueda.
1
1
1 3
1
2 3
2
+
+ =
+
+
x
x
x
x x
Aplaqumosle la integral.
dx
x
x dx
x
x x

+
+ =
+
+
1
1
1 3
1
2 3
2
Aplicando la linealidad obtenemos:

+ + + =
+
+ c x Ln x x dx
x
dx xdx ) 1 (
2
3
1
1
3
2

Un ejemplo ms para adquirir destreza en este tipo de situaciones:
Ejemplo No 9:
Resolver la integral dada a continuacin:
dx
x
x sen

) ( cos
) (
2

Solucin:
Separemos el cos
2
(x) en cos(x)*cos(x) y reorganizando:
dx
x x
x sen
dx
x x
x sen
dx
x
x sen

= =
) cos(
1
*
) cos(
) (
) cos( * ) cos(
) (
) ( cos
) (
2

UNIVERSIDADNACIONALABIERTAYADISTANCIA UNAD
ESCUELADECIENCIASBSICAS,TECNOLOGAEINGENIERA
CONTENIDODIDCTICODELCURSO:100411ClculoIntegral

Por identidades trigonomtricas:

= dx x x dx
x x
x sen
) sec( * ) tan(
) cos(
1
*
) cos(
) (
Esta ltima integral tiene la forma de
la frmula 11 de las integrales inmediatas, entonces:

+ = c x dx x x ) sec( ) sec( * ) tan(


Por consiguiente:
c x dx
x
x sen
+ =

) sec(
) ( cos
) (
2


EJERCICIOS:
1.

+ 4
2
x
dx

2.
dx
x sen
x

+ ) ( 1
) cos(
2

3.
( ) dx x x

+
2
2
2

4. dx
x


2
4
1

5.
dx
x x

+ + 5 2
4
2

6.

dx x senh ) 5 (




UNIVERSIDADNACIONALABIERTAYADISTANCIA UNAD
ESCUELADECIENCIASBSICAS,TECNOLOGAEINGENIERA
CONTENIDODIDCTICODELCURSO:100411ClculoIntegral

Leccin 20: Integracin por cambio de variable.


( ) [ ] ( ) ( ) ( )

+ = = c U P dU U f dx x g x g f


La tcnica de sustitucin por cambio de variable, se utiliza cuando la funcin que
se desea integrar NO se le puede aplicar las frmulas de las integrales inmediatas,
pero haciendo un Truco Matemtico llamado cambio de variable, es posible la
resolucin de muchas integrales.
Pero la pregunta es Qu funciones se pueden integrar por cambio de variable?
Cuando la funcin que conforma el integrando es tal que una parte es la derivada
de la otra parte y las dos estn en forma de producto, se puede aplicar esta
tcnica. Las condiciones bsicas para establecer que se puede aplicar una
sustitucin es una buena observacin de la funcin a integrar y algo de
perspicacia matemtica.
Como el mtodo tiene que ver con el producto de una funcin y su derivada,
estara implcita la regla de la cadena, el siguiente teorema sustenta dicha tcnica:
TEOREMA:

Sea g(x) una funcin derivable y supongamos que P(x) es una antiderivada de la
funcin f(x). Si adems U = g(x), entonces:

( ) [ ] ( ) ( ) ( )

+ = = c U P dU U f dx x g x g f

Por consiguiente:

( ) ( ) [ ] C x g P C U P + = +

Demostracin:
Podemos demostrar que la derivada de P(g(x)) + C es la funcin que conforma el
integrado, veamos:

UNIVERSIDADNACIONALABIERTAYADISTANCIA UNAD
ESCUELADECIENCIASBSICAS,TECNOLOGAEINGENIERA
CONTENIDODIDCTICODELCURSO:100411ClculoIntegral

( ) ) ( ' * )) ( ( ' )) ( ( x g x g P C x g P
dx
d
= +
Pero por hiptesis P(x) es antiderivada

De f(x), luego:
( ) ( ) ) ( ' ) ( ) ( ' * ) ( ' x g x g f x g x g P =

As queda demostrado el teorema.
Los pasos para aplicar la tcnica de sustitucin son:
1. elegir una variable digamos u, v, w, que sustituya parte del integrado.
2. Hallar el diferencial de la variable seleccionada: du, dv, dw,
3. Reemplazar todos los trminos en el integrado de tal forma que queden
expresados solo en funcin de la nueva variable
4. Resolver la integral bajo la nueva variable. A veces no se puede hacer
esto, lo cual indica que dicha sustitucin no es la adecuada y se debe
intentar con otra forma de sustituir.
5. Una vez realizada la integracin, la nueva variable se reemplaza por la
variable original y as obtenemos la integral deseada.

Ejemplo No 1:
Desarrollar:
( ) dx x x

+
3
62
4
4 10

Solucin:
Vemos que la funcin es un producto de dos funciones: ( )
62
4
10 + x y
3
4x lo que
nos sugiere una sustitucin. Definimos la nueva variable 10
4
+ = x U , ahora
derivemos esta funcin: dx x dU x
dx
dU
3 3
4 4 = = Reemplazando es la integral

UNIVERSIDADNACIONALABIERTAYADISTANCIA UNAD
ESCUELADECIENCIASBSICAS,TECNOLOGAEINGENIERA
CONTENIDODIDCTICODELCURSO:100411ClculoIntegral

Original:
( )

= + dU U dx x x
62 3
62
4
4 10
Esta si se puede resolver:
c
U
dU U + =

63
63
62
. Pero la funcin original es x y no U, por lo cual se hace el
reemplazo de nuevo: 10
4
+ = x U entonces:
( )
( )
c
x
dx x x +
+
= +

63
10
4 10
63
4
3
62
4


Ejemplo No 2:
Hallar:

dx x sen x ) ( 3
3 2

Solucin:
Elegimos la nueva variable V = x
3
, ahora derivamos dx x dV x
dx
dV
2 2
3 3 = = Si
reemplazamos:
c V dV V sen dx x sen x + = =

) cos( ) ( ) ( 3
3 2
Reemplazando el
valor de V en funcin de x, tenemos:
( ) c x c V + = +
3
cos ) cos(
por consiguiente:
c x dx x sen x + =

) cos( ) ( 3
3 3 2


Ejemplo No 3:
Desarrollar:
dx
x
x sen

) (

UNIVERSIDADNACIONALABIERTAYADISTANCIA UNAD
ESCUELADECIENCIASBSICAS,TECNOLOGAEINGENIERA
CONTENIDODIDCTICODELCURSO:100411ClculoIntegral

Solucin:
Siguiendo los pasos descritos.
x
dx
du
x
dx
du x
dx
du
x u = = = =

2
2
2
1
2
1

Reemplazando en la integral.
c u du u sen du u sen dx
x
x sen
+ = = =

) cos( 2 ) ( 2 2 ) (
) (

Como x u = reemplazamos en la integral obtenida, nos resulta.

c x dx
x
x sen
+ =

) cos( 2
) (


Ejemplo No 4:
Hallemos la integral de tan(x).
Solucin:

= = dx x sen
x
dx
x
x sen
dx x ) ( *
) cos(
1
) cos(
) (
) tan(
Hacemos cambio de
variable: u = cos(x), luego:
dx x sen du x sen
dx
du
) ( ) ( = =
reemplazando:
c u Ln du
u
dx
x
x sen
+ = =

1
) cos(
) (
Pero u = cos(x) entonces:
c x Ln dx x + =

) cos( ) tan(

UNIVERSIDADNACIONALABIERTAYADISTANCIA UNAD
ESCUELADECIENCIASBSICAS,TECNOLOGAEINGENIERA
CONTENIDODIDCTICODELCURSO:100411ClculoIntegral

Ejemplo No 5:
Resolver:
dx
e
e
x
x


6
3
4

Solucin:
Observando detenidamente esta integral, vemos que tiene la forma de Sen
-1
(x),
pero primero debemos ajustarlo par poder aplicar este tipo de integral.
( )


=

dx
e
e
dx
e
e
x
x
x
x
2
3 2
3
6
3
2
4
Ahora si podemos hacer cambio de
variable.
x
e w
3
= Luego: dx e dw
x 3
3 = y
x
e
dw
3
3
=
reemplazando en la
integral:

+


c
w
Sen
w
dw
w
dw
dx
e
e
x
x
2 3
1
2
3
1
2
3
4
1
2 2 2 2 6
3

Finalmente:
c
e
Sen dx
e
e
x
x
x
+

2 3
1
4
3
1
6
3





UNIVERSIDADNACIONALABIERTAYADISTANCIA UNAD
ESCUELADECIENCIASBSICAS,TECNOLOGAEINGENIERA
CONTENIDODIDCTICODELCURSO:100411ClculoIntegral

CAPITULO 5: Mtodos de integracin II

Introduccin
En este captulo vamos a detallar los mtodos de integracin por racionalizacin,
los tres casos de integracin por sustitucin trigonomtrica y la integracin por
partes temas claves en el desarrollo del curso.
Para este captulo tenemos que las matemticas se vuelven una ciencia un poco
terica, debido a que parte de teoras y definiciones cuyas demostraciones se
soportan en el principio de la lgica, los axiomas y postulados, que permiten el
desarrollo de habilidades de pensamiento de orden superior, especialmente la
Deduccin, Induccin y la Abstraccin, pero a su vez presenta dificultades para
poder desplegar dichas habilidades, ya que se requiere trabajar el sentido de
anlisis, desarrollo del raciocinio, aspectos no fciles de activar en la mente
humana.
El manejo complejo del trabajo mental para el estudio de las Matemticas,
requiere un esfuerzo sistemtico en el anlisis de contenidos, esto indica que para
comprender un tema, se debe comprender uno previo que facilite la comprensin
del siguiente y de esta forma ir avanzando en el desarrollo de los temas, con la
realizacin de muchos ejercicios, como se ha sugerido en todas las lecciones, esto
quiere decir que trabajando es como aprendemos, entonces ANIMO y a realizar
todas las integrales que se nos presenten.

Leccin 21: Integracin por racionalizacin.
Cuando el integrado presenta radicales, se puede presentar problemas para
resolver la integral, la racionalizacin puede ser un camino para superar dicho
problema, veamos algunos casos.
Ejemplo No 1:
Resolver:


dx
x x
1

Solucin:
Haciendo un cambio de variable: dx udu x u = = 2
2
luego reemplazamos:

UNIVERSIDADNACIONALABIERTAYADISTANCIA UNAD
ESCUELADECIENCIASBSICAS,TECNOLOGAEINGENIERA
CONTENIDODIDCTICODELCURSO:100411ClculoIntegral

c u Lu du
u
du
u u
u
u u
udu
dx
x x
+ =


1 2
1
1
2
) 1 (
2
2 1
2

Reemplazando x u = tenemos finalmente:
c x Ln dx
x x
+ =

1 2
1


Ejemplo No 2:
Hallar la integral de:
dx x x
3
+

Solucin:
Haciendo
= + = + =
3 3 3
V x x V x V
derivamos: dx dV V =
2
3 ,

reemplazamos en la integral original:
( )

= = + dV V V dV V V V dx x x ) 3 3 ( ) 3 )( (
3 6 2 3 3


Desarrollando:
c V V +
4 7
4
3
7
3

Si volvemos a reemplazar, obtenemos


finalmente:
( ) ( ) c x x dx x x + + + = +
3
4
3
7
4
3
7
3
3


Esperamos que estos ejemplos modelos permitan desarrollar destreza para
resolver este tipo de integrales.


UNIVERSIDADNACIONALABIERTAYADISTANCIA UNAD
ESCUELADECIENCIASBSICAS,TECNOLOGAEINGENIERA
CONTENIDODIDCTICODELCURSO:100411ClculoIntegral

EJERCICIOS:
En los siguientes ejercicios desarrollar la integral, indicando paso por paso.
1.
dx x
x

2
3 *
Rta:
c
Ln
x
+
2
3
3 2
1

2.

+
+
dx
x
x x
1
2 3
2
Rta: c x Ln x x + + + 1
2
3
2

3.
dx
x sen
x x sen


) (
) cos( ) (
Rta:
c x sen Ln x + ) (

4.
( )
dx
x
x

+
4
3
3
Rta:
( ) ( )
c
x x
x
x Ln +
+
+
+

+
+ +
3 2
3
9
3 2
27
3
9
3
5.

+
1
0
4
3
dx
x
Rta:

5
4
24 6 Ln
6.
dx
x
x

+
4
2
2
Rta:

+
3
2
2 2 Ln
7.
dx x x

+ 2
3 3
Rta:
( ) c x + +
3
3
2
9
2

8.
dx x sen x

+1 ) ( ) cos(
Rta:
( ) c x sen + +
3
1 ) (
3
2






UNIVERSIDADNACIONALABIERTAYADISTANCIA UNAD
ESCUELADECIENCIASBSICAS,TECNOLOGAEINGENIERA
CONTENIDODIDCTICODELCURSO:100411ClculoIntegral

Leccin 22: Integrales por sustitucin trigonomtrica caso I

a
x a x
a
x
sen a dx x a
2 2
1 2 2 2
5 . 0

+


La sustitucin trigonomtrica, es una tcnica que se puede utilizar cuando en el
integrando se presentan expresiones como:
2 2
x a ,
2 2
x a + ,
2 2
a x ;
siendo
a > 0, analicemos los tres casos:

PRIMER CASO:
:
2 2
x a : La sustitucin es de la forma
) ( asen x =
para
2 2

. La
restriccin se debe a que en este intervalo, la funcin mantiene sus condiciones
para
serlo como tal. Haciendo el reemplazo ) (
2 2 2 2 2
sen a a x a = , organizando:
( ) ) ( cos ) ( 1 ) (
2 2 2 2 2 2 2
a sen a sen a a = = . Como
2 2
x a est dentro de una raz,
entonces nos resulta ) cos( a .






Fig. No. 7 Sustitucin trigonomtrica caso 1

UNIVERSIDADNACIONALABIERTAYADISTANCIA UNAD
ESCUELADECIENCIASBSICAS,TECNOLOGAEINGENIERA
CONTENIDODIDCTICODELCURSO:100411ClculoIntegral

Pero la expresin final debe expresarse en funcin de x
y no de , lo que se resuelve usando el siguiente grfico:
Desarrollemos:

dx x a
2 2
Siendo ) ( asen x = entonces:
d a dx ) cos( = Haciendo el reemplazo:
d a sen a a ) cos( ) (
2 2 2

esto es equivalente a:



= d a a d a sen a ) cos( ) ( cos ) cos( ) ( 1 (
2 2 2 2


= d a d a ) ( cos ) ( cos
2 2 2 2
Por la identidad:
2
) 2 cos( 1
) ( cos
2
x
x
+
=
reemplazamos para poder integrar:
( )

+ =

+
=

d
a
d
a
d a d a ) 2 cos(
2 2 2
2 cos 1
) ( cos
2 2
2 2 2

La ltima parte si se puede integrar, luego:
c sen
a a
d a + + =

) 2 (
4 2
) ( cos
2 2
2 2


Pero la variable original est en funcin de X y no de , luego transformamos a
en X, la grfica anterior nos ayuda a hacer dicha transformacin.
Por la grfica:

= =

a
x
Sen
a
x
sen
1
) ( Por otro lado, Tambin por la grfica:
a
x a
2 2
) cos(

= Reemplazando:

UNIVERSIDADNACIONALABIERTAYADISTANCIA UNAD
ESCUELADECIENCIASBSICAS,TECNOLOGAEINGENIERA
CONTENIDODIDCTICODELCURSO:100411ClculoIntegral

a
x a
a
x a
a
x
Sen
a
sen
a a
dx x a
2 2 2
1
2 2 2
2 2
*
2 2
) 2 (
4 2

+

= + =



c x a
x
a
x
Sen
a
dx x a + +

2 2 1
2
2 2
2 2

Ejemplo 1:
Desarrollar:


dx
x
2
9
1

Solucin:
Hacemos:
d dx sen x cos 3 ) ( 3 = =
Reemplazando:

( )
( )

=

d d
sen
d dx
x ) ( cos 3
) cos( 3
) ( 1 9
) cos( 3
) cos( 3 9
) cos( 3
9
1
2 2 2 2

Simplificando:
c d d + = =

) cos(
) cos(
Pero debemos expresarlo como X,
lo que se hace por medio del cambio que se propuso inicialmente:
) ( 3 sen x =

Despejamos

=

a
x
Sen
a
x
sen
1
) (
Finalmente:
c
a
x
Sen dx
x
+

1
2
9
1



UNIVERSIDADNACIONALABIERTAYADISTANCIA UNAD
ESCUELADECIENCIASBSICAS,TECNOLOGAEINGENIERA
CONTENIDODIDCTICODELCURSO:100411ClculoIntegral

Leccin 23: Integrales por sustitucin trigonomtrica caso II
2 2
x a + La sustitucin es de la forma
) tan( a x =
para
2 2

El
procedimiento es similar al caso anterior, solo que la grfica cambia:






Fig. No. 8 sustitucin trigonomtrica caso 2

a
x
= ) tan(
Despejando el ngulo:

=

a
x
Tan
1



Ejemplo 2:
Resolver:
dx x

+
2
16

Solucin:
Hacemos el cambio de variable:
) tan( 4 = x
luego:
d dx ) ( sec 4
2
=
y
reemplazamos:
d dx x ) ( sec 4 * ) ( tan 4 16 16
2 2 2 2

+ = +
resolviendo:

UNIVERSIDADNACIONALABIERTAYADISTANCIA UNAD
ESCUELADECIENCIASBSICAS,TECNOLOGAEINGENIERA
CONTENIDODIDCTICODELCURSO:100411ClculoIntegral

( ) d d ) ( sec 4 * ) ( sec 4 ) ( sec 4 * ) ( tan 1 16
2 2 2 2

= +
Simplificando:

d ) ( sec 16
3

. Esta integral se puede resolver por la siguiente frmula.



( ) ( ) ( ) ( )

= du u
n
n
u u
n
du u
n n n 2 2
sec
1
2
tan sec
1
1
sec

Para n 1
Siguiendo con el ejemplo:
c d d +

+ =

) sec(
2
1
) tan( ) sec(
2
1
16 ) ( sec 16
3
Resolviendo:
c Ln d + + + =

+

) tan( ) sec( 8 ) tan( ) sec( 8 ) sec(
2
1
) tan( ) sec( 8
Debemos transformar el ngulo en la variable x.

c
x x
Ln
x x
Ln + +
+
+
+
= + +
4 4
16
8
4
*
4
16
8 ) tan( ) sec( 8 ) tan( ) sec( 8
2 2

Resumiendo: c
x x
Ln x x d +
+ +
+ + =

4
16
8 16
2
1
) ( sec 16
2
2 3

Finalmente:
c
x x
Ln x x dx x +
+ +
+ + = +

4
16
8 16
2
1
16
2
2 2

UNIVERSIDADNACIONALABIERTAYADISTANCIA UNAD
ESCUELADECIENCIASBSICAS,TECNOLOGAEINGENIERA
CONTENIDODIDCTICODELCURSO:100411ClculoIntegral


Leccin 24: Integrales por sustitucin trigonomtrica caso III
:
2 2
a x La sustitucin es de la forma: ) sec( a x = para
2 2

. Los
pasos para desarrollar integrales de este tipo son similares a los casos anteriores.






Fig. No. 9 Sustitucin trigonomtrica caso 3


a
x
= ) sec(

=

a
x
Sec
1



Ejemplo 3:
Solucionar la integral propuesta:
dx
x
x

4
2

Solucin:
La sustitucin: d tn dx x ) ( ) sec( 2 ) sec( 2 = = si reemplazamos:

d dx
x
x
) tan( ) sec( 2
) sec( 2
4 ) ( sec 4 4
2 2


=

UNIVERSIDADNACIONALABIERTAYADISTANCIA UNAD
ESCUELADECIENCIASBSICAS,TECNOLOGAEINGENIERA
CONTENIDODIDCTICODELCURSO:100411ClculoIntegral

Operando:
( )
( )

= = =

d d d d 1 ) ( sec 2 ) ( tan 2 ) tan( ) ( tan 2 ) tan( 2


2
1 sec 4
2 2 2
2

Si aplicamos la linealidad, tenemos:

+ = c d d 2 ) tan( 2 2 ) ( sec 2
2

Por la sustitucin hecha, reemplazamos el ngulo por x, luego:
Segn la grfica siguiente:




Fig. No. 10 aplicacin


a
x
= ) sec(
2
4
) tan(
2

=
x



Si reemplazamos:
c
x
Tan
x
dx
x
x
+

2
4
2
2
4
2
4
2
1
2 2

El propsito de esta tcnica es que cuando se presenten casos de integrales que
contengan las formas descritas anteriormente, se utilicen adecuada y
correctamente. Esto se adquiere con mucha observacin de la integral propuesta
y algo de perspicacia. Pero es pertinente que se desarrollen ejercicios sobre el
caso para adquirir destreza en la misma.

UNIVERSIDADNACIONALABIERTAYADISTANCIA UNAD
ESCUELADECIENCIASBSICAS,TECNOLOGAEINGENIERA
CONTENIDODIDCTICODELCURSO:100411ClculoIntegral


EJERCICIOS: Leccin No. 4
1. dx
x
x


2
4
Rta: c x
x
x
Ln + +

2
2
4
4 2
2
2.

+
dx
x
x
2
2
1
Rta:
c x Tan x +

) (
1

3.

dx
x
x
2
1
3 2
Rta: c x Sen x +

) ( 3 1 2
1 2

4.

dx
x
x
2
2
25
3
Rta: c
x
Sen x x +



25
25
2
19
25
2
1
1 2

5. dx
x
x

1
1
2
2
4
2
Rta: 3
6.
( )
dx
x x

+16
2
2
Rta: c
x
x
Ln +
+ 16
16
1
2










UNIVERSIDADNACIONALABIERTAYADISTANCIA UNAD
ESCUELADECIENCIASBSICAS,TECNOLOGAEINGENIERA
CONTENIDODIDCTICODELCURSO:100411ClculoIntegral

Leccin 25: Integracin por partes.


= vdu v u udv .

En el mundo matemtico, cientfico y otros, se presentan casos donde la integral
es un Producto de Funciones, casos donde se aplica la tcnica llamada
integracin por partes. En muchas ocasiones se ha manifestado que toda regla de
derivacin da origen a una regla de integracin. La integracin por partes est
relacionada con la regla de la cadena.
Sean f(x) y g(x) dos funciones diferenciables, entonces:
( ) ) ( * ) ( ) ( * ) ( ) ( * ) ( x f
dx
d
x g x g
dx
d
x f x g x f
dx
d
+ = Si integramos las dos ecuaciones:
( )

+ = ) ( * ) ( ) ( * ) ( ) ( * ) ( x f
dx
d
x g x g
dx
d
x f x g x f
dx
d
Tenemos:
[ ]

+ = dx x f x g dx x g x f x g x f ) ( ' * ) ( ) ( ' * ) ( ) ( * ) ( Reorganizando:

= dx x f x g x g x f dx x g x f ) ( ' ) ( ) ( * ) ( ) ( ' * ) (
Llamemos a u = f(x) y v = g(x),
Si reemplazamos en la ecuacin anterior:


= vdu v u udv .
Frmula para la regla de la cadena.
El xito de la tcnica est en la seleccin de las funcin u y v, tal que la integral
del segundo miembro de la ecuacin se pueda integrar fcilmente. La eleccin
debe ser tal que u se pueda derivar y v se pueda integrar. Esto se adquiere con
la prctica; es decir, haciendo diversos ejercicios, aqu vamos a relacionar algunos
modelos que darn las pautas para aplicar esta tcnica.
Ejemplo 1:
Desarrollar:

dx x xsen ) (

UNIVERSIDADNACIONALABIERTAYADISTANCIA UNAD
ESCUELADECIENCIASBSICAS,TECNOLOGAEINGENIERA
CONTENIDODIDCTICODELCURSO:100411ClculoIntegral

Solucin:
Vemos se presenta un producto, luego se sospecha una integracin por partes.
Hacemos el cambio de variable. x u = y dx x sen dv ) ( = , entonces debemos
derivar u e integrar v, veamos: dx du = y ) cos(x v =
Como ya tenemos todas las partes que necesitamos, reemplazamos en la
ecuacin:

= = = dx x x x dx x xsen vdu v u udv ) cos( )) cos( ( ) ( *

c x sen x x dx x x x + + =

) ( ) cos( ) cos( )) cos( (

Finalmente:
c x sen x x dx x xsen + + =

) ( ) cos( ) (

Ejemplo 2:
Resolver:

dx x Ln ) (
Solucin:
Una integral muy conocida, una vez desarrollada podemos asimilarla.
dx
x
du x Ln u
1
) ( = = Por otro lado: x v dx dv = = Si aplicamos la frmula:
c x x xLn dx
x
x x xLn vdu v u udv + = =

) (
1
) ( *

Entonces:
( ) c x Ln x dx x Ln + =

1 ) ( ) (

Ejemplo 3:
Desarrollar la integral:

dx xe
x

UNIVERSIDADNACIONALABIERTAYADISTANCIA UNAD
ESCUELADECIENCIASBSICAS,TECNOLOGAEINGENIERA
CONTENIDODIDCTICODELCURSO:100411ClculoIntegral

Solucin:
Sea
dx dx x u = =
y
x x
e v dx e dv = = luego:

= = dx e xe dx xe vdu v u udv
x x x
*
Resolviendo:
( ) c x e dx xe
x x
+ =

1


Ejemplo 4:
Resolver:

2
6
2
) ( csc

dx x x


Solucin:
Elegimos:
dx du x u = =
Por otro lado:
) cot( ) ( csc
2
x v dx x dv = =

Entonces:

= dx x x x dx x x ) cot( ) cot( ) ( csc
2
Resolviendo la segunda
integral:

+ = ) ( ) cot( ) ( csc
2
x sen Ln x x dx x x
Ahora evaluemos los lmites:
[ ]
2
6
2
6
2
) ( ) cot( ) ( csc

x sen Ln x x dx x x + =

Desarrollando:
2
6
3
) ( csc
2
6
2
Ln dx x x + =


NOTA: No podemos olvidar que elegir u debe ser tal que se pueda derivar y dv
tal que se pueda integrar, tengamos esto muy en cuenta.

UNIVERSIDADNACIONALABIERTAYADISTANCIA UNAD
ESCUELADECIENCIASBSICAS,TECNOLOGAEINGENIERA
CONTENIDODIDCTICODELCURSO:100411ClculoIntegral

Fenmeno de Recurrencia: Hay situaciones donde se debe aplicar la integracin
por partes varias veces. Teniendo en cuenta los principios del mtodo y lo
desarrollado sobre integracin, se puede resolver cualquier problema de este tipo.

Ejemplo 1:
Desarrollar

dx e x
x 2

Solucin:
Aplicando la resolucin por partes: xdx du x u 2
2
= = y
x x
e v dx e dv = =
luego:

= = dx xe e x dx xe e x dx e x
x x x x x
2 2
2 2 2

La ltima integral la resolvemos por el mismo mtodo:
dx du x u = = y
x x
e v dx e dv = = Reemplazando de nuevo:

+ = = c e xe dx e xe dx xe
x x x x x
Reagrupando:

( ) c e xe e x dx xe e x dx e x
x x x x x x
+ = =

2 2
2 2 2

( ) c x x e c e xe e x dx e x
x x x x x
+ + = + + =

2 2 2 2
2 2 2

El problema exigi aplicar el mtodo dos veces.
Ejemplo 2:
Resolver:

dx x e
x
) cos(

Solucin:
dx e du e u
x x
= =
y
) ( ) cos( x sen v dx x dv = =
reemplazando:

UNIVERSIDADNACIONALABIERTAYADISTANCIA UNAD
ESCUELADECIENCIASBSICAS,TECNOLOGAEINGENIERA
CONTENIDODIDCTICODELCURSO:100411ClculoIntegral


= dx e x sen x e dx x e
x x x
) ( ) cos( ) cos(

Debemos integrar esta ltima expresin:
dx e dx e u
x x
= = y
) cos( ) ( x v dx x sen dv = =
Reemplazando:

+ = = dx e x e x dx e x e x dx e x sen
x x x x x
) cos( ) cos( ) cos( ) cos( ) (

Como esta integral est precedida de un signo negativo, entonces la integral
quedara:

dx e x e x
x x
) cos( ) cos(
. Agrupando toda la integral, obtenemos:

+ = dx e x x e x sen e dx x e
x x x x
) cos( ) cos( ) ( ) cos(
La
ltima integral es similar a la primera, luego las podemos agrupar as:
) cos( ) ( ) cos( ) cos( x e x sen e dx e x dx x e
x x x x
+ = +


c x e x sen e dx x e
x x x
+ + =

) cos( ) ( ) cos( 2

Finalmente:
( ) c x x sen e dx x e
x x
+ + =

) cos( ) (
2
1
) cos(


EJERCICIOS:
Resolver las siguientes integrales usando el mtodo de integracin por partes,
justificar porque se utiliza este mtodo.

1.

dx x x ) cos(
Rta:
c x x xsen + + ) cos( ) (

UNIVERSIDADNACIONALABIERTAYADISTANCIA UNAD
ESCUELADECIENCIASBSICAS,TECNOLOGAEINGENIERA
CONTENIDODIDCTICODELCURSO:100411ClculoIntegral
2.

dx x Ln x ) (
2
Rta:
c x x Ln x +
3 3
9
1
) (
3
1

3.

dx x x ) ( sec
2
Rta:
c x Ln x x + + ) cos( ) tan(

4.
dy y y

+
2
0
1
Rta:
15
4 3 48

5.
dx
x
x Ln

) (
Rta:
c x x Ln x + 4 ) ( 2

6.
dx x x

+4
3 5
Rta:
( ) ( ) c x x x + + +
2
5
4
45
4
4
3
2
3
2
3
3 3

7.

2
0
2
) ( cos

dx x
Rta:
4

UNIVERSIDADNACIONALABIERTAYADISTANCIA UNAD
ESCUELADECIENCIASBSICAS,TECNOLOGAEINGENIERA
CONTENIDODIDCTICODELCURSO:100411ClculoIntegral


CAPITULO 6: Mtodos de integracin III
Introduccin
En este captulo vamos a ver los mtodos de integracin un poco ms
complicados debido a los temas a tratar como son las fracciones parciales, la
funcin exponencial, la funcin logartmica, la funcin trigonomtrica y la funcin
hiperblica, sin embargo los temas son tratados con mucho cuidado y con
ejercicios muy sencillos fciles de digerir por el lector. Sin embargo si se
presentan dificultades se puede acudir al tutor, se pueden realizar las graficas de
cada ejercicio con la ayuda de un graficador (software libre) de los cuales existen
pginas web indicadas en la cibergrafia.
La graficacion de los ejercicios facilita y visualiza las cantidades que se estn
pidiendo, esto y la realizacin de muchos ejercicios y ejemplos resueltos los
prepara con el fin de enfrentar este tipo de problemas.

Leccin 26: Integracin por fracciones parciales.

( )
( )
dx
x
b
x
a
dx
x Q
x P

=



En el curso de lgebra, Trigonometra y Geometra Analtica, se estudiaron los
principios sobre fracciones parciales, se dio el concepto y algunos ejemplos
ilustrativos, en este aparte se utiliza esta herramienta para desarrollar un tipo
particular de integrales. Profundizaremos un poco sobre las fracciones parciales y
luego las llevaremos al mundo de las integrales.
Por un teorema de lgebra avanzada se afirma que toda fraccin racional; es
decir, el cociente de dos polinomios, se puede descomponer en suma de
fracciones racionales ms simples. Para desarrollar el mtodo de fracciones
parciales, se debe tener en cuenta:

UNIVERSIDADNACIONALABIERTAYADISTANCIA UNAD
ESCUELADECIENCIASBSICAS,TECNOLOGAEINGENIERA
CONTENIDODIDCTICODELCURSO:100411ClculoIntegral

Para la fraccin
) (
) (
) (
x g
x f
x p = con g(x) 0 sea una fraccin racional propia; es
decir, f(x) debe tener menor grado que g(x) y por otro lado, que g(x) se pueda
descomponer en factores primos. Tericamente cualquier polinomio con
coeficientes reales se puede escribir como producto de factores lineales reales y /
o factores cuadrticos, es posible que obtenerlos no sea tarea fcil.

Veamos a continuacin algunos tipos de descomposicin en fracciones parciales.

Descomposicin En Factores Lineales Simples: Cuando g(x) se puede
descomponer fcilmente; digamos por factorizacin, para obtener factores
lineales simples de la forma (x
1
), (x
2
) . . .
Ejemplo 1:
Descomponer en fracciones parciales la expresin:
6
1 3
) (
2


=
x x
x
x p
Solucin:
El polinomio lo podemos expresar de la siguiente manera:
3 2 6
1 3
) (
2

+
+
=


=
x
B
x
A
x x
x
x p
El trabajo consiste en encontrar los valores de A y B. Veamos cmo se realizara.
) 3 )( 2 (
2 3
) 3 )( 2 (
) 2 ( ) 3 (
3 2 +
+ +
=
+
+ +
=

+
+ x x
B Bx A Ax
x x
x B x A
x
B
x
A

Como el denominador es equivalente, entonces se debe igualar los numeradores.
B Bx A Ax x 2 3 1 3 + + =
. Luego hacemos equivalencia entre trminos:
Para x: 3 = A + B
Para trminos independientes: -1 = -3A + 2B

UNIVERSIDADNACIONALABIERTAYADISTANCIA UNAD
ESCUELADECIENCIASBSICAS,TECNOLOGAEINGENIERA
CONTENIDODIDCTICODELCURSO:100411ClculoIntegral

Tenemos dos ecuaciones con dos incgnitas, que se pueden resolver por los
mtodos estudiados en el curso de lgebra, Trigonometra y Geometra Analtica.
Resolviendo tenemos que: A = 7/5 y B = 8/5, reemplazando en las fracciones
propuestas obtenemos:
) 3 ( 5
8
) 2 ( 5
7
6
1 3
) (
2

+
+
=


=
x x x x
x
x p

Ejemplo 2:
Descomponer en fracciones parciales
2 3
7 5
) (
2
+

=
x x
x
x D
Solucin:
Factorizando el denominador:
( )( ) 1 2
7 5
2 3
7 5
2


=
+

x x
x
x x
x

Expresamos la ltima fraccin como suma e fracciones parciales.

( )( ) ( )( ) ( )( ) 1 2
2
1 2
) 2 ( ) 1 (
1 2 1 2
7 5

+
=

+
=

=


x x
B Bx A Ax
x x
x B x A
x
B
x
A
x x
x

Haciendo equivalencia de numeradores:

Descomposicin En Factores Lineales Repetidos: En algunos casos cuando se
busca linealizar el denominador aparece un trmino lineal al cuadrado, entonces
se escribe la suma con dos trminos lineales, uno con grado 1 y el otro con grado
2.
( )
( )
( )
2
1
1
2
) (
) (
) (

+
+
+
=
+
+
= =
x
B
x
A
p x
b ax
x g
x f
x p

Veamos unos ejemplos:

UNIVERSIDADNACIONALABIERTAYADISTANCIA UNAD
ESCUELADECIENCIASBSICAS,TECNOLOGAEINGENIERA
CONTENIDODIDCTICODELCURSO:100411ClculoIntegral

Ejemplo 3:
Descomponer en fracciones parciales:
( )
2
1
2
) (
+

=
x
x
x p

Solucin:
( )
( )
( )
2 2
1
1
1
2
+
+
+
=
+

x
B
x
A
x
x

El desarrollo es similar al caso anterior, sumar fracciones e igualar trminos:
( )
( )
( )
( )
( )
2 2 2
1
1
1
1
1
2
+
+ +
=
+
+
+
=
+

x
B x A
x
B
x
A
x
x
Desarrollando el numerador.
( )
( ) ( )
2 2
1 1
1
+
+ +
=
+
+ +
x
B A Ax
x
B x A
Igualemos los numeradores:
B A Ax x + + = 2

Para la variable x: 1 = A
Para trmino independiente: -2 = A + B
Desarrollando, obtenemos que A = 1 y B = -3
Entonces:
( )
( )
( )
2 2
1
3
1
1
1
2
) (

=
+

=
x
x
x
x
x p


Descomposicin En Factores Cuadrticos: Cuando se presentan caso donde el
denominador presenta trminos cuadrticos que no se pueden reducir, se debe
proceder como se ilustra en los siguientes ejemplos.

UNIVERSIDADNACIONALABIERTAYADISTANCIA UNAD
ESCUELADECIENCIASBSICAS,TECNOLOGAEINGENIERA
CONTENIDODIDCTICODELCURSO:100411ClculoIntegral

Ejemplo 4:
Reducir a fracciones parciales:
( )( ) 1 1
3 4
2
+
+
x x
x

Solucin:
La expresin la podemos escribir como:
( )( ) 1 1 1 1
3 4
2 2
+
+
+

=
+
+
x
c Bx
x
A
x x
x

Observemos que como el denominador tiene trmino cuadrtico, el numerador
debe tener trmino lineal, entonces:
( ) ( )( )
( )( ) ( )( ) 1 1 1 1
1 1
1 1
2
2 2
2
2
2
+
+ + +
=
+
+ + +
=
+
+
+
x x
C Cx Bx Bx A Ax
x x
x c Bx x A
x
c Bx
x
A

Como en los casos anteriores, el denominador es similar, luego solo igualamos el
numerador:
Para la variable x
2
: 0 = A + B
Para la variable x: 4 = -B + C
Para el trmino independiente: 3 = a C
Tenemos tres ecuaciones con tres incgnitas, existen varios mtodos de
resolucin, aplique la que desees y debes llegar a :
A = 7/3
B = -7/3
C =
Reemplazamos en la fraccin original:
( )( ) ( ) 1
2
1
3
7
1 3
7
1 1 1 1
3 4
2 2 2
+
+
+

=
+
+
+

=
+
+
x
x
x x
c Bx
x
A
x x
x
Simplificando:
( )( ) ( ) ) 1 ( 6
14 3
1 3
7
1 1 1 1
3 4
2 2 2
+

=
+
+
+

=
+
+
x
x
x x
c Bx
x
A
x x
x

UNIVERSIDADNACIONALABIERTAYADISTANCIA UNAD
ESCUELADECIENCIASBSICAS,TECNOLOGAEINGENIERA
CONTENIDODIDCTICODELCURSO:100411ClculoIntegral

Como en el caso de los factores lineales, en los cuadrticos, tambin se pueden
encontrar factores repetidos, en este caso el procedimiento es similar a los casos
anteriores, veamos un ejemplo.
Ejemplo 5:
Expresar como fracciones lineales:
( )( )
2
2
2
2 3
12 15 6
+ +
+
x x
x x

Solucin:
El planteamiento es as:
( )( )
2 2 2 2
2
2
) 2 ( 2 3
2 3
12 15 6
+
+
+
+
+
+
+
=
+ +
+
x
E Dx
x
c Bx
x
A
x x
x x

Realice todo el procedimiento como se ha venido haciendo y debe obtener:
A = 1, B = -1, C = 3, D = 5, E = 0
Finalmente:
( )( )
2 2 2 2 2 2 2
2
2
) 2 (
5
2
3
3
1
) 2 ( 2 3
2 3
12 15 6
+
+
+
+
+
+
=
+
+
+
+
+
+
+
=
+ +
+
x
x
x
x
x x
E Dx
x
c Bx
x
A
x x
x x

Podemos resumir esta temtica, diciendo afirmando que las fracciones parciales,
es un mtodo algebraico que permite rescribir expresiones racionales, como
fracciones racionales sencillas, de tal forma que permite hacer integraciones para
este tipo de expresiones algebraicas.
INTEGRACIN POR FRACCIONES PARCIALES:
Sabiendo como se resuelven las fracciones parciales, ahora apliqumosla para
desarrollar integrales:

Ejemplo 1:
Desarrollar: dx
x x
x



6
1 3
2

Solucin:

UNIVERSIDADNACIONALABIERTAYADISTANCIA UNAD
ESCUELADECIENCIASBSICAS,TECNOLOGAEINGENIERA
CONTENIDODIDCTICODELCURSO:100411ClculoIntegral

Debemos tratar de linealizar el numerador, por medio de la factorizacin: Del
ejemplo 1 de fracciones lineales simples, vemos que esta fraccin se puede
escribir as:
( )( )

+
+
=
+

=


dx
x x
dx
x x
x
dx
x x
x
) 3 ( 5
8
) 2 ( 5
7
2 3
1 3
6
1 3
2

Aplicando la propiedad de la suma de integrales, podemos hacer:



+
+
=

+
+
=

+
+
dx
x
dx
x
dx
x
dx
x
dx
x x 3
1
5
8
2
1
5
7
) 3 ( 5
8
) 2 ( 5
7
) 3 ( 5
8
) 2 ( 5
7

Desarrollando:

+ + + =

+
+
c x Ln x Ln dx
x
dx
x
3
5
8
2
5
7
3
1
5
8
2
1
5
7

Ejemplo 2:
Desarrollar:
( )

2
1
2
x
x
para x -1
Solucin:
Debemos aplicar fracciones parciales a la fraccin dada, en el ejemplo 3 de la
descomposicin de fracciones parciales, obtuvimos que:
( )
( )
( )
2 2
1
3
1
1
1
2

=
+

x
x
x
x

Luego a partir de esto podemos aplicar la integral:
( ) ( )
dx
x
x
dx
x
x

=
+

2 2
1
3
1
1
1
2
Operando:
( ) ( )

dx
x
dx
x
dx
x
x
2 2
1
3
1
1
1
3
1
1

UNIVERSIDADNACIONALABIERTAYADISTANCIA UNAD
ESCUELADECIENCIASBSICAS,TECNOLOGAEINGENIERA
CONTENIDODIDCTICODELCURSO:100411ClculoIntegral

La primera integral es inmediata, la segunda se puede resolver por cambio de
variable, veamos:
( )
( )


=

dx x dx
x
2
2
1 3
1
3
Definimos u = x 1 luego du = dx, reemplazando:
c
u
du u +


1
2
Agrupando los trminos:
( )
c
x
x Ln
u
x Ln dx
x
dx
x
+

+ =


1
3
1
1
3 1
1
3
1
1
2

Finalmente:
c
x
x Ln dx
x
x
+

+ =
+

1
3
1
) 1 (
2
2

Como se puede ver en los ejemplos expuestos, la transformacin de la expresin
racional en fracciones ms simples, es con el fin de llevar la funcin a una forma
de integracin inmediata. Una vez integrada se hace el cambio de la variable de
sustitucin a la variable original.
Ejemplo 3:
Integrar la funcin:
3 2
3 4 2
2
2 3


x x
x x x

Solucin:
Como se trata de una fraccin impropia, primero se debe hacer la divisin, para
obtener una fraccin propia y as aplicar el mtodo analizado.
( )( ) 1 3
3 5
2
3 2
3 5
2
3 2
3 4 2
2 2
2 3
+

+ =


+ =


x x
x
x
x x
x
x
x x
x x x
Ahora:
( )( )

+

+ =


dx
x x
x
x
x x
x x x
1 3
3 5
2
3 2
3 4 2
2
2 3

UNIVERSIDADNACIONALABIERTAYADISTANCIA UNAD
ESCUELADECIENCIASBSICAS,TECNOLOGAEINGENIERA
CONTENIDODIDCTICODELCURSO:100411ClculoIntegral

Separando las integrales:
( )( ) ( )( )

+

+ =

+

+ dx
x x
x
xdx dx
x x
x
x
1 3
3 5
2
1 3
3 5
2

La primera integral es directa, la segunda debemos hacer fracciones parciales,
veamos:
( )( ) 1 3 1 3
3 5
+
+

=
+

x
B
x
A
x x
x

Desarrollando, tenemos que A = 3 y B = 2. Favor corroborar este resultado:
Entonces:
dx
x x
x dx
x x
x x x

+
+

+ =



1
2
3
3
2
3 2
3 4 2
2
2 3
Aplicando la linealidad:
c x Ln x Ln x dx
x
dx
x
xdx + + + + =
+
+

+

1 2 3 3
1
2
3
3
2
2
Reorganizando:
( ) ( ) [ ] c x x Ln x dx
x x
x x x
+ + =

2 3
2
2
2 3
1 3
3 2
3 4 2

EJERCICIOS:
En cada caso, resolver la integral aplicando las fracciones parciales.
1.
dx
x

1 4
3
Rta: c x Ln + 1 4
4
3

2.
dx
x
x

1
5
2
Rta: c x Ln x Ln + + 1 2 1 3
3.
dx
x x
x


+
6
1
2
Rta: c x Ln x Ln + + + 3
5
4
2
5
1

UNIVERSIDADNACIONALABIERTAYADISTANCIA UNAD
ESCUELADECIENCIASBSICAS,TECNOLOGAEINGENIERA
CONTENIDODIDCTICODELCURSO:100411ClculoIntegral

4.
( )


dx
x
2
1 8
3
Rta:
c
x
+

8 64
3

5.
dx
x x
x

+ +
+
4 2 8
9 5
2
Rta:
( ) c x Tan x x Ln +

+ + + +

1 8
31
31
248
31 67
2 4
16
5
1 2

6.
dx
x x
x

+

2 3
3 17
2
Rta: c x Ln x Ln + + + 1 4 2 3
3
5

7. dx
x x
x

+ 2
2
3
Rta:
c x Ln x Ln x x + + + + 1
3
1
2
3
8
2
1
2
















UNIVERSIDADNACIONALABIERTAYADISTANCIA UNAD
ESCUELADECIENCIASBSICAS,TECNOLOGAEINGENIERA
CONTENIDODIDCTICODELCURSO:100411ClculoIntegral

Leccin 27: Integracin de funcin exponencial.

+ = c e dx e
x x


Toda funcin exponencial tiene una base a > 0 y diferente de uno.
Funcin:
x
a y =

Para definir la integral de este tipo de funcin, podemos partir de la derivada y
proceder a obtener la integral, veamos:
x
a y =
Entonces:
) (a Ln a
dx
dy
x
=
para a 1. Para resolver la integral, hacer
una transformacin de la siguiente manera:
) ( ) ( a xLn a Ln x
e e a
x
= =

Aplicando la integral:

= dx e dx a
a xLn x ) (
Multiplicamos y dividimos por
) (
1
a Ln
tenemos:

dx a Ln e
a Ln
a xLn
) (
) (
1
) (
Hacemos cambio de variable: dx a Ln du a xLn u ) ( ) ( = =
c e
a Ln
e
a Ln
du e
a Ln
a xLn u u
+ = =

) (
) (
1
) (
1
) (
1
Pero
x a xLn
a e =
) (
,
luego hacemos el reemplazo para obtener:

( )

+ = c
a Ln
a
dx a
x
x

UNIVERSIDADNACIONALABIERTAYADISTANCIA UNAD
ESCUELADECIENCIASBSICAS,TECNOLOGAEINGENIERA
CONTENIDODIDCTICODELCURSO:100411ClculoIntegral

Ejemplo 1:
Desarrollar:

dx
x
2

Solucin:
Segn la frmula obtenida la integracin ser de la siguiente manera:
c
Ln
dx
x x
+ =

2
) 2 (
1
2

Ejemplo 2:
Resolver

dx
x 2
3

Solucin:
Para facilitar el proceso hagamos un cambio de variable u = 2x luego du = 2dx
reemplazando:
u u u x
Ln
du
du
dx 3
) 3 (
1
*
2
1
3
2
1
2
3 3
2
= = =
Reemplazando la
variable u por 2x obtenemos:
c
Ln
dx
x x
+ =

2 2
3
) 3 ( 2
1
3

Ejemplo 3:
Resolver
dx
x

1
0
3
4

Solucin:
Siguiendo el procedimiento anterior podemos hallar dicha integral, solo que aqu
debemos evaluarla.

UNIVERSIDADNACIONALABIERTAYADISTANCIA UNAD
ESCUELADECIENCIASBSICAS,TECNOLOGAEINGENIERA
CONTENIDODIDCTICODELCURSO:100411ClculoIntegral

dx
x

1
0
3
4
Como u = 3x, entonces:
u u x
Ln
du
dx 4
) 4 (
1
*
3
1
3
4 4
1
0
3
= =


Reemplacemos la variable:
1
0
3 3
4
) 4 ( 3
1
4
) 4 (
1
*
3
1
x x
Ln Ln
=
Evaluando:
) 4 ( 3
63
) 4 ( 3
4
) 4 ( 3
4
4
0 3
1
0
3
Ln Ln Ln
dx
x
= =


Funcin:
x
e y =

La funcin exponencial natural tiene como base el nmero de Euler, para
determinar su integral, realizamos el mismo procedimiento que el caso anterior.
dx e dy e
dx
dy
e y
x x x
= = =
Aplicamos la integral a ambos lados:
c e y dx e dy
x x
+ = =

Por consiguiente:

+ = c e dx e
x x


Ejemplo 1:
Hallar la integral de
x
e
3

Solucin:
Haciendo cambio de variable u = 3x, luego du = 3dx, reemplazando:
c e du e
du
e dx e
u u u x
+ = = =

3
1
3
1
3
3
Finalmente reemplazamos u por 3x.
c e dx e
x x
+ =

3 3
3
1

UNIVERSIDADNACIONALABIERTAYADISTANCIA UNAD
ESCUELADECIENCIASBSICAS,TECNOLOGAEINGENIERA
CONTENIDODIDCTICODELCURSO:100411ClculoIntegral

Ejemplo 2:
Desarrollar:
dx e
x

+1 3

Solucin:

Definimos u = 3x + 1, luego du = 3dx, entonces:
c e du e
du
e dx e
u u u x
+ = = =

+
3
1
3
1
3
1 3
Reemplazando u por 3x +1
obtenemos:
c e dx e
x x
+ =
+ +

1 3 1 3
3
1


Leccin 28: Integracin de funcin logartmica.

Al igual que en la funcin exponencial, las funciones logartmicas tienen su
derivada. Es pertinente recordar las propiedades de este tipo de funciones.
Funcin:
( ) x Ln y =

La integral de esta funcin se puede hacer por partes.

dx x Ln ) ( donde u = Ln(x), luego du = dx / x.


Por otro lado: dv = dx, luego v = x
Reemplazando:

+ = = c x d x xLn dx
x
x x xLn dx x Ln ) (
1
) ( ) (

UNIVERSIDADNACIONALABIERTAYADISTANCIA UNAD
ESCUELADECIENCIASBSICAS,TECNOLOGAEINGENIERA
CONTENIDODIDCTICODELCURSO:100411ClculoIntegral

Finalmente:

( ) ( ) c x x xLn dx x Ln + =



Funcin:
( ) x Log y
10
=

Para resolver este tipo de integral, aplicamos la conversin de cualquier logaritmo
en logaritmo natural, lo cual se hace de la siguiente manera:
) 10 (
) (
) (
10
Ln
x Ln
x Log = As, podemos desarrollar la integral.

= = dx x Ln
Ln
dx
Ln
x Ln
dx x Log ) (
) 10 (
1
) 10 (
) (
) (
10
La ltima integral, ya se desarrollo
por partes, luego:

( )
( )
( ) [ ]

+ = c x x xLn
Ln
dx x Log
10
1
10

Una forma alternativa para esta integral es:

( ) ( ) c e xLog x xLog dx x Log + =

10 10 10

Intente demostrar la ltima igualdad, ser muy interesante.
Si la base del logaritmo es cualquier a > 0 y a 1, la integral es de la misma
manera, solo cambia la base.
Ejemplo 1:
Resolver:
dx
x
x Log

) (
2

Solucin:

UNIVERSIDADNACIONALABIERTAYADISTANCIA UNAD
ESCUELADECIENCIASBSICAS,TECNOLOGAEINGENIERA
CONTENIDODIDCTICODELCURSO:100411ClculoIntegral

Primero hacemos la transformacin a logaritmo natural y luego hacemos una
sustitucin para poder integrar, veamos:
dx
x
x Ln
Ln
dx
xLn
x Log
dx
x
x Log

= =
) (
) 2 (
1
) 2 (
) ( ) (
2 2
para x > 0
Por sustitucin: u = Ln(x), du = (1/x)dx Si reemplazamos:
c u
Ln
udu
Ln
dx
x
x Ln
Ln
+ = =

2
2
1
*
) 2 (
1
) 2 (
1 ) (
) 2 (
1
Reemplazamos u por
Ln(x), para obtener:
c
Ln
x Ln
dx
x
x Log
+ =

) 2 ( 2
) ( ) (
2
2

Ejemplo 2:
Desarrollar: dx
x
x Log Ln

4
1
2
) ( ) 2 (

Solucin:
dx
x
x Log
Ln dx
x
x Log Ln

=
) (
) 2 (
) ( ) 2 (
2
4
1
2
Esta integral se desarroll en el ejemplo
anterior, luego:
2
) 4 (
2
) 1 (
2
) 4 (
2
) (
) 2 ( 2
) (
* ) 2 (
) (
) 2 (
2 2 2
4
1
2
4
1
2
2
Ln Ln Ln x Ln
Ln
x Ln
Ln dx
x
x Log
Ln = = = =


Ejercicios:
Resolver las siguientes integrales, desarrollando paso por paso, para justificar la
respuesta obtenida.
1.
dx e
x

+1 3
Rta:
c e
x
+
+1 3
3
1

UNIVERSIDADNACIONALABIERTAYADISTANCIA UNAD
ESCUELADECIENCIASBSICAS,TECNOLOGAEINGENIERA
CONTENIDODIDCTICODELCURSO:100411ClculoIntegral

2.

dx x
x
) 2 (
2
Rta:
c
Ln
x
+
1
2
2
) 2 (
1

3.
( ) dx e e
x x


+
2
2 2
2
Rta: c e e x
x x
+ +

4 4

4.
( )
dx
x
x Ln

+
2
1 ) (
Rta:
( ) c x Ln + +
3
1 ) (
3
1

5.
dx
x
x Ln

) (
2
Rta:
c x Ln + ) (
3
1
3



Leccin 29: Integracin de la funcin trigonomtrica
( ) ( ) c nx sen
n
dx nx + =

1
cos

Dentro de las funciones trascendentales tenemos las funciones trigonomtricas,
las cuales tambin tienen integrales, analizaremos a continuacin las integrales de
dichas funciones.

Funcin:
( ) x sen y =

La integral de la funcin sen(x) la desarrollamos por el concepto de antiderivada,
pero es pertinente relacionarla en este momento, ya que estamos analizando la
integral de las funciones trigonomtricas.

( ) ( ) c nx
n
dx nx sen +

cos
1

UNIVERSIDADNACIONALABIERTAYADISTANCIA UNAD
ESCUELADECIENCIASBSICAS,TECNOLOGAEINGENIERA
CONTENIDODIDCTICODELCURSO:100411ClculoIntegral

Demostracin:
Tomemos:

dx nx sen ) ( y hacemos una sustitucin: u = nx, du = ndx, luego:



= = du u sen
u u
du
u sen dx nx sen ) (
1
) ( ) ( La funcin cuya derivada es sen(u), es el
cos(u), por consiguiente:
c x
n
c x
n
dx nx sen + = + =

) cos(
1
)) cos( (
1
) (

Funcin:
( ) x y cos =

La integral de cos(x), tambin se desarroll por el concepto de antiderivada.

( ) ( ) c nx sen
n
dx nx + =

1
cos

Demostracin:
Tomemos:

dx nx) cos( y hacemos una sustitucin: u = nx, du = ndx, luego:



= = du u
n n
du
u dx nx ) cos(
1
) cos( ) cos( La funcin cuya derivada es cos(u), es el
sen(u), luego:
c nx sen
n
dx nx + =

) (
1
) cos(


Funcin:
( ) x y tan =


Con los conocimientos sobre identidades trigonomtricas y la integracin por
cambio de variable, podemos hallar la integral de la tangente, veamos:

( ) ( )

+ = c x Ln dx x cos tan

UNIVERSIDADNACIONALABIERTAYADISTANCIA UNAD
ESCUELADECIENCIASBSICAS,TECNOLOGAEINGENIERA
CONTENIDODIDCTICODELCURSO:100411ClculoIntegral

Demostracin:
Descomponemos la tangente en su equivalencia con seno y coseno, as:

= dx
x
x sen
dx x
) cos(
) (
) tan( Sea u = cos(x), luego du = -sen(x)dx, reemplazando:
c u Ln
u
du
dx
x
x sen
+ = =

) cos(
) (
Reemplazando el valor de u, obtenemos:
c x Ln dx x + =

) cos( ) tan(


Funciones:
( ) x y cot = ( ) x y sec = ( ) x y csc =

Con los conocimientos adquiridos, se pueden obtener las siguientes integrales.
( ) ( )

+ = c x sen Ln dx x cot

( ) ( ) ( )

+ + = c x x Ln dx x tan sec sec




( ) ( ) ( )

+ = c x x Ln dx x cot csc csc



Demostracin:
Vamos a demostrar la integral de sec(x) y las dems se dejan como ejercicio para
que lo resuelvan en el pequeo grupo colaborativo y luego lo compartan con el
docente.

+
+
=
+
+
= dx
x x
x x x
dx
x x
x x
x dx x
) tan( ) sec(
) tan( ) sec( ) ( sec
)
) tan( ) sec(
) tan( ) sec(
* ) sec( ) sec(
2

Por sustitucin: ( )dx x x x du x x u ) ( sec ) tan( ) sec( ) tan( ) sec(
2
+ = + =
Reemplazando:

UNIVERSIDADNACIONALABIERTAYADISTANCIA UNAD
ESCUELADECIENCIASBSICAS,TECNOLOGAEINGENIERA
CONTENIDODIDCTICODELCURSO:100411ClculoIntegral

c u Ln
u
du
dx
x x
x x x
+ = =
+
+

) tan( ) sec(
) tan( ) sec( ) ( sec
2

Como u = sec(x) + tan(x), reemplazamos para obtener finalmente:
c x x Ln dx x + + =

) tan( ) sec( ) sec(



Para la csc(x) la demostracin es similar.
Funciones tipo:
( ) x sen y
n
=

( ) x y
n
cos =

Para resolver integrales de este tipo, echamos mano de las identidades
trigonomtricas, las potencias y las tcnicas de integracin estudiadas. Algunos
ejemplos nos ayudarn a comprender la metodologa.
Ejemplo 1:
Resolver:

dx x sen ) (
2

Solucin:
Por la identidad:
2
) 2 cos( 1
) (
2
x
x sen

= Reemplazamos
( )

=


= dx x dx
x
dx x sen ) 2 cos( 1
2
1
2
) 2 cos( 1
) (
2

Por la propiedad de la linealidad:
c x sen x dx x dx +

=

) 2 (
2
1
2
1
2
1
) 2 cos(
2
1
2
1
Finalmente:
c x sen x dx x sen + =

) 2 (
4
1
2
1
) (
2

Ejemplo 2:

UNIVERSIDADNACIONALABIERTAYADISTANCIA UNAD
ESCUELADECIENCIASBSICAS,TECNOLOGAEINGENIERA
CONTENIDODIDCTICODELCURSO:100411ClculoIntegral

Resolver la siguiente integral:

dx x) ( cos
4

Solucin:
Podemos expresar ( ) ( ) ) 2 ( cos ) 2 cos( 2 1
4
1
2
) 2 cos( 1
) ( cos ) ( cos
2
2
2
2 4
x x
x
x x + =


= =
Aplicndole la integral:
( ) dx x x dx x

+ = ) 2 ( cos ) 2 cos( 2 1
4
1
) ( cos
2 4
=

+ dx x dx x dx ) 2 ( cos
4
1
) 2 cos(
2
1
4
1
2
Desarrollando obtenemos:
dx x x sen x

+ + ) 2 ( cos
4
1
) 2 (
4
1
4
1
2

Para resolver la ltima integral, utilizamos la identidad siguiente:
2
) 4 cos( 1
) 2 ( cos
2
x
x
+
= luego:
( )dx x dx
x
dx x

+ =

+
= ) 4 cos( 1
8
1
2
) 4 cos( 1
4
1
) 2 ( cos
4
1
2
=
c x sen x dx x dx + + = +

) 4 (
32
1
8
1
) 4 cos(
8
1
8
1
Reagrupando los resultados:
c x sen x x sen x + + + + ) 4 (
32
1
8
1
) 2 (
4
1
4
1
Operando trminos semejante:
x x x
8
3
8
1
4
1
= + Finalmente:
c x sen x sen x dx x + + + =

) 4 (
32
1
) 2 (
4
1
8
3
) ( cos
4

UNIVERSIDADNACIONALABIERTAYADISTANCIA UNAD
ESCUELADECIENCIASBSICAS,TECNOLOGAEINGENIERA
CONTENIDODIDCTICODELCURSO:100411ClculoIntegral

Funciones tipo:
( ) ( ) x x sen y
n m
cos =

Siendo m y n par. Cuando m y n son pares positivos, usamos las identidades de
ngulo mitad, para reducir el grado del integrado.
Recordemos:

2
) 2 cos( 1
) (
2
x
x sen

=
y
2
) 2 cos( 1
) ( cos
2
x
x
+
=

Ejemplo 1:
Resolver: dx x x sen

) ( cos ) (
2 2

Solucin:
Aplicando la identidad que se referencia
dx
x x
dx x x sen

+
=
2
) 2 cos( 1
*
2
) 2 cos( 1
) ( cos ) (
2 2
Desarrollando:
( )( ) dx x dx dx x x

= + ) 2 ( cos
4
1
4
1
) 2 cos( 1 ) 2 cos( 1
4
1
2

Por la identidad referenciada anteriormente:
2
) 4 cos( 1
) 2 ( cos
2
x
x
+
=
Reemplazamos:
( )dx x x dx
x
x dx x dx

+ =

+
= ) 4 cos( 1
8
1
4
1
2
) 4 cos( 1
4
1
4
1
) 2 ( cos
4
1
4
1
2

Resolviendo la ltima integral, obtenemos:
( ) c x sen x x dx x dx x dx x x + = = +

) 4 (
32
1
8
1
4
1
) 4 cos(
8
1
8
1
4
1
) 4 cos( 1
8
1
4
1
Simplificando trminos semejantes: x x x
8
1
8
1
4
1
=

UNIVERSIDADNACIONALABIERTAYADISTANCIA UNAD
ESCUELADECIENCIASBSICAS,TECNOLOGAEINGENIERA
CONTENIDODIDCTICODELCURSO:100411ClculoIntegral

Finalmente:
c x sen x dx x x sen + =

) 4 (
32
1
8
1
) ( cos ) (
2 2

Ejemplo 2:
Utilizando los argumentos que se han trabajado, mostrar que:
c x sen x sen x dx x x sen + + =

) 2 (
48
1
) 4 (
64
1
16
1
) ( cos ) (
3 4 2

Trabjelo con el pequeo grupo colaborativo y luego comprtalo con el docente
para identificar posibles fallas.
Sugerencia: ...
2
) 2 cos( 1
) ( cos
2
4

+
=
x
x Lo dems es como se ha venido
trabajando.
Funciones tipo:
( ) ( ) x x sen y
n m
cos =


Siendo m n par. Para el caso donde m o n es un entero par y el otro
cualquier valor, se utiliza la factorizacin con la identidad fundamental.
Ejemplo 1:
Desarrollar la integral dada:
dx x sen x

) ( ) ( cos
3 4

Solucin:
Descomponemos el integrado as:
( ) dx x sen x ) ( cos 1 ) ( cos
2 4



Por cambio de variable: u = cos(x) luego du = -sen(x)dx, entonces:
( ) ( )( ) ( )du u u du u u dx x sen x

= =
6 4 2 4 2 4
1 ) ( cos 1 ) ( cos

UNIVERSIDADNACIONALABIERTAYADISTANCIA UNAD
ESCUELADECIENCIASBSICAS,TECNOLOGAEINGENIERA
CONTENIDODIDCTICODELCURSO:100411ClculoIntegral

Operando la integral:
( ) c u u du u u + + =

7 5 6 4
7
1
5
1
Reemplazando la variable:
c x x dx x sen x + =

) ( cos
5
1
) ( cos
7
1
) ( ) ( cos
5 7 3 4


Ejemplo 2:
Mostrar que
c x dx x sen x + =

) ( cos
5
1
) ( ) ( cos
5 4

Solucin:
Por cambio de variable u = cos(x), luego du = -sen(x)dx, entonces:

= = du u du u dx x sen x
4 4 4
) ( ) ( ) ( cos

Operando la integral, tenemos:
c u du u + =

5 4
5
1

Reemplazando la equivalencia de u por cos(x), tenemos:
c x dx x sen x + =

) ( cos
5
1
) ( ) ( cos
5 4
As queda demostrada esta
integral.
Funciones tipo:
( ) ( )dx nx mx sen cos

( ) ( )dx nx sen mx sen

( ) ( )dx nx mx cos cos

Para resolver este tipo de integrales, muy utilizadas en las reas de ingeniera
elctrica, qumica, de alimentos y otras, se utilizan identidades trigonomtricas
que permiten resolver integrales de este tipo.

UNIVERSIDADNACIONALABIERTAYADISTANCIA UNAD
ESCUELADECIENCIASBSICAS,TECNOLOGAEINGENIERA
CONTENIDODIDCTICODELCURSO:100411ClculoIntegral

Las identidades que resuelven el problema son:
1.
[ ] x n m sen x n m sen nx mx sen ) ( ) (
2
1
) cos( ) ( + + =

2.
[ ] x n m x n m nx sen mx sen ) cos( ) cos(
2
1
) ( ) ( + =

3.
[ ] x n m x n m nx mx ) cos( ) cos(
2
1
) cos( ) cos( + + =

Algunos ejemplos modelos nos ilustrarn el proceso.
Ejemplo 1:
Resolver:
dx x x sen

) 5 cos( ) 4 (

Solucin:
Utilizando la primera identidad tenemos:
dx x sen x sen dx x x sen ) ) 5 4 ( ) 5 4 ( (
2
1
) 5 cos( ) 4 (

+ + =

[ ]dx x sen x sen dx x sen x sen

+ = + + ) ( ) 9 (
2
1
) ) 5 4 ( ) 5 4 ( (
2
1

Desarrollando la integral, tenemos:
[ ] [ ]

= dx x sen dx x sen dx x sen x sen ) ( ) 9 (
2
1
) ( ) 9 (
2
1

[ ] dx x sen dx x sen dx x sen dx x sen

+ = ) (
2
1
) 9 (
2
1
) ( ) 9 (
2
1

c x x dx x sen dx x sen + + = +

) cos(
2
1
) 9 cos(
18
1
) (
2
1
) 9 (
2
1

UNIVERSIDADNACIONALABIERTAYADISTANCIA UNAD
ESCUELADECIENCIASBSICAS,TECNOLOGAEINGENIERA
CONTENIDODIDCTICODELCURSO:100411ClculoIntegral

Por consiguiente:
c x x dx x x sen + =

) 9 cos(
18
1
) cos(
2
1
) 5 cos( ) 4 (

Ejemplo 2:
Resolver: dx x x

) 4 cos( ) cos(
Solucin:
Aplicando la identidad 3 de las vistas atrs podemos resolver dicha integral.
[ ]dx x x dx x x

+ + = ) 4 1 cos( ) 4 1 cos(
2
1
) 4 cos( ) cos(

[ ] [ ]dx x x dx x x

+ = + + ) 3 cos( ) 5 cos(
2
1
) 4 1 cos( ) 4 1 cos(
2
1

[ ] dx x dx x dx x x

+ = + ) 3 cos(
2
1
) 5 cos(
2
1
) 3 cos( ) 5 cos(
2
1

c x sen x sen dx x dx x + + = +

) 3 (
6
1
) 5 (
10
1
) 3 cos(
2
1
) 5 cos(
2
1

Finalmente:
c x sen x sen dx x x + + =

) 3 (
6
1
) 5 (
10
1
) 4 cos( ) cos(

Ejercicios:
Desarrollar los siguientes ejercicios, identificando con qu principio se est
trabajando, para su solucin.
1.
dx x x sen

) cos( ) (
4
Rta:
c x sen + ) (
5
1
5

2.
dx x sen x

) ( ) ( cos
2 2
Rta:
c x sen x + ) 4 (
32
1
8
1

UNIVERSIDADNACIONALABIERTAYADISTANCIA UNAD
ESCUELADECIENCIASBSICAS,TECNOLOGAEINGENIERA
CONTENIDODIDCTICODELCURSO:100411ClculoIntegral

3.
dx x x sen

) 4 ( cos ) 4 (
2 5
Rta:
c x x x + + ) 4 ( cos
28
1
) 4 ( cos
10
1
) 4 ( cos
12
1
7 5 3

4.
dx
x x
sen

2
cos
2
2 4
Rta: c x sen x sen x + ) (
24
1
) 2 (
32
1
16
1
3

5.
dx x x

) cos( ) 2 cos(
Rta:
c x sen x x x sen + ) ( ) 2 cos(
3
1
) cos( ) 2 (
3
2

6.
( )dx x sen Ln x

) ( ) cos(
Rta: ( ) c x sen x sen Ln x sen + ) ( ) ( ) (
















UNIVERSIDADNACIONALABIERTAYADISTANCIA UNAD
ESCUELADECIENCIASBSICAS,TECNOLOGAEINGENIERA
CONTENIDODIDCTICODELCURSO:100411ClculoIntegral

Leccin 30: Integracin de la funcin hiperblica.
En cursos anteriores se ha analizado las funciones hiperblicas y sus derivadas,
en este aparte se estudiarn las integrales de dichas funciones.
Funcin senh(x): Recordemos que el seno hiperblico es de la forma:
( )
x x
e e

2
1
y su derivada es el cosh(x), lo que est plenamente demostrado
Entonces para obtener la integral de cosh(x), utilizamos el principio de la primitiva.

( ) ( )

+ = c x dx x senh cosh

Demostracin:
Por el principio de la antiderivada, sabemos que la funcin cuya derivada es el
senh(x) corresponde al cosh(x), as se justifica la integral del senh(x).
Funcin cosh(x): El coseno hiperblico es de la forma
( )
x x
e e

+
2
1
y su derivada
es senh(x), luego:

( ) ( )

+ = c x senh dx x cosh

Demostracin:
Se deja como ejercicio para que el estudiante con los principios aprendidos lo
demuestre y por supuesto lo comparta con sus compaeros y su tutor.
Funcin tanh(x): Por definicin sabemos que:
) cosh(
) (
) tanh(
x
x senh
x = .

( ) ( )

+ = c x Ln dx x cosh tanh

Demostracin:
Apliqumosle la identidad a la integral, entonces:

= dx
x
x senh
dx x
) cosh(
) (
) tanh(

hacemos cambio de variable:

UNIVERSIDADNACIONALABIERTAYADISTANCIA UNAD
ESCUELADECIENCIASBSICAS,TECNOLOGAEINGENIERA
CONTENIDODIDCTICODELCURSO:100411ClculoIntegral

u = cosh(x) y du = senh(x)dx, reemplazando:
c u Ln
u
du
dx
x
x senh
dx x + = = =

) cosh(
) (
) tanh(
Reemplazando el valor de u
tenemos:
c x Ln dx x + =

) cosh( ) tanh(

Funcin coth(x): De la misma manera que la tangente hiperblica, la cotangente
hiperblica tiene su equivalencia:
) (
) cosh(
) coth(
x senh
x
x = donde:

( ) ( )

+ = c x senh Ln dx x coth

Demostracin:
Se deja como ejercicio para hacer en el pequeo grupo colaborativo.
Funcin sech(x) y csch(x): Al igual que las funciones hiperblicas anteriores,
estas ltimas funciones, tambin tiene su integral. La demostracin se
recomienda trabajarla con el tutor, para afianzar los conocimientos sobre este tipo
de funciones.
( ) ( )

+ =

c x senh dx x h
1
tan sec


( )

= c
x
Ln dx x h
2
tanh csc

Ejemplo 1:
Resolver:

dx x) 6 cosh(

Solucin:
Hacemos cambio de variable u = 6x, du = 6dx, entonces reemplazamos:

UNIVERSIDADNACIONALABIERTAYADISTANCIA UNAD
ESCUELADECIENCIASBSICAS,TECNOLOGAEINGENIERA
CONTENIDODIDCTICODELCURSO:100411ClculoIntegral

c u senh du u
du
u dx x + = = =

) (
6
1
) cosh(
6
1
6
) cosh( ) 6 cosh(

Como u = 6x, reemplazamos a u por 6x, para dejar la integral en funcin de x y no
de u.
c x senh dx x + =

) 6 (
6
1
) 6 cosh(

Ejemplo 2:
Desarrollar:

dx x) 3 tanh(

Solucin:
Como en el caso anterior, hacemos cambio de variable u = 3x, du = 3dx,
entonces:
c u Ln du u
du
u dx x + = = =

) cosh(
3
1
) tanh(
3
1
3
) tanh( ) 3 tanh(
reemplazando el valor de u por 3x, obtenemos:
c x Ln dx x + =

) 3 cosh(
3
1
) 3 tanh(

Ejemplo 3:
Resolver:

dx x senh ) (
2

Solucin:
Para resolver esta integral, primero debemos aplicar la identidad que dice:
2
1 ) 2 cosh(
) (
2

=
x
x senh
Luego:

UNIVERSIDADNACIONALABIERTAYADISTANCIA UNAD
ESCUELADECIENCIASBSICAS,TECNOLOGAEINGENIERA
CONTENIDODIDCTICODELCURSO:100411ClculoIntegral

dx
x
dx x senh


=
2
1 ) 2 cosh(
) (
2
Desarrollando tenemos:
( )

= =


dx dx x dx x dx
x
2
1
) 2 cosh(
2
1
1 ) 2 cosh(
2
1
2
1 ) 2 cosh(

Operando tenemos:
c x x senh +
2
1
) 2 (
2
1
*
2
1
Resumiendo:
c x x senh +
2
1
) 2 (
4
1

Finalmente:
c x x senh dx x senh + =

2
1
) 2 (
4
1
) (
2

Ejercicios:
1.
dx
x

7
tanh
Rta:
c e e Ln
x x
+ +

7 7
7

2.
dt
t
t t h

tanh sec
Rta:
c t h + sec 2

3.
dx x senh

) 2 1 (
Rta:
c x + ) 2 1 cosh(
2
1

4.
dx
x senh
x

) (
) cosh(
Rta:
c x senh Ln + ) (

5.
dx x x

+ ) 5 cosh(
2

Rta:
c x senh + + ) 5 (
2
1
2


6.

dx x sen senh x )) ( ( ) cos(


Rta:
c x sen + )) ( cosh(

UNIVERSIDADNACIONALABIERTAYADISTANCIA UNAD
ESCUELADECIENCIASBSICAS,TECNOLOGAEINGENIERA
CONTENIDODIDCTICODELCURSO:100411ClculoIntegral

ACTIVIDADES DE AUTOEVALUACIN DE LA UNIDAD
PREGUNTAS DE SELECCIN MLTIPLE CON NICA RESPUESTA
A continuacin, usted encontrar preguntas que se desarrollan en torno a un
enunciado, problema o contexto, frente al cual, usted debe seleccionar aquella que
responde correctamente a la pregunta planteada entre cuatro opciones
identificadas con las letras A, B, C, D. Una vez la seleccione, mrquela en su hoja
de respuestas rellenando el valo correspondiente.

1. Al resolver las siguientes integrales

dx
x
2 y

dx x) ( sec
2
,se obtiene:
A.
( )
c
Ln
x
+
2
2

B.
( ) c x Tg +

C.
( )
c
Ln
x
+
2
2

D.
( ) c x Sen +


2. Cuando se desarrolla la expresin
dx
x sen
x x sen


) (
) cos( ) (
,se obtiene:
A.
( ) [ ] c x sen Ln x +

B.
( ) [ ] c x Ln + + cos 2

C.
( ) [ ] c x Ln + + cos 1

D.
( ) [ ] c x sen Ln + 2

UNIVERSIDADNACIONALABIERTAYADISTANCIA UNAD
ESCUELADECIENCIASBSICAS,TECNOLOGAEINGENIERA
CONTENIDODIDCTICODELCURSO:100411ClculoIntegral

3. Si se desea resolver la integral de la funcin
2 2
x b la sustitucin mas
adecuada es:
A.
( ) x bsen x =

B.
( ) x bTg x =

C.
( ) x b x sec =

D.
( ) x b x cos =


4. El valor de la integral
( )
( )( )

+

dx
x x x
x
1
1
2
2
,es:
A. ( ) k x Ln + 1
B.
( ) k x Ln + +
2
1
C.
( ) k x Ln + + 1
2

D. ( ) k x Ln +

5. El procedimiento para solucionar la integral
( )
( )( )

+
+
dx
x x
x x x
6 3
18 3
2 3
,es:
A. Sustitucin trigonomtrica
B. Por partes
C. Simplificacin
D. Racionalizacin

UNIVERSIDADNACIONALABIERTAYADISTANCIA UNAD
ESCUELADECIENCIASBSICAS,TECNOLOGAEINGENIERA
CONTENIDODIDCTICODELCURSO:100411ClculoIntegral

HOJA DE RESPUESTAS.

A B C D
1
2
3
4
5

UNIVERSIDADNACIONALABIERTAYADISTANCIA UNAD
ESCUELADECIENCIASBSICAS,TECNOLOGAEINGENIERA
CONTENIDODIDCTICODELCURSO:100411ClculoIntegral

FUENTES DOCUMENTALES DE LA UNIDAD 2

RONDON, J.E (2007) Calculo Integral. Primera edicin, UNAD Ciencias bsicas
PURCELL, E (2001) Clculo, Pearson Education: Prentice hall, Octava Edicin,
Mxico.
THOMAS Y FINNEY (1987). Clculo con Geometra Analtica Vol. 1. Edicin
sexta, Addison Wesley Iberoamericana. Mxico.
STEWART, J. (2001) Clculo de una Variable. Thomsom-Learning. Cuarta edicin,
Bogot.
LARSON, R. Y HOSTETLER, R. (1998) Clculo Vol. 1, Mc Graw Hill, sexta
edicin, Mxico.
SMITH, R. Y MINTON, R. (2002) Clculo Vol. 1. Segunda Edicin, Mc Graw Hill,
Bogot.
BAUM Y MILLES. (1992). Clculo Aplicado. Limusa, Mxico.
LEYTOLD, L. (1987) El Clculo con Geometra Analtica. Harla, Mxico.
PITA, C. (1998) Clculo de una Variable. Pearson educacin, Mxico.
DE BURGOS, J. (2007) Clculo infinitesimal de una Variable. McGraw Hill, Madrid.

FUENTES DOCUMENTALES DE LA INTERNET

http://www.xtec.cat/~jlagares/integral.esp/integral.htm
http://thales.cica.es/rd/Recursos/rd97/Problemas/541pIntegral.html
http://sigma.univalle.edu.co/index_archivos/calculo1y2/formulasdecalculo1y2.pdf
http://www.matematicasbachiller.com/temario/calculin/index.html
http://es.wikipedia.org/wiki/Teorema_fundamental_del_c%C3%A1lculo
http://www.aulafacil.com/matematicasintegrales/curso/Temario.htm

UNIVERSIDADNACIONALABIERTAYADISTANCIA UNAD
ESCUELADECIENCIASBSICAS,TECNOLOGAEINGENIERA
CONTENIDODIDCTICODELCURSO:100411ClculoIntegral
http://www.monografias.com/trabajos10/historix/historix.shtml
http://www.fata.unam.mx/tecnologia/material/sem01/Calculo_I_Historia_1.pdf
http://www.uam.es/personal_pdi/ciencias/barcelo/histmatem/calculo/calculo.html
http://integrals.wolfram.com/index.jsp?expr=x%5E2*%28x4%29%5E0.5&random=false
http://www.dma.fi.upm.es/docencia/primerciclo/calculo/tutoriales/integracion/
http://www.matematicasypoesia.com.es/ProbIntegral/problema110.htm
http://usuarios.iponet.es/ddt/logica1.htm

UNIVERSIDADNACIONALABIERTAYADISTANCIA UNAD
ESCUELADECIENCIASBSICAS,TECNOLOGAEINGENIERA
CONTENIDODIDCTICODELCURSO:100411ClculoIntegral


UNIDAD 3: APLICACIN DE LAS INTEGRALES.

Introduccin:


Existen mltiples aplicaciones de las integrales en la ingeniera civil, elctrica,
electrnica, industrial, economa, en la hidrulica, en el trabajo, en el movimiento,
en la estadstica, etc.
En esta unidad se insiste en las tcnicas de solucin de las integrales vistas en la
unidad anterior, sino tambin en los principios propios de cada tipo de problema
de aplicacin partiendo del anlisis de graficas (rea bajo curvas, longitud de
curvas), hallar los volmenes de slidos de revolucin mediante diferentes
tcnicas, centros de masa y por ltimo la aplicacin en la solucin de problemas
prcticos de la fsica, la hidrulica, la estadstica y la economa.
El manejo de este tipo de problemas su entendimiento y posterior aplicacin en la
vida profesional del estudiante es clave y fundamental como objetivo del presente
modulo.

Justificacin:
En esta unidad nos vamos a concentrar en la realizacin de ejercicios de
aplicacin a algunas de las ciencias ya mencionadas y que el lector puede
necesitar solucionar en su vida profesional.
Hablemos de la aplicacin de las integrales a la astronoma y dentro de ella la
velocidad de escape de un objeto para sacarlo de la rbita de la tierra; al lanzar un
objeto macizo y metlico al aire esta volver a caer a la tierra debido a la fuerza de
la gravedad, si se lanza con ms fuerza volver a caer ms lejos.
Cul es la velocidad inicial
0
v
para sacar ese objeto de la tierra y que permanezca
alejndose?

Para esta TERCERA UNIDAD tenemos tres captulos en los cuales tratamos las
aplicaciones prcticas de las integrales.

UNIVERSIDADNACIONALABIERTAYADISTANCIA UNAD
ESCUELADECIENCIASBSICAS,TECNOLOGAEINGENIERA
CONTENIDODIDCTICODELCURSO:100411ClculoIntegral


Intencionalidades formativas:

Que los estudiantes se observen las aplicaciones en la vida diaria de las
integrales.
La solucin de diversos con la ayuda de las matemticas.
Nuevamente se recalca en la realizacin de diversos problemas de
aplicacin, esto con el fin de adquirir destrezas en el manejo de las
mltiples variables que intervienen en la solucin de dichos problemas.


Presentamos un cuadro con el resumen del contexto terico de esta unidad


Denominacin de
los captulos
CAPITULO 1: ANLISIS DE GRAFICAS.

CAPITULO 2 Volumen de superficies de revolucin

CAPITULO 3 En las ciencias.

Asimilacin de
conceptos

Analizar los caminos adecuados para la solucin de
problemas de aplicacin.

Conceptos

Se presentan problemas de aplicacin sencillos y se indican
todos los pasos para su realizacin.

Competencias


De conocimientos
Adquirir las tcnicas propias para la solucin de
problemas prcticos.
Adquirir conocimiento mediante la realizacin del
mayor nmero posible de ejercicios.

Contextuales:
Adquirir los conocimientos propios de la solucin de
problemas de aplicacin.
Los estudiantes deben desarrollar habilidades para
aplicar los conocimientos adquiridos en la solucin de
problemas prcticos.


UNIVERSIDADNACIONALABIERTAYADISTANCIA UNAD
ESCUELADECIENCIASBSICAS,TECNOLOGAEINGENIERA
CONTENIDODIDCTICODELCURSO:100411ClculoIntegral

Comunicativas:
Adquirir el manejo de los elementos involucrados en
los diferentes mtodos de solucin de problemas
prcticos.
Adquirir facilidad de expresin y vencer el miedo en
la interaccin con las NTIC

Valorativas:
Adoptar, identificar y practicar lo valores de la UNAD.
Adquirir capacidad de valoracin y tolerancia con
nuestros compaeros virtuales o presenciales.


















UNIVERSIDADNACIONALABIERTAYADISTANCIA UNAD
ESCUELADECIENCIASBSICAS,TECNOLOGAEINGENIERA
CONTENIDODIDCTICODELCURSO:100411ClculoIntegral

CAPITULO 7: Anlisis de graficas.

Introduccin




Fig. 11 rea bajo la curva.

Analizados y aprendidos los principios sobre integracin; adems, estudiadas las
diferentes tcnicas de integracin, estamos en capacidad de realizara diversas
aplicaciones que tiene esta maravillosa rea de las matemticas. Las integrales
se pueden aplicar y tiene aplicaciones en Ingeniera, Fsica, Estadstica,
Economa, Administracin, Geometra y otras. Como ejercicio de ilustracin
vamos a abordar diversos contextos que permitan comprender la amplitud que
tiene las integrales como herramienta matemtica para resolver problemas de
diversa ndole.

Leccin 31: rea de regiones planas.
Dentro de las reas de regiones planas, tenemos dos casos, el rea bajo la curva
y el rea bajo curvas. Analicemos estos casos:
rea Bajo Una Curva: Cuando tenemos una lnea recta de la forma como se
ilustra en la figura, el rea se puede calcular por una simple frmula geomtrica.



UNIVERSIDADNACIONALABIERTAYADISTANCIA UNAD
ESCUELADECIENCIASBSICAS,TECNOLOGAEINGENIERA
CONTENIDODIDCTICODELCURSO:100411ClculoIntegral


y a b h b A ) (
2
1
*
2
1
= =

La situacin es relativamente fcil de manejar, la situacin dificulta cuando la
lnea no es recta, sino una curva, para dicho caso el procedimiento es ms largo y
cuidadoso.


Sea y = f(x) una funcin definida en el
intervalo I = [a, b] y continua en el intervalo
abierto I; adems f(x) 0, Consideremos la
regin R acotada por la curva y = f(x), las
rectas x = a y x = b, la idea es hallar el rea
de la regin R.

Fig. 12 Particiones.

Primero dividamos el intervalo I en n subintervalos iguales
n
a b
x

=
Los puntos de la particin:
x
o
= a, x
1
= x
o
+ x, x
2
= x
1
+ x = x
o
+ 2x, , x
i
= x
i-1
+ x = x
o
+ ix para i = 1,
2, 3, , n. Cada subintervalo ser la base de un rectngulo cuya altura ser f(c
i
),
para
c
i
[x
i-1
, x
i
]. luego se obtienen A
i
= f(c
i
) x, al sumar todas las reas se obtiene una
aproximacin al rea de la regin R as:

=
=
n
i
i
x c f A
1
) (


UNIVERSIDADNACIONALABIERTAYADISTANCIA UNAD
ESCUELADECIENCIASBSICAS,TECNOLOGAEINGENIERA
CONTENIDODIDCTICODELCURSO:100411ClculoIntegral

Si aumentamos el nmero de subintervalos; es decir, que n se haga
suficientemente grande, el rea de R ser cada vez ms exacta.

= =
=

b
a
n
i
i
n
dx x f x c f Lim A ) ( ) (
1
Por consiguiente rea bajo la curva:
( )

=
b
a
dx x f A

Ejemplo 1:
Hallar el rea bajo la curva para la funcin f(x) = 2x, entre x = 0 y x = 4.
Solucin:
Aplicando la frmula definida podemos hallar el rea pedida
16 0 4 2
2 2
4
0
2
4
0
= = = =

x xdx A
El rea es de 16 unidades cuadradas.

Si resolvemos el problema
por el mtodo geomtrico; es
decir, aplicando la frmula
para un tringulo, obtenemos
el mismo resultado.

16 8 4
2
1
2
1
= = = x bxh A

Fig. No 13 Grafica de 2x
Ejemplo 2:
Hallar el rea acotada por la curva g(x) = x
3
en el intervalo [1, 3]
FUNCIN y = 2x
0
2
4
6
8
10
0 1 2 3 4 5
Variable x
V
a
r
a
i
b
l
e

y

UNIVERSIDADNACIONALABIERTAYADISTANCIA UNAD
ESCUELADECIENCIASBSICAS,TECNOLOGAEINGENIERA
CONTENIDODIDCTICODELCURSO:100411ClculoIntegral

Solucin:


Por la frmula tenemos:
Fig. No. 14 Grafica de x
3
3
1
4
3
1
3
4
1
x dx x A = =


( ) ( ) 1 81
4
1
1 3
4
1
4
1
4 4
3
1
4
= = = x A

( ) 20
4
80
1 81
4
1
4
1
3
1
4
= = = = x A
Unidades cuadradas
Ejemplo 3:
Calcular el rea bajo la curva
2
3 ) ( x x f = en el intervalo [0, 1]
Solucin:


dx x A ) 3 (
1
0
2

=



1
0
3
3
1
3

= x x A


Fig. No. 15 Grafica de
2
3 x y =

FUNCIN y = x3
0
10
20
30
0 1 2 3 4
Var iable x
f
(
x
)
FUNCIN Y = 3 - x2
0
0,5
1
1,5
2
2,5
3
3,5
0 0,2 0,4 0,6 0,8 1 1,2
Variable x
V
a
r
i
a
b
l
e

y

UNIVERSIDADNACIONALABIERTAYADISTANCIA UNAD
ESCUELADECIENCIASBSICAS,TECNOLOGAEINGENIERA
CONTENIDODIDCTICODELCURSO:100411ClculoIntegral

= o A *
3
1
0
3
1
3

3
8
3
1
3 = = A
Unidades cuadradas.

Leccin 32: rea entre curvas.

Para desarrollar esta temtica, partimos de dos funciones f(x) y g(x), asumiendo
que f(x) g(x) para todo x en el intervalo [a, b], la idea es hallar el rea entre las
curvas f(x) y g(x) sobre el intervalo dado.
El mtodo se hace utilizando rectngulos que aproximen el rea de la regin
descrita.

Dividimos el intervalo en n
subintervalos
n
a b
x

= Los puntos
de la particin sern: x i a x
i
+ = , para
i = 1, 2, 3, Cada subintervalo
[x
i-1
, x
i
]. Forma la base de un
rectngulo cuya altura ser: f(c
i
)-g(c
i
)
para c
i
[x
i-1
, x
i
].


Fig. No. 16 rea entre curvas.

El rea del rectngulo i-simo ser:
A
i
= h
i
* x reemplazando: A
i
= [f(c
i
)-g(c
i
)] * x

UNIVERSIDADNACIONALABIERTAYADISTANCIA UNAD
ESCUELADECIENCIASBSICAS,TECNOLOGAEINGENIERA
CONTENIDODIDCTICODELCURSO:100411ClculoIntegral

El rea total ser la suma de las reas de los n rectngulos, luego:
[ ] x c g c f Lim A
n
i
i i
n
=

=

* ) ( ) (
1


DEFINICIN:
Sean f(x) y g(x) funciones continuas en el intervalo [a, b], luego el rea de la regin
acotada por las curvas f(x) y g(x) desde a hasta b esta dado por:
( ) ( ) [ ]

=
b
a
dx x g x f A

Para hallar el rea entre dos curvas se debe:
1. Hacer La grfica explicativa, para identificar f(x) y g(x) y saber cual es la
funcin superior e inferior
2. Identificar los lmites de integracin
3. Establecer la frmula de integracin
4. Desarrollar la integracin y valorar para hallar el rea.

Ejemplo 1:
Hallar el rea entre las curvas f(x) = 4 x y g(x) = x
2
16.
Solucin:

Hallamos los lmites, que consiste en
buscar en donde x coinciden:
( )( ) 4 5 20 16 4
2 2
+ = + = x x x x x x L
uego los lmites son: x = -5 y x = 4

Fig. No. 17 Grafico solucin problema

UNIVERSIDADNACIONALABIERTAYADISTANCIA UNAD
ESCUELADECIENCIASBSICAS,TECNOLOGAEINGENIERA
CONTENIDODIDCTICODELCURSO:100411ClculoIntegral

Vemos que la funcin f(x) > g(x), entonces:
( ) ( ) [ ]dx x x A

=
4
5
2
16 4
( )
4
5
3 2
4
5
2
3
1
2
1
20 20

= =

x x x dx x x A
Evaluando tenemos:
6
675
6
425
3
125
) 5 (
3
1
) 5 (
2
1
) 5 ( 20 ) 4 (
3
1
) 4 (
2
1
) 4 ( 20
3 2 3 2
= + =

= A

El rea entre las curvas es de 675/6 unidades cuadradas.

Ejemplo 2:
Determinar el rea comprendida entre las curvas f(x) = 2 x
2
y g(x) = x
2
, en el
intervalo [0, 2].
Solucin:






Fig. No. 18 Grafico solucin rea bajo curvas.

Como se conocen los lmites, no hay necesidad de calcularlos, como se pudiera
pensar, luego lo que debemos hacer es utilizar la frmula para obtener el rea,
solo que se debe establecer cual ser la funcin menor y cual la funcin mayor.

UNIVERSIDADNACIONALABIERTAYADISTANCIA UNAD
ESCUELADECIENCIASBSICAS,TECNOLOGAEINGENIERA
CONTENIDODIDCTICODELCURSO:100411ClculoIntegral

El problema lo debemos resolver en dos partes: La primera ser el intervalo de [0,
1], donde la funcin mayor es f(x) = 2 x
2
y la menor g(x) = x
2
. La segunda parte
ser el intervalo [1, 2], donde la funcin mayor es g(x) = x
2
y la menor 2 x
2
.
Para la primera parte:
dx x g x f A

=
1
0
1
)] ( ) ( [
Reemplazando:
( )

= =
1
0
1
0
2
1
0
2 2
2
2 2 2 dx x dx dx x x A

Integrando:
( )
3
4
3
2
2 0
3
2
1
3
2
) 0 ( 2 ) 1 ( 2
3
2
2
3 3
1
0
3
1
0
1
= =

= = x x A
Ahora hallamos la segunda parte.
( ) [ ]dx x x dx x f x g A

= =
2
1
2 2
2
1
2
2 )] ( ) ( [
Operando:
( ) [ ] dx dx x dx x dx x x A

= = =
2
1
2
1
2
2
1
2
2
1
2 2
2
2 2 ) 2 2 ( 2
Integrando:
( ) ( ) [ ] 1 2 2 1 2
3
2
2
3
2
2 2
3 3
2
1
2
1
3
2
1
2
1
2
2

= = =

x x dx dx x A
Desarrollando:
( ) ( ) [ ]
3
8
2
3
14
2 7 *
3
2
1 2 2 1 2
3
2
3 3
2
= = =

= A

Finalmente sumamos las dos reas para hallar el rea total:
4
3
12
3
8
3
4
2 1
= = + = + A A
El rea entre las curvas es de 4 unidades cuadradas.

UNIVERSIDADNACIONALABIERTAYADISTANCIA UNAD
ESCUELADECIENCIASBSICAS,TECNOLOGAEINGENIERA
CONTENIDODIDCTICODELCURSO:100411ClculoIntegral

Ejemplo 3:
Determinar que el rea entre las curvas cuyas funciones son: 2 ) (
2
+ = x x f y
x x g = ) ( en el intervalo [-2, 2] es 40 / 3.
Solucin:




Fig. No. 19 Grafico solucin
[ ]dx x x A ) ( 2
2
2
2
+ =



[ ]dx x x A ) 2
2
2
2
+ + =

Integrando:

2
2
2 3
2
2
1
3
1

+ + = x x x A
Evaluando:

+ + =

+ +

+ + = 4 2
3
8
4 2
3
8
) 2 ( * 2 ) 2 (
2
1
) 2 (
3
1
2 * 2 2
2
1
2
3
1
2 3 2 3
A
3
40
3
14
3
26
= + = A
As el rea entre las curvas son efectivamente 40 / 3 unidades cuadradas.



UNIVERSIDADNACIONALABIERTAYADISTANCIA UNAD
ESCUELADECIENCIASBSICAS,TECNOLOGAEINGENIERA
CONTENIDODIDCTICODELCURSO:100411ClculoIntegral

EJERCICIOS: Leccin No. 2
De los ejercicios propuestos, hallar el rea entre las curvas propuestas
1.
2
2
) (
x
x f = y
3 ) ( = x g
Rta: 18 unidades cuadradas
2.
4 4
2
= +y x
y
1
4
= y x
Rta:
15
104
unidades
cuadradas
3.
) ( cos ) (
2
x x f =
y
1 ) ( = x g
Rta:
2

unidades cuadradas
4.
1 ) (
2
=x x h
y
2
7 ) ( x x j =
Rta:
3
64
unidades cuadradas
5. La interseccin entre
x
e x p = ) ( y
2
1 ) ( x x q = Rta: Trabajarla con el Tutor.
Leccin 33: rea de superficies de revolucin

( )

=
B
A
ds x f a 2

Sabemos que toda curva representa una funcin, que se puede ilustrar en el plano
cartesiano. Si giramos la curva alrededor de uno de sus ejes, se genera una
Superficie de Revolucin, el objetivo es determinar el rea de la superficie
generada.





Fig. No. 20 Superficie de revolucin

UNIVERSIDADNACIONALABIERTAYADISTANCIA UNAD
ESCUELADECIENCIASBSICAS,TECNOLOGAEINGENIERA
CONTENIDODIDCTICODELCURSO:100411ClculoIntegral

En la grfica el giro se est realizando alrededor del eje x. y = f(x) denota una
curva suave, con b x a . Subdividimos el intervalo a = x
o
< x
1
< x
2
< . . . < x
n
= b
As la curva se divide en n partes.
Sea s
i
la longitud de i-simo pedazo de la superficie y y
i
la ordenada, al girar se
observa la banda de color amarillo que se forma. El rea se puede aproximar por
la de un cono truncado; es decir,
i i
s y 2
Al sumar las reas de todas las bandas y tomando el lmite cuando la norma de la
particin tiende a cero, obtenemos lo que llamamos el rea de la superficie de
revolucin.

=
n
i
i i
p
s y Lim A
1
0
2
Aplicando los principios de sumatorias y lmites:

=
b
a
ds x f A ) ( 2
Como ds es el diferencial de longitud y equivale a:
( ) dx x f
2
) ( ' 1+ , reemplazando, obtenemos finalmente la ecuacin del rea de una
superficie de revolucin.
rea generada de la curva f(x) alrededor del eje x:

( ) ( ) [ ] dx x f x f A
b
a

+ =
2
1 2

Para calcular reas de de superficies de revolucin, se puede utilizar las integrales
ordinarias, pero en muchas ocasiones se requiere un mtodo de aproximacin
conocido como los mtodos numricos.
Ejemplo 1:
Hallar el rea de la superficie generada al rotar sobre el eje x la funcin y = x
2
en
el intervalo [0, 1]

UNIVERSIDADNACIONALABIERTAYADISTANCIA UNAD
ESCUELADECIENCIASBSICAS,TECNOLOGAEINGENIERA
CONTENIDODIDCTICODELCURSO:100411ClculoIntegral

Solucin:







Fig. 21 Superficie de revolucin de
2
x y =

Como y = x
2
entonces. y = 2x
Ahora aplicamos la ecuacin para obtener
el rea.
2
x y =
Cuando gira alrededor de x.
( )

+ =
1
0
2
2
2 1 * 2 dx x x A



Esta integral no se puede resolver por los mtodos tradicionales de integracin,
por lo cual se recurre a los mtodos numricos, para s obtener el valor
aproximado de: 3,8097.
Ejemplo 2:
Dada la curva x y = la cual gira alrededor del eje x, cual ser el rea de la
superficie de revolucin generada, para 2 0 x

+ =
1
0
2 2
4 1 2 dx x x A

UNIVERSIDADNACIONALABIERTAYADISTANCIA UNAD
ESCUELADECIENCIASBSICAS,TECNOLOGAEINGENIERA
CONTENIDODIDCTICODELCURSO:100411ClculoIntegral







Fig. No. 22 superficie de revolucin de x y =
Solucin:
Dado que
x
x f x x f
2
1
) ( ' ) ( = =
Ahora: ( )
x
x f
4
1
) ( '
2
= Luego aplicamos la formula:
dx
x
x A

+ =

4
1
1 2
2
0


( )
dx x dx
x
x x
dx
x
x
x A

+ =
+
=

+
=
2
0
2
0
2
0
1 4
4
1 4
2
4
1 4
2

= =

2
3
2
1
2
1
4
1
u du u A Como hicimos cambio de variable, volvemos a
cambiar
la variable original. Entonces:
( ) ( ) 3825 , 42
2
728
1 729
2
1 9
2
1 4
2
3 3
2
0
3
= = = + =

x A

UNIVERSIDADNACIONALABIERTAYADISTANCIA UNAD
ESCUELADECIENCIASBSICAS,TECNOLOGAEINGENIERA
CONTENIDODIDCTICODELCURSO:100411ClculoIntegral

En muchos casos el giro de la curva se hace alrededor del eje y, luego en estos
casos la ecuacin cambio en algunos aspectos.
Si x = f(y), siendo f(y) una curva suave y adems mayor o igual a cero, el rea de
la superficie generada al girar la curva f(y) alrededor del eje y es de la forma:
rea generada de la curva f(y) alrededor del eje y:
( ) ( ) [ ] dy y f y f A
b
a

+ =
2
1 2

Ejemplo 3:
Calcular el rea de la superficie generada al girar alrededor del eje y, de la curva
3
3
1
y x = en 1 0 y
Solucin:

Como
2 3
'
3
1
y x y x = = por otro lado: ( )
4 2 2
' ' y x y x = = Ahora aplicamos la
frmula:
dy y y dy y y A

+ = + =
1
0
4 3
1
0
4 3
1
3
2
1
3
1
2
Por cambio de variable:
dx y du y u
3 4
4 1 = + =
Luego:
du u
du
u dy y y

= = +
2
1
2
1
1
0
4 3
6
1
4 3
2
1
3
2


Integrando:

2
3
3
2
6
1
u Cambiando de nuevo la variable, tenemos:
( ) ( ) 1 8
9
1
9
1
1
0
3
4
=

+ =

y A Por consiguiente:
( ) 1 2 2
9
=

A

UNIVERSIDADNACIONALABIERTAYADISTANCIA UNAD
ESCUELADECIENCIASBSICAS,TECNOLOGAEINGENIERA
CONTENIDODIDCTICODELCURSO:100411ClculoIntegral

EJERCICIOS:
Calcular el rea de la superficie de revolucin generada al girar alrededor del eje
establecido, de las funciones propuestas.
1.
2
4
8
1
4
1
) (
x
x x f + =
Para
2 1 x
Eje Y Rta:
20
253

2.
( )
2
3
2
2
3
1
) ( x x g + =
Para
2 0 x
Eje Y Rta: 12
3.
3
) ( x x h =
Para
1 0 x
Eje X Rta: ( ) 1 10 10
27


4. La parbola
py x 4
2
=
En los puntos (0, 0) y
(2p, p) Eje Y Rta: ( ) 1 2 2
3
8
2
p
5.
2 2
) ( x r x m =
Para
r x r
Eje X Rta:
2
4 r

6.
3
3
1
) ( x x q =
Para
7 1 x
Eje X Rta: 2 28







UNIVERSIDADNACIONALABIERTAYADISTANCIA UNAD
ESCUELADECIENCIASBSICAS,TECNOLOGAEINGENIERA
CONTENIDODIDCTICODELCURSO:100411ClculoIntegral

Leccin 34: Longitud de una curva


( ) ( )
2 2
y x PQ + =

Cuando queremos medir la longitud de una lnea recta, solo colocamos una regla
o metro y hacemos la medicin. La situacin cambia cuando la lnea que se desea
medir es curva, por ejemplo medir la longitud de la cuerda de luz que va de un
poste a otro.






Fig. No. 23 Longitud de curva.
Se desea calcular la longitud de la curva y = f(x) entre [a, b], donde la funcin
es continua. Haciendo la particin de manera usual, y uniendo los puntos de los
segmentos de tal forma que se forme una trayectoria poligonal que aproxime la
curva.
Longitud
2 2
) ( ) ( y x
PQ
+ =



Para calcular la longitud del segmento total, se debe hacer la sumatoria de la
particin, es decir:
( ) ( )

=
+ =
n
i
i i
y x L
1
2 2

La aproximacin a la longitud de la curva mejora, si la particin se hace ms fina;
o sea, la sumatoria tiene lmite calculable, cuando la norma de la particin tiende a
cero.

UNIVERSIDADNACIONALABIERTAYADISTANCIA UNAD
ESCUELADECIENCIASBSICAS,TECNOLOGAEINGENIERA
CONTENIDODIDCTICODELCURSO:100411ClculoIntegral

DEFINICIN:
Una funcin f(x) cuya primera derivada es continua, se denomina SUAVE y su
grfica es una curva suave.

Para una funcin f(x) suave, por el teorema del valor medio, existe un punto
( ) ) ( ,
k k
c f c de la curva
PQ

donde la tangente es paralela a dicho segmento.









Fig. No. 24 Demostracin longitud de curva.
Podemos inferir que:
k
k
x
y
x f

= ) ( '

Luego:
k k
x x f y = ) ( '


Si reemplazamos en la frmula que tenemos para la longitud L, obtenemos:

( ) ( )

=
+ =
n
k
k k k
x c f x L
1
2 2
) ( '

UNIVERSIDADNACIONALABIERTAYADISTANCIA UNAD
ESCUELADECIENCIASBSICAS,TECNOLOGAEINGENIERA
CONTENIDODIDCTICODELCURSO:100411ClculoIntegral

Reorganizando el radical:
( )
k
n
k
k
x c f L + =

=1
2
) ( ' 1
Suma de Riemman.
DEFINICIN:
Sea f(x) una funcin suave en [a, b], la longitud de la curva y = f(x) desde a hasta
b equivale a:

( ) [ ] dx x f L
b
a

+ =
2
1

Ejemplo 1:
Hallar la longitud de la curva y = x
2
en el intervalo [-1, 1].
Solucin:
La ecuacin para hallar la longitud de la curva, muestra que se debe hallar la
derivada de la funcin y luego elevarla al cuadrado, realicemos esto:
f(x) = x
2
luego: f(x) = 2x y (f(x))
2
= (2x)
2
, con estos argumentos podemos aplicar
la ecuacin para hallar la longitud. De la curva.
[ ] ( ) dx x dx x f L
b
a


+ = + =
2
1
2 2
2 1 ) ( ' 1
Para resolver esta integral
podemos aplicar la frmula siguiente:
[ ] c x a x Ln
a
x a
x
dx x a + + + + + = +

2 2
2
2 2 2 2
2 2
Reemplazando:
( ) ( ) ( )
2
1
2 2
2
1
2
2 1 2
2
1
2 1
2
2
2 1

+ + + + = + =

x x Ln x
x
dx x L

UNIVERSIDADNACIONALABIERTAYADISTANCIA UNAD
ESCUELADECIENCIASBSICAS,TECNOLOGAEINGENIERA
CONTENIDODIDCTICODELCURSO:100411ClculoIntegral

Simplificando y evaluando:

( ) ( ) ( ) ( ) ( )
2
1
2 2
2
1
2 2
2 1
2
1
2 1 2 1 2
2
1
2 1
2
2

+ + + =

+ + + + = x Ln x x x x Ln x
x
L

( ) ( ) ( ) ( )

+ + +

+ + + =
2 2
2 * 1 1
2
1
2 * 1 1 1 16 1
2
1
16 1 2 Ln Ln L
Desarrollando:
094 . 11
2
) 5 (
5
2
) 17 (
17 2 = + + =
Ln Ln
L


Ejemplo 2:
Hallar la longitud de la curva
3
) ( x x f = en el intervalo [1, 4].

Solucin:
Como en el caso anterior, veamos:
2
1
3
2
3
) ( ' ) ( X x f x x f = = Ahora aplicamos la frmula de longitud.
[ ] dx x dx x dx x f L
b
a

+ =

+ = + =
4
1
4
1
2
2
1
2
4
9
1
2
3
1 ) ( ' 1

Hacemos cambio de variable:
dx du x u
4
9
4
9
1 = + = Despejamos dx = (4/9) du, luego reemplazamos:

UNIVERSIDADNACIONALABIERTAYADISTANCIA UNAD
ESCUELADECIENCIASBSICAS,TECNOLOGAEINGENIERA
CONTENIDODIDCTICODELCURSO:100411ClculoIntegral

= = + =

2
3
4
1
3
2
9
4
9
4
4
9
1 u du u dx x L

Como x u
4
9
1+ = , lo sustituimos para que la solucin quede en funcin de x, como
se propone originalmente.
4
1
2
3
2
3
4
1
4
9
1
27
8
3
2
9
4
4
9
1

+ =

= + =

x u dx x L
Evaluando:
2
3
2
3
4
1
2
3
1 *
4
9
1
27
8
4 *
4
9
1
27
8
4
9
1
27
8

+ =

+ = x L

( )
2
3
2
3
2
3
2
3
4
13
27
8
10
27
8
1 *
4
9
1
27
8
4 *
4
9
1
27
8

+ = L

( ) 634 . 7 7360 . 1 3697 . 9
4
13
27
8
10
27
8
2
3
2
3
= =

= L








UNIVERSIDADNACIONALABIERTAYADISTANCIA UNAD
ESCUELADECIENCIASBSICAS,TECNOLOGAEINGENIERA
CONTENIDODIDCTICODELCURSO:100411ClculoIntegral

Leccin 35: Longitud de un arco en forma para mtrica.

) (t f x =

En este aparte se analizar la longitud de curvas suaves, donde las funciones
estn dadas en forma paramtrica. Las funciones paramtricas, definidas en x y
y dependen de un parmetro t, segn:
) (t f x =

) (t f y =

Para
b t a

Al igual que en el caso de la longitud de curvas suaves, la idea es aproximar la
curva por medio de un segmento formado por una trayectoria polinomial.








Fig. No. 25 Longitud de curva paramtrica.

i
s Corresponde al segmento de longitud de la curva.
i
w Es la proyeccin de la longitud de la curva en un tringulo rectngulo, del
cual es la hipotenusa. La longitud de
i
w se obtiene de la siguiente manera:

UNIVERSIDADNACIONALABIERTAYADISTANCIA UNAD
ESCUELADECIENCIASBSICAS,TECNOLOGAEINGENIERA
CONTENIDODIDCTICODELCURSO:100411ClculoIntegral

( ) ( )
2 2
i i i
y x w + =
Debemos definir
i
x y
i
y , los cuales son equivalentes a:
) ( ) (
1
=
i i i
t f t f x
y
) ( ) (
1
=
i i i
t g t g y
Por el teorema del valor
medio, sabemos que existen los puntos c
i
y k
i
que pertenecen al intervalo ( t
i-1
, t
i
)
tal que:
i i i i
t c f t f t f =

) ( ' ) ( ) (
1
y
i i i i
t k g t g t g =

) ( ' ) ( ) (
1
Luego:
( ) ( )
2 2
) ( ' ) ( '
i i i i i
t k g t c f w + =
La longitud total de la trayectoria
Polinomial ser:
( ) ( )
i
n
i
i i
n
i
i
t k g c f w + =

= = 1
2 2
1
) ( ' ) ( '
Si observamos bien, deducimos que
corresponde la suma de Riemman.
Por consiguiente, la longitud del arco establecido, ser el lmite de la ecuacin
anterior, cuando la norma de la particin tiende a cero, entonces:




Dicho de otra manera:




As se puede calcular la longitud de una curva con ecuaciones paramtricas.
[ ] [ ] dt t g t f L
b
a

+ =
2 2
) ( ' ) ( '
dt
dt
dy
dt
dx
L
b
a

=
2 2

UNIVERSIDADNACIONALABIERTAYADISTANCIA UNAD
ESCUELADECIENCIASBSICAS,TECNOLOGAEINGENIERA
CONTENIDODIDCTICODELCURSO:100411ClculoIntegral

Ejemplo 1:
Encontrar el permetro del crculo 16
2 2
= + y x , para 2 0 t . La forma
paramtrica de la ecuacin es: y = 4sen (t) y x = 4cos (t)
Solucin:
Como se sabe como se comporta x e y respecto a t, derivamos las dos variables
respecto al parmetro.
) ( 4 ) cos( 4 t sen
dt
dx
t x = =
y
) cos( 4 ) ( 4 t
dx
dy
t sen y = =

Por la frmula de longitud:
( ) ( )

+ =

=
2
0
2 2
2 2
) cos( 4 ) ( 4 dt t t sen dt
dt
dy
dt
dx
L
b
a

( ) ( )

+ = + =
2
0
2
0
2 2 2 2
) ( cos ) ( ( 16 ) cos( 4 ) ( 4 dt t t sen dt t t sen L
( ) dt t t sen dt t t sen L

+ = + =
2
0
2 2
2
0
2 2
) ( cos ) ( 4 ) ( cos ) ( 16


8 ) 0 2 ( 4 4 4 ) ( cos ) ( 4
2
0
2
0
2
0
2 2
= = = = + =

t dt dt t t sen L

El permetro del crculo propuesto tiene como longitud 8.
Ejemplo 2:
Calcular la longitud de la curva, cuya ecuacin paramtrica est dada por:
3
3
1
t x =
y
2
2
1
t y = para 1 0 t

UNIVERSIDADNACIONALABIERTAYADISTANCIA UNAD
ESCUELADECIENCIASBSICAS,TECNOLOGAEINGENIERA
CONTENIDODIDCTICODELCURSO:100411ClculoIntegral

Solucin:
Primero calculamos las derivadas de las funciones x e y.
2 3
3
1
t
dt
dx
t x = = y t
dt
dy
t y = =
2
2
1
, ahora aplicamos la ecuacin para
hallar la longitud.
( ) ( ) dt t t dt
dt
dy
dt
dx
L
b
a

+ =

=
1
0
2
2
2
2 2

( ) ( ) ( ) dt t t dt t t dt t t L

+ = + = + =
1
0
2
1
0
2 4
1
0
2
2
2
1 )

Por cambio de variable: tdt du t u 2 1
2
= + = despejamos
2
du
tdt = Luego:
( )
2
3
2
1
1
0
2
3
2
*
2
1
2
1
2
1 u du u
du
u dt t t L = = = + =


No utilizamos los lmites, ya que estamos trabajando con la variable u, cuando
sustituyamos de nuevo u por x s hacemos la evaluacin de los lmites.
Reemplazando obtenemos:
( ) ( ) ( ) 1 2 2
3
1
1 8
3
1
1
6
2
3
2
*
2
1
1
0
3
2
2
3
= = + = = t u L
La longitud de la curva paramtrica es de
( ) 1 2 2
3
1


EJERCICIOS:
Solucionar los siguientes ejercicios.
1. Por integracin hallar la longitud de la curva 3 2 ) ( + = x x f en el intervalo [1, 3]
Rta: 5 2

UNIVERSIDADNACIONALABIERTAYADISTANCIA UNAD
ESCUELADECIENCIASBSICAS,TECNOLOGAEINGENIERA
CONTENIDODIDCTICODELCURSO:100411ClculoIntegral

2. Hallar la longitud de la curva ( )
3
3 2
4 ) ( x x g = Entre 8 1 x Rta: 9
3. Cual ser la longitud de la curva
2
2 ) ( x x x h = en el intervalo 2 0 x
Rta: dx x x

+
2
0
2
5 8 4 Por integracin numrica se obtiene: 2,9578
4. Cual ser la longitud de la curva ) (
2
1
) (
2
x Ln x x p = en el intervalo [2, 4]
Rta: ) 2 (
4
1
6 Ln + 6,1732

5. Los hilos de un tendido elctrico suspendidos entre dos torres tiene la forma de
una catenaria, cuya ecuacin es:

=
20
cosh 20 ) (
x
x f , x e y se miden en metros,
Cual ser la longitud de la cuerda que descansa
entre los dos postes.
2
) cosh(
x x
e e
x

+
=

Rta: ( ) metros senh senh 47 ) 1 ( ) 1 ( 20

6. Hallar la longitud de la curva, cuya ecuacin paramtrica est dada por:
) ( 4 t sen x =
y 5 ) cos( 4 = t y , en el intervalo [0, ] Rta:
4


7. Calcular la longitud de la hipocicloide de cuatro vrtices, que tiene como
ecuaciones paramtricas: ) (
3
t asen x = y ) ( cos
3
t a y = donde 2 0 t
Rta: 6a

UNIVERSIDADNACIONALABIERTAYADISTANCIA UNAD
ESCUELADECIENCIASBSICAS,TECNOLOGAEINGENIERA
CONTENIDODIDCTICODELCURSO:100411ClculoIntegral

CAPITULO 8: Volumen de superficie de revolucin.

Introduccin

( )dx x A V
b
a

=

Haciendo un seguimiento a las secciones anteriores, vemos que por medio de
integrales podemos hallar reas bajo la curva, longitud de curvas y rea de
superficies de una curva al rotar. Ahora nos preguntaremos Que ocurre con el
volumen de figuras engendradas al girar una curva? La respuesta est dada
tambin por medio de integrales.
Para hallar el volumen de un slido de revolucin, hay varias tcnicas, las cuales
analizaremos en seguida, solo es pertinente resaltar que sea el camino que se
tome, las demostraciones siguen la lnea de las sumas de Riemman.

Leccin 36: Volumen de slidos de revolucin: mtodo de arandelas.
Imaginmonos un tubo macizo, que al hacerle una rebanada por el centro, se nos
forma un tubo hueco, al particionarlo obtenemos arandelas.
Toda arandela tiene un rea y un volumen, situacin que vamos a analizar.





Fig. No. 26 Arandelas
V = Volumen de la arandela
A = rea de la base

UNIVERSIDADNACIONALABIERTAYADISTANCIA UNAD
ESCUELADECIENCIASBSICAS,TECNOLOGAEINGENIERA
CONTENIDODIDCTICODELCURSO:100411ClculoIntegral

h = Grosor
h A V * = Pero:
( )
2 2
r R A = Luego:
( ) h r R V *
2 2
=
Utilizando un procedimiento similar al caso de las rebanadas o discos, podemos
obtener el volumen del slido formado por las arandelas.
( ) ( ) ( ) ( ) [ ]dx x r x R V
b
a
2 2
=


Podemos ver que R y r son funciones de x.
Ejemplo 1:
Dadas las curvas
2
) ( x x f = y x x g 2 ) ( = , ubicadas en el primer cuadrante. Hallar
el volumen del slido generado al girar alrededor del eje y acotado por las curvas
dadas.
Solucin:







Fig. No. 27 Solucin volumen

UNIVERSIDADNACIONALABIERTAYADISTANCIA UNAD
ESCUELADECIENCIASBSICAS,TECNOLOGAEINGENIERA
CONTENIDODIDCTICODELCURSO:100411ClculoIntegral

Como las funciones giran alrededor del eje Y, se debe expresar las funciones as:
x = f(y), como vemos en la grfica.
Con estos argumentos podemos hallar el volumen.

Reemplazando:

Integrando:
( ) ( )

3
8
3
16
8 64
12
16
2 12 2
4
0
3
4
0
2
= = = = y y V
El volumen del slido generado es de
3
8
unidades cbicas.
NOTA: Si observamos detenidamente, para este tipo de problemas, lo esencial es
identificar las funciones R(x) y r(x); adems, sobre cual eje gira. Si es alrededor
del eje x se expresa las funciones de la forma R(x) y r(x), pero si es alrededor del
eje y se expresa como R(y) y r(y). Importante hacer la grfica explicita de la
situacin.
Ejemplo 2:
Hallar el volumen del slido generado al rotar sobre el eje x las curvas dadas por:
3 + = x y
y
1 ) (
2
+ = x x g

Solucin:
Primero hallemos los lmites de integracin, esto ocurre cuando: 1 3
2
+ = + x x , si
despejamos x obtenemos: x = -2 y x = 1. (Por favor corroborar estos lmites)
Entonces: 3 ) ( + = x x R y 1 ) (
2
+ = x x r
( ) ( ) [ ]dx x x V

+ + =
1
2
2
2
2
1 3
Desarrollando:
( ) ( ) [ ]dy y r y R V
b
a

=
2 2
) ( ) (
( ) dy y ydy dy
y
y V

=

=
4
0
2
4
0
4
0
2
2
4
1
2

UNIVERSIDADNACIONALABIERTAYADISTANCIA UNAD
ESCUELADECIENCIASBSICAS,TECNOLOGAEINGENIERA
CONTENIDODIDCTICODELCURSO:100411ClculoIntegral

( ) ( ) [ ]


= + + + =
1
2
4 2
1
2
2 4 2
) 6 8 ( 1 2 9 6 dx x x x dx x x x x V


5
117
5
1
3
1
3 8
1
2
5 3 2
=

x x x x V
El volumen del slido generado es de
5
117
unidades cbicas.
Ejemplo 3:
Hallar el volumen del slido generado al rotar la curva 4
2 2
= + y x alrededor
del eje x = -1.
Solucin:








Fig. No. 28 Solucin ejemplo 3

Observando las figuras, podemos ver que la curva gira alrededor del eje x = -1. lo
que origina un cilndrico hueco de radio 1, ahora:
2
4 1 ) ( y y R + = y 1 ) ( = y r , luego:

UNIVERSIDADNACIONALABIERTAYADISTANCIA UNAD
ESCUELADECIENCIASBSICAS,TECNOLOGAEINGENIERA
CONTENIDODIDCTICODELCURSO:100411ClculoIntegral

( ) ( )

+ =
2
2
2
2
2
1 4 1 dy y V Luego: ( ) [ ]

+ + =
2
2
2 2
1 4 4 2 1 dy y y V
( ) [ ] ( )

+ = + =

2
0
2 2
2
2
2 2
4 4 2 2 4 4 2 dy y y dy y y V Por simetra
( )

+ = + =
2
0
2
0
2
2
0
2
2
0
2 2
2 8 4 4 4 4 2 2 dy y dy dy y dy y y V Integrando:
2
0
3
2
0
2
0
1
2
2
2
0
2
0
2
2
0
2
3
2
8 )
2
(
2
4
2
4 2 8 4 4 y y
y
Sen
y
y
y
dy y dy dy y V +

+ = + =



Evaluando:
2
0
3
2
0
2
0
1
2
2
2
0
2
0
2
2
0
2
3
2
8 )
2
(
2
4
2
4 2 8 4 4 y y
y
Sen
y
y
y
dy y dy dy y V +

+ = + =



) 8 (
3
2
16 0
2
2 4
3
2
8 )
2
(
2
4
2
4
2
0
3
2
0
2
0
1
2
2

= +

+ =

y y
y
Sen
y
y
y
V

3
16
16 4 ) 8 (
3
2
16 0
2
2 4
2
+ = +

= V
Operando:

3
32
4
2
+ = V
Unidades Cbicas.
EJERCICIOS:
1. Sea la regin R la cual est delimitada por las curvas y = f(x) y y = g(x), donde
f(x) > g(x), si R se hace girar alrededor del eje x entre los valores a y b. Cual sera
el volumen del slido generado.


UNIVERSIDADNACIONALABIERTAYADISTANCIA UNAD
ESCUELADECIENCIASBSICAS,TECNOLOGAEINGENIERA
CONTENIDODIDCTICODELCURSO:100411ClculoIntegral

2. Cmo se hallara el volumen del slido generado, cuando se hace girar la curva
3
1
x
y = alrededor del eje y = -1, entre x = 1 y x = 3 Rta:
dx
x x
V

+ =
3
1
3 6
2 1

3. Calcular el volumen del slido generado al girar alrededor del eje x las
curvas: 1 = y
y ) cos(x y = entre -/2 y /2 Rta: 2
2

4. Hallar el volumen del slido generado al girar alrededor del eje y las curvas
1 = x y
) tan(y x = en:
4
0

y Rta: ( ) 4
4
1
2

5. Cual ser el volumen del slido generado por las curvas x y = y 1 = y , cuando
giran alrededor del eje x. Rta:
3
2

6. Calcular el volumen del slido generado al girar alrededor del eje y las curvas

2
x y = , la recta 0 = x y la recta 2 = x Rta: 8
Todas las respuestas, estn dadas en unidades cbicas.






UNIVERSIDADNACIONALABIERTAYADISTANCIA UNAD
ESCUELADECIENCIASBSICAS,TECNOLOGAEINGENIERA
CONTENIDODIDCTICODELCURSO:100411ClculoIntegral

Leccin 37: Volumen de slidos de revolucin: mtodo de casquetes
cilndricos.

( )h r R V
2 2
=

En muchos problemas de diferentes reas del saber, el mtodo del casquete
cilndrico es muy adecuado para la solucin de la situacin presentada. Un
cascarn de forma cilndrica, es un slido acotado por dos cilindros circulares
rectos, de forma concntrica, con radio interior r y radio exterior R; adems, una
altura h








Fig. No. 29 Casquetes.

El volumen ser: h A V
b
* = Donde A
b
es el rea de la base y h la altura. Pero
el rea de la base ser:
2 2
r R A
b
= , luego:
( ) h r R V *
2 2
= Desarrollando:
( )( ) h r R r R V * + =
Para obtener la ecuacin que permite hallar el volumen, debemos hacer una
transformacin:
Multiplicamos y dividimos por 2 la ltima ecuacin, luego:

UNIVERSIDADNACIONALABIERTAYADISTANCIA UNAD
ESCUELADECIENCIASBSICAS,TECNOLOGAEINGENIERA
CONTENIDODIDCTICODELCURSO:100411ClculoIntegral

( ) h r R
r R
V *
2
2

+
=
Ahora, definimos radio promedio como:
2
r R
R
+
= y cambio del radio como:
r R r = . Por consiguiente: r Rh V = 2
Para hallar el volumen del slido de revolucin al girar la regin acotada por la
curva y = f(x) al rededor de un eje de coordenadas, hacemos la particin, llevando
la norma de sta a cero y, sumamos las fracciones formadas, de esta manera se
logra obtener el volumen del slido.

Fig. No. 30 Desarrollo slidos de revolucin.
Segn la primera grfica:
x x xf V ) ( 2
Si llevamos la particin a cero y
sumamos todas las partes, obtenemos:
La obtencin de la ecuacin, ha seguido los mismos principios que hemos venido
utilizando, o sea por medio de las sumas de Riemman.
( )dx x xf V
b
a

= 2



UNIVERSIDADNACIONALABIERTAYADISTANCIA UNAD
ESCUELADECIENCIASBSICAS,TECNOLOGAEINGENIERA
CONTENIDODIDCTICODELCURSO:100411ClculoIntegral

Ejemplo 1:
Al hacer girar la curva x y = alrededor del eje y entre las rectas x = 0 y x = 4, se
genera un slido de revolucin, ilustrar el caso y hallar el volumen del slido
generado.
Solucin:

Fig. No. 31 Solucin ejemplo 1

Como:

=
b
a
dx x xf V ) ( 2 reemplazamos en los datos que tenemos:
dx x dx x x dx x x V

= = =
4
0
2
3
4
0
2
1
4
0
2 * 2 * 2 Aqu ya podemos integrar:
( ) [ ]
5
128
0 4
5
4
5
2
2
2
5
4
0
2
5
= = = x V
Unidades cbicas.
Ejemplo 2:
Dada la recta x
h
r
y

= con r > 0 y h > 0, el eje x y la recta . h x = La recta y se


hace girar alrededor del eje x. Encontrar el volumen del slido generado.

UNIVERSIDADNACIONALABIERTAYADISTANCIA UNAD
ESCUELADECIENCIASBSICAS,TECNOLOGAEINGENIERA
CONTENIDODIDCTICODELCURSO:100411ClculoIntegral

Resolverlo por:
a- Mtodo de arandelas
b- Mtodo de casquetes

Solucin:
a- Por el mtodo de arandelas.








Fig. No. 32 Solucin ejemplo 2

( ) dx x
h
r
V dx x r x R V
h b
a

= =
0
2
2 2
) ( ) (
Desarrollando el cuadrado e
integrando:
h
h h h
x
h
r
dx x
h
r
dx x
h
r
dx x
h
r
V
0
3
2
2
0
2
2
2
2
0
2
2
0
2
3
1

= = =

=


Evaluando:
h r h
h
r
x
h
r
V
h
2 3
2
2
0
3
2
2
3
1
3
1
3
1
=

=

h r V
2
3
1
=
Corresponde al volumen de un cono circular recto.

UNIVERSIDADNACIONALABIERTAYADISTANCIA UNAD
ESCUELADECIENCIASBSICAS,TECNOLOGAEINGENIERA
CONTENIDODIDCTICODELCURSO:100411ClculoIntegral

b- Por el mtodo de casquete:





Fig. No. 33 Demostracin casquetes.

dy y
r
y h dy y
r
h
h y dy y yf V
b
a
r r

= =
0
2
0
1
2 2 ) ( 2

Desarrollando la integral.

r
r r
h y
r
y
h dy y
r
y V
r
r
3 2
2
3
1
2
2
1
3 2
0
3
2
0
2

Simplificando:
h r V
2
3
1
=
Volumen de un cono circular recto.
Como podemos observar los dos mtodos conllevan al mismo resultado.

Ejemplo 3:

Hallar el volumen del slido generado al girar alrededor del eje y, la regin por
encima de la parbola
2
) ( x x f = y por debajo de la curva
2
2 ) ( x x g = .
Solucin:
Por el tipo de grafica, las rebanadas verticales nos llevan a una buena solucin.

UNIVERSIDADNACIONALABIERTAYADISTANCIA UNAD
ESCUELADECIENCIASBSICAS,TECNOLOGAEINGENIERA
CONTENIDODIDCTICODELCURSO:100411ClculoIntegral


2 2
2 x x h =

Ahora:

=
b
a
dx x xf V ) ( 2

Fig. No. 34 Rebanadas.

( ) ( ) ( )

= = =
1
0
1
0
3 2
1
0
2
4 1 4 2 2 2 dx x x dx x x dx x x V
( )

= =

4
1
4
4
1
2
1
4
4
1
2
1
4 4
1
0
4 2
1
0
3
x x dx x x V
= V Unidades cbicas.

Ejercicios:
1. Hallar el volumen del slido generado por los planos perpendiculares a la
recta x = -1 y x = 1, las secciones transversales perpendiculares al eje entre
estos planos son cuadrados verticales cuyas bases van del semicrculo
2
1 x y = al semicrculo
2
1 x y = .
Rta: 16/3
2. Hallar el volumen del slido generado entre los planos perpendiculares al eje
y por y = 0 y, y = 2. Las secciones transversales perpendiculares al eje y son
discos circulares cuyos dimetros van desde el eje y hasta la parbola
2
5y x =
Rta: 8
3. Encontrar el volumen del slido generado al girar alrededor del eje x la curva
x
y
1
= , con x = 2, x = 4 y el eje y. Rta:
4

UNIVERSIDADNACIONALABIERTAYADISTANCIA UNAD
ESCUELADECIENCIASBSICAS,TECNOLOGAEINGENIERA
CONTENIDODIDCTICODELCURSO:100411ClculoIntegral

4. Encontrar el volumen del slido generado al girar alrededor del eje y la curva
1
4
1
3
+ = x y , y = 1 x y x = 1. Rta:
30
23

5. Se perfora un agujero redondo de radio r que para por el centro de una esfera
slida de radio R, (R > r) encontrar el volumen del slido producido Rta:
( )
3
2 2
3
4
r R


Leccin 38: Volumen de slidos de revolucin: mtodo de rebanadas o
discos.

( )

=
b
a
dx x A V

Para hallar el volumen del slido descrito en la grfica, en cada punto x del
intervalo definido, la seccin transversal del slido corresponde a la regin R(x),
cuya rea es A(x). Luego A es funcin de x de valor real. Las capas o rebanadas
formadas se suman para formar el volumen del slido en el intervalo definido.







Fig. No. 35 Discos.
i i i
x c A V = ) (

UNIVERSIDADNACIONALABIERTAYADISTANCIA UNAD
ESCUELADECIENCIASBSICAS,TECNOLOGAEINGENIERA
CONTENIDODIDCTICODELCURSO:100411ClculoIntegral

Donde
i
c es el punto contenido en el intervalo [ ]
i i
x x ,
1
. El volumen del slido
ser aproximadamente la suma de Riemman, cuando la particin se hace muy
pequea.

=

n
i
i i
x c A V
1
) (
Sabiendo que el rea se debe obtener segn el tipo de figura
que
se obtiene, el volumen ser de la forma:

A(x) es el rea de la figura obtenida.

NOTA: Es pertinente tener presente que para resolver problemas de este tipo, se
requieren buenos principios de geometra plana y espacial, por lo cual se
recomienda en caso de recordar algo al respecto, consultar el mdulo de
Matemticas Bsicas de la UNAD.
Ejemplo 1:
Una pirmide de 3 m. de altura tiene base cuadrada de 3 m. de lado, hallar el
volumen de la pirmide.
Solucin:
El rea de la seccin transversal es: A(x) = x
2

ahora:
( ) ( ) 27
3
1
1 3
3
1
3
1
3 3
3
0
3
3
0
2
= = = =

x dx x V
Resolviendo: Fig. No. 36 Solucin problema 1
9
3
0
2
= =

dx x V
Unidades cbicas.

=
b
a
dx x A V ) (

UNIVERSIDADNACIONALABIERTAYADISTANCIA UNAD
ESCUELADECIENCIASBSICAS,TECNOLOGAEINGENIERA
CONTENIDODIDCTICODELCURSO:100411ClculoIntegral

El ejercicio fue relativamente fcil, ya que la figura el muy conocida, pero no ocurre
siempre as, veamos otros ejemplos.

Ejemplo 2:
Hallar el volumen del slido de revolucin al hacer girar la regin R(x) alrededor
del eje X y acotada por la curva x x f = ) ( en el intervalo 3 0 x .
Solucin:





Fig. No. 37 solucin problema 2

El rea para un crculo es:
2
R A = , como el volumen es rea de la base por la
altura, entonces:
( ) x x f V =
2
) ( Luego:


= = =
3
0
3
0
3
0
2
) ( xdx xdx dx x V

Integrando obtenemos:
( )
2
9
0 3
2
1
2
1
2 2
3
0
2
3
0
= = = =

x xdx V

UNIVERSIDADNACIONALABIERTAYADISTANCIA UNAD
ESCUELADECIENCIASBSICAS,TECNOLOGAEINGENIERA
CONTENIDODIDCTICODELCURSO:100411ClculoIntegral

Luego:

2
9
= V
unidades cbicas.
Ejemplo 3:
Encontrar el volumen del slido generado al rotar sobre el eje Y la regin acotada
por la curva y = x
2
en el intervalo [0, 4].

Solucin:
Como y x x y = =
2

Ya que necesitamos rotarlo alrededor de Y,
como

2
R A = , Siendo y R = , entonces:
( ) y y V =
2
Por la suma de Riemman, obtenemos:

( )
2 2
4
0
4
0
2
4
0
0 4
2
1
2
1
= = = =

y ydy ydy V


Finalmente:

( ) 8 0 4
2
1
2 2
= = V
Unidades cbicas.
Ejemplo 4:
Dada la funcin
2
2
1
2 x y = . Hallar el volumen del slido generado por la curva
alrededor del eje y para 2 0 x .

UNIVERSIDADNACIONALABIERTAYADISTANCIA UNAD
ESCUELADECIENCIASBSICAS,TECNOLOGAEINGENIERA
CONTENIDODIDCTICODELCURSO:100411ClculoIntegral

Solucin:

Como el giro es alrededor del eje y, despejamos
x,

luego: y x x y 2 4
2
1
2
2
= = . En seguida

aplicamos la ecuacin para el volumen del slido alrededor del eje y, entonces:

= =
2
0
2
0
2 4 dy y ydy V

Desarrollando:
( ) 4 4 2 4
2
0
2
2
0
= = =

y y dy y V
Unidades cbicas.
Ejercicios:
1. Calcular el volumen del slido generado al rotar la regin acotada por x y = 2 ,
en x = 0 y y = 0, alrededor del eje x. Rta:
3
8


2. Calcular el volumen del slido generado al rotar la regin limitada por la curva
x y = para y = 2 y x = 0, alrededor del eje y. Rta:
5
32


3. Calcular el volumen del slido generado al rotar la regin limitada por la curva
x y = para y = 2 y x = 0, alrededor del eje x = 4. Rta:
15
224

UNIVERSIDADNACIONALABIERTAYADISTANCIA UNAD
ESCUELADECIENCIASBSICAS,TECNOLOGAEINGENIERA
CONTENIDODIDCTICODELCURSO:100411ClculoIntegral

4. Calcular el volumen del slido generado al rotar la regin limitada por la curva
3
x y = para y = 0 y x = 1, alrededor de x = 1 Rta:
10
1
.

5. Calcular el volumen del slido generado al rotar la regin limitada por la curva
1
2
+ = x y para [0, 2], alrededor del eje x. Rta:
15
206

6. Calcular el volumen del slido generado al rotar la regin limitada por la curva
2
4 x y = para [0, 2], alrededor del eje x. Rta:
15
256

7. Calcular el volumen del slido generado al rotar la regin limitada por la curva
2
4 x y = para [0, 2], alrededor del eje y. Rta: 8

8. Una pirmide se levanta 500 metros sobre una base cuadrada de 750 metros
de lado. Cual ser el volumen de la pirmide. Rta: 93.750
metros cbicos










UNIVERSIDADNACIONALABIERTAYADISTANCIA UNAD
ESCUELADECIENCIASBSICAS,TECNOLOGAEINGENIERA
CONTENIDODIDCTICODELCURSO:100411ClculoIntegral

Leccin 39: Momentos y centros de masa
Recordando los principios de dinmica y mecnica, sabemos que el momentum es
el producto de la masa y la distancia respecto a un punto de equilibrio.





Fig. No. 38 equilibrio

Momentum = x
1
*m
1
+ x
2
*m
2


El tringulo nos indica el punto de equilibrio.

Para un sistema de masas = m
1
, m
2
, m
3
, m

ubicados en los puntos x
1
, x
2
, x
3
,
, x

respectivamente a lo largo del eje x, el momentum total M, ser la suma


de los momentum individuales.

=
=
n
i
i i
m x M
1
* Cuando M = 0, se presenta equilibrio si el punto de equilibrio esta
en el
origen. Generalmente esto no ocurre, la situacin es cmo hallar el punto para
que un sistema de masas este en equilibrio. Si llamamos C
e
el punto donde un
sistema de masas puede estar en equilibrio, entonces:
( ) ( ) ( ) ( ) 0 ...
3 3 2 2 1 1
= + + + +
n e n e e e
m C x m C x m C x m C x Operando:
e n e e e n n
C m C x C m C m m x m x m x m x + + + + = + + + + ... ...
3 2 1 3 3 2 2 1 1

Despejando C
e
obtenemos lo que se conoce como el centro de masa:

UNIVERSIDADNACIONALABIERTAYADISTANCIA UNAD
ESCUELADECIENCIASBSICAS,TECNOLOGAEINGENIERA
CONTENIDODIDCTICODELCURSO:100411ClculoIntegral

=
=
=
n
i
i
n
i
i i
e
m
m x
C
1
1

Fsicamente el centro de masa es el punto donde concentramos toda la masa del
sistema. Si deseamos distribuir dicha masa a lo largo de una recta de alambre con
densidad variable, llegamos al siguiente planteamiento:

x x m = ) ( . Siendo m la masa, ) (x densidad en el punto x y x ubicacin
de la masa respecto al punto de equilibrio. Luego:


Corresponde a la masa en un punto dado de la recta

Por otro lado: x x x M = ) ( Por medio de la teora de integrales llegamos a:




Con todo lo anterior, podemos hallar el centro de masa a lo largo de una varilla
con densidad variable.




=
b
a
dx x x M ) (

=
b
a
dx x m ) (

=
b
a
b
a
e
dx x
dx x x
C
) (
) (

UNIVERSIDADNACIONALABIERTAYADISTANCIA UNAD
ESCUELADECIENCIASBSICAS,TECNOLOGAEINGENIERA
CONTENIDODIDCTICODELCURSO:100411ClculoIntegral

Dicho de otra manera:


La parte fundamental para resolver problemas de este tipo, es identificar
claramente la funcin densidad en el punto establecido.
Ejemplo 1:

Una varilla de 20 cm. De longitud presenta una densidad de 1 2 ) (
2
= x x , Hallar
el centro de masa.
Solucin:

Aplicando la ecuacin:

Reemplazando trminos:


Integramos cada parte:

800 . 79
2
400
2
000 . 160
2
1
2
1
) 2 (
20
0
2 4
20
0
3
= = =

x x dx x x

3
940 . 15
20
3
000 . 16
3
2
) 1 2 (
20
0
3
20
0
2
= = =

x x dx x
Agrupamos los dos resultados:
m
M
C
e
=

=
b
a
b
a
e
dx x
dx x x
C
) (
) (

=
20
0
2
20
0
2
) 1 2 (
) 1 2 (
dx x
dx x x
C
e

UNIVERSIDADNACIONALABIERTAYADISTANCIA UNAD
ESCUELADECIENCIASBSICAS,TECNOLOGAEINGENIERA
CONTENIDODIDCTICODELCURSO:100411ClculoIntegral


Centmetros
Ejemplo 2:
Encontrar el centro de masa de una lmina homognea, cuya forma es una regin
acotada por la curva y = sen(x) para 0 x .
Solucin:
Cuando la lmina es homognea, la densidad es constante, luego el centro de
masa de la regin, ser dada por la forma de la regin, para estos casos hablamos
de centrides.

La regin es simtrica en x = /2,
pero para ya el centro de masa ser
menor a 1/2, ya que la mayor
cantidad est por debajo de .

Ahora:

) ( * ) (
2
1
x f x f M
x
=
Fig. No. 38 Centro de masa
) (x f m
x
=
Aplicando la frmula obtenida:

= = =

0
0
2
0
0
0
) (
0
) (
) (
) (
2
1
) (
) ( * ) (
2
1
dx x sen
dx x sen
dx x sen
dx x sen x sen
dx m
dx M
Ce
x
x
y
Por identidades tenemos:

13 , 14
3
940 . 15
800 . 79
=
e
C

UNIVERSIDADNACIONALABIERTAYADISTANCIA UNAD
ESCUELADECIENCIASBSICAS,TECNOLOGAEINGENIERA
CONTENIDODIDCTICODELCURSO:100411ClculoIntegral

( )

0
0
) (
) (
) 2 cos( 1
2
1
dx x sen
dx x
Ce
y
Integremos por separado y al final agrupamos:
( )
2
) 2 (
4 2
) 2 (
2
1
2
1
) 2 cos( 1
2
1
0 0

= =

sen x sen x dx x
( ) 2 ) 0 cos( ) cos( ) cos( ) (
0
0
= = =

x dx x sen Agrupando:
4 2
2
) (

= =
y
Ce

Ejemplo 3:
Mostrar que el centroide de la regin acotada por las curvas
3
x y = y x y = , es:
7
3
) (
=
y
Ce
25
12
) (
=
x
Ce
Solucin:



Fig. No. 39 Centroide.
( )

=
1
0
3
1
0
3
) (
) ( dx x x
dx x x x
Ce
x

UNIVERSIDADNACIONALABIERTAYADISTANCIA UNAD
ESCUELADECIENCIASBSICAS,TECNOLOGAEINGENIERA
CONTENIDODIDCTICODELCURSO:100411ClculoIntegral

1
0
4
2
3
1
0
5
2
5
) (
4
1
3
2
5
1
5
2


=
x x
x x
Ce
x


Desarrollando:
( )
( )
25
12
12
5
5
1
) 1 (
4
1
) 1 (
3
2
) 1 (
5
1
) 1 (
5
2
) (
= =

=
x
Ce


Referente al eje y tenemos:

( )( ) [ ]
( )

+
=
1
0
3
1
0
2 3 2
1
0
3
1
0
3 3
) (
) ( ) (
2
1
) (
2
1
dx x x
dx x x
dx x x
dx x x x x
Ce
y
Desarrollando:

( )
( )
7
3
12
5
18
5
4
1
3
2
7
1
2
1
2
1
2
1
1
0
4
2
3
1
0
7 2
1
0
3
1
0
6
) (
= =


x x
x x
dx x x
dx x x
Ce
y

EJERCICIOS:
1. Hallar el centro de masa de un objeto cuya funcin densidad es: 2
6
) ( + =
x
x
para
0 x 6 Rta: C
e
= 16/5

UNIVERSIDADNACIONALABIERTAYADISTANCIA UNAD
ESCUELADECIENCIASBSICAS,TECNOLOGAEINGENIERA
CONTENIDODIDCTICODELCURSO:100411ClculoIntegral

2. Calcular el centro de masa para un objeto que tiene como densidad:
4
4
) (
2
+ =
x
x
para el intervalo: -2 x 2. Rta: Ce = 0

3. Tres partculas tienen masas 8, 4 y 6, estn ubicadas a lo largo de una recta en
3, -2 y

3 respectivamente, cual ser el centro de masa? Rta: Ce = 17/9

4. Un alambre tiene forma semicircular con radio 10 cm y densidad de 12 gr/cm
3

Cul
ser el centro de masa del alambre? Rta: Ce = 20/











UNIVERSIDADNACIONALABIERTAYADISTANCIA UNAD
ESCUELADECIENCIASBSICAS,TECNOLOGAEINGENIERA
CONTENIDODIDCTICODELCURSO:100411ClculoIntegral

Leccin 40: Volumen

TEOREMA DE PAPPUS:
Pappus, un griego de Alejandra, en el siglo III propuso dos frmulas para
relacionar los centrides de superficies y con slidos de revolucin. Dichas
frmulas simplifican el procedimiento para este tipo de problemas.
Teorema Del Volumen:
Si una regin plana R se gira alrededor de una recta en el plano que no interfecta
el interior de la regin, entonces el volumen del slido que se genera es igual al
rea de la regin multiplicada por la distancia recorrida por el centroide de la
regin durante el giro. Si D es la distancia desde el eje de rotacin al centroide,
entonces:



Veamos la demostracin:







Fig. No. 40 teorema de Pappus
Sea L(y) = Longitud transversal de la seccin R, perpendicular a y. Como L(y) es
continua.

A D V * 2 =

UNIVERSIDADNACIONALABIERTAYADISTANCIA UNAD
ESCUELADECIENCIASBSICAS,TECNOLOGAEINGENIERA
CONTENIDODIDCTICODELCURSO:100411ClculoIntegral

=
2
1
) ( 2
y
y
dy y yL V

Para y
1
y y
2
dados.
La coordenada en y del centroide esta dado por :
A
dy y yL
y
y
y

=
2
1
) (
Luego:

=
2
1
) (
y
y
dy y yL y A
Reemplazando y A en la ecuacin de volumen, tenemos:
y A V 2 =
Pero D y = Por
consiguiente:

Ejemplo No 1:
La regin acotada por ) (x sen y = para x 0 , se hace girar alrededor de x.
Hallar el volumen por el teorema de Pappus.
Solucin:
Como
Hallamos el rea: ( ) 2 ) 0 cos( ) cos( ) cos( ) (
0
0
= = = =

x dx x sen A
El volumen del slido de revolucin ser:
[ ]
2
0 0 0
2
2
1
) 2 (
2
1
2
) 2 cos( 1
2
) (

= = =

x sen x dx x dx x sen V
Si aplicamos Pappus:
D D DA V 4 * 2 * 2 2 = = =

A D V * 2 =
A D V * 2 =

UNIVERSIDADNACIONALABIERTAYADISTANCIA UNAD
ESCUELADECIENCIASBSICAS,TECNOLOGAEINGENIERA
CONTENIDODIDCTICODELCURSO:100411ClculoIntegral

CAPITULO 9: En las ciencias.

Introduccin
Existen numerosas aplicaciones del clculo integral a las ciencias, aparte de los
temas que vamos a estudiar en este captulo, tambin existen aplicaciones en el
software computacional, como por ejemplo en el diseo de programas
graficadores y que solucionan integrales indefinidas y definidas como DERIVE,
MAPLE, SOLVED, ETC en los cuales se incluye la integracin simblica y el
diseo de graficas.

Leccin 41: Integrales en la fsica: trabajo y movimiento.
TRABAJO: En el curso de Fsica General, aprendimos que cuando un objeto se
mueve una distancia dada, se realiza un trabajo, pero para mover el objeto, se
requiere de una fuerza constante w = f*d*cos (), donde w = trabajo, d = distancia
y el ngulo entre el vector fuerza y el vector distancia.
La mayora de los fenmenos de la naturaleza, presentan una caracterstica la
cual consiste en que a medida que el objeto se mueve en una trayectoria, la
fuerza varia, lo que indica que la fuerza es funcin de la distancia. Sea F(x) la
fuerza a lo largo de la trayectoria x y sea [a, b] un intervalo donde x es continua.
La idea es hallar el trabajo realizado por la fuerza F(x) en dicho intervalo.
Particionamos el intervalo [a, b] en k subintervalos y sea el punto c
k
en cada
subintervalo [x
k-1
, x
k
]. Tomamos un [ ]
k k k
x x c ,
1
. El trabajo realizado a lo largo del
intervalo ser aproximadamente F(c
k
) multiplicado por x
k
, luego el trabajo total
ser:

Si la particin es grande, y su norma tiende a cero, podemos definir el trabajo W
realizado por una fuerza F(x) a lo largo del intervalo [a, b] de la siguiente manera:


=
=
n
k
k k
x c F W
1
) (

=
b
a
dx x F W ) (

UNIVERSIDADNACIONALABIERTAYADISTANCIA UNAD
ESCUELADECIENCIASBSICAS,TECNOLOGAEINGENIERA
CONTENIDODIDCTICODELCURSO:100411ClculoIntegral

La parte crucial para resolver problemas de este tipo es identificar claramente la
funcin fuerza.
Ejemplo 1:
Cual ser el trabajo realizado por una fuerza
2
2
) (
x
x F = a lo largo del intervalo [ ] 5 , 1
Solucin:
Como tenemos la funcin fuerza, podemos aplicar directamente la ecuacin del
trabajo.
5
8
5
4
2
1
1
5
1
2
2 2
5
1
5
1
2
=

= =

x
dx
x
W Julios
En este ejemplo la funcin fuerza est definida, pero en muchas ocasiones se
debe determinar la funcin fuerza a partir del anlisis del fenmeno presentado.
Ejercicios:
1. Un objeto se mueve a lo largo del eje x debido a una fuerza ( )
2
1 2 ) ( + = x x F en
Newton, cual ser el trabajo realizado si el objeto se desplaza de x = 1 metro a x =
3 metros. Rta: 158/3 Julios
2. Una partcula se mueve a lo largo del eje x debido a una fuerza 2 4 ) ( = x x F
dinas. Si 100 ergios es el trabajo realizado para mover la partcula desde el origen
hasta un punto x = c Hallar c si debe cumplir que c>0 Rta: C 7,588
3. Por la Ley de Gravitacin Universal de Newton, cuando dos partculas de masa
m
1
y m
2
se atraen mutuamente, la magnitud de la fuerza de atraccin es
directamente proporcional al producto de las masas e inversamente proporcional
al cuadrado de la distancia entre ellas.
2
2 1
*
x
m m
G F = Donde G es la constante
universal de la gravedad y x la distancia entre las masas. Si m
1
=2 Kg y est en el
origen, m
2
= 4 Kg Qu trabajo se realiza para mover m
2
de del primer metro a
quinto metro de distancia.
Rta: (32/4)G Julios.

UNIVERSIDADNACIONALABIERTAYADISTANCIA UNAD
ESCUELADECIENCIASBSICAS,TECNOLOGAEINGENIERA
CONTENIDODIDCTICODELCURSO:100411ClculoIntegral

LEY DE HOOKE:
Por teora de la Fsica general, se sabe la ley de Hooke, la cual establece que
para mantener un resorte estirado o comprimido x unidades de su longitud natural,
se requiere una fuerza kx x F = ) ( , donde k es la constante del resorte; adems, se
ha establecido que a mayor rigidez del resorte, mas alto es el valor de la
constante. El trabajo realizado para estirar o comprimir un resorte, se puede
calcular con la ecuacin definida para trabajo realizado por una fuerza variable.
Ejemplo 1:
Un resorte tiene una longitud de 2 metros, al aplicarle una fuerza de 35 Newton,
dicho resorte se estira hasta 3,5 metros.
Qu trabajo se requiere para que le resorte se estire 4 metros

Solucin:
Como

=
b
a
dx x F W ) ( pero F(x) = kx, luego debemos determinar el valor de la
constante, lo cual se puede hacer con los datos del problema.
Al aplicar 35 Newton, el resorte se estira de 2 a 3,5 Metros, entonces x = 1,5
metros. Entonces, por la ley de Hooke: 35 = k (1,5), despejamos k y obtenemos: k
= 23,33 Nw/m.
Ahora planteamos la funcin fuerza: x x F 33 , 23 ) ( = y as podemos hallar el trabajo.

( ) 64 , 186 0 4 665 , 11
2
33 , 23
33 , 23 33 , 23 ) (
2 2
4
0
4
0
2
4
0
= = = = = =

x xdx xdx dx x F W
b
a
Julios.
Los lmites de integracin se obtiene sabiendo que el resorte se estira 4 metros
desde su posicin original; es decir, x = 0.


UNIVERSIDADNACIONALABIERTAYADISTANCIA UNAD
ESCUELADECIENCIASBSICAS,TECNOLOGAEINGENIERA
CONTENIDODIDCTICODELCURSO:100411ClculoIntegral

Ejemplo 2:
Un resorte es tal que la fuerza requerida para mantenerlo estirado s centmetros
esta dado por F = 12s. Si la longitud del resorte es de 30 centmetros y se estira
hasta 45 centmetros, cual ser el trabajo realizado para estira el resorte.
Solucin:
Tenemos la funcin fuerza F = 12s, por otro lado el resorte se estira de 30 a 45
centmetros; es decir, 15 centmetros, luego:
( ) 1350 0 15 6
2
1
* 12 12
2 2
15
0
2
15
0
= = = =

s sds W ergios.
Ejercicios:
1. La fuerza que mantiene un resorte estirado x centmetros es F(x) = 12x dado en
dinas, qu trabajo se realiza para estirar dicho resorte 8 centmetros. Rta: 384
ergios

2. El motor de un automvil ejerce una fuerza ) 1 ( 800 ) ( x x x F = en la posicin x,
cual ser el trabajo realizado para x 0 , Rta: 703.983
pie-Lb

3. Una cuerda tiene 50 metros de longitud, que trabajo se hace para recogerla
completamente, si se encuentra completamente vertical Rta: 780 Julios






UNIVERSIDADNACIONALABIERTAYADISTANCIA UNAD
ESCUELADECIENCIASBSICAS,TECNOLOGAEINGENIERA
CONTENIDODIDCTICODELCURSO:100411ClculoIntegral

Movimiento: Cuando estudiamos las derivadas, veamos que a partir de la
funcin posicin obtenamos la funcin velocidad y aceleracin.
y = s ( t )
y = v(t)
y = a(t)
Ahora la idea es que a partir de la funcin aceleracin obtener la funcin velocidad
y luego la funcin posicin. As describir el movimiento del cuerpo.

dt
ds
v =

2
2
dt
s d
dt
dv
a = =
Unos ejemplos nos ayudan a aclarar estos conceptos.
Ejemplo No 1:
Desde una altura de 20 metros, un nadador se lanza con una velocidad de 10
m/seg. en direccin ascendente. Con qu velocidad toca el agua el nadador?
Solucin:
h(t) es las altura sobre el nivel del mar.
g
dt
h d
=
'
'
Por la segunda ley de Newton
h(0) = 20 metros y 10 ) 0 ( =
dt
dh
metros. Por las condiciones iniciales.
Luego:

+ = = = c gt
dt
dh
gdt
dt
dh
g
dt
h d
2
2

Como
10 ) 0 ( 10 ) ( = + = = c c g t v
dt
dh
por consiguiente:
10 ) ( + = gt t v
Ahora:

UNIVERSIDADNACIONALABIERTAYADISTANCIA UNAD
ESCUELADECIENCIASBSICAS,TECNOLOGAEINGENIERA
CONTENIDODIDCTICODELCURSO:100411ClculoIntegral


+ + = + = = c t gt h dt gt dt t v h 10
2
1
) 10 ( ) (
2

Para hallar la constante, tenemos:
20 ) 0 ( 10 ) 0 (
2
1
20
2
= + + = c c g
Luego:
20 10
2
1
2
+ + = t gt h

Ejemplo No 2:
Una pelota es lanzada verticalmente hacia arriba con una velocidad de 50 m/seg.
(se ignora la resistencia el aire) cual ser la altura de la pelota cuando han
transcurrido 2 seg. del lanzamiento.
Solucin:
Por definicin:
g
dt
h d
=
2
2
Siendo g = 9,8
Ahora:
c gt gdt
dt
dh
+ = =


Como
50 ) 0 ( =
dt
dh
Entonces:
) ( 50 t v gt
dt
dh
= + =

Pero v(0) = 50 entonces:

UNIVERSIDADNACIONALABIERTAYADISTANCIA UNAD
ESCUELADECIENCIASBSICAS,TECNOLOGAEINGENIERA
CONTENIDODIDCTICODELCURSO:100411ClculoIntegral

( ) c t gt dt gt h + + = + =

50
2
1
50
2

Como h(0) = o, Luego:

t gt h 50
2
1
2
+ =

Cuando t = 2 seg, entonces:
6 , 21 4 , 78 100 ) 2 ( 50 ) 2 (
2
1
2
= = + = g h
metros.
Ejemplo No 3:
Del problema anterior, calcular la velocidad a los 3 seg. de haber sido lanzada la
pelota.
Solucin:
Como
t t v 8 , 9 50 ) ( =
Entonces, reemplazando el tiempo tenemos:
. / 6 , 30 4 , 29 50 ) 3 ( 8 , 9 50 ) ( seg m t v = = =

Ejercicios:
1. Las condiciones iniciales para un objeto que se deja caer desde una altura de
150 metros son:
a-) y(0) = 150 y(0) = 0
b-) y(0) = 0 y(0) = 150
c-) y(0) = 50 y(0) = 15
d-) y(0) = 15 y(0) = 150
Rta: a

UNIVERSIDADNACIONALABIERTAYADISTANCIA UNAD
ESCUELADECIENCIASBSICAS,TECNOLOGAEINGENIERA
CONTENIDODIDCTICODELCURSO:100411ClculoIntegral

2. Una persona se encuentra a 20 metros de altura de una piscina olmpica,
Cul ser la velocidad con que la persona toca el agua al dejarse caer de dicha
altura?
Rta: 20 8 m/seg.
3. Un objeto se mueve segn la ecuacin: ) ( 25 ) ( ' '
0
+ = wt sen t x , siendo x(t) = 0
y
x(0) = 0, adems
0
= 0 y w = 1. Cual ser la ecuacin de x(t) para este
problema.

Rta: t t sen t x
4
24
) 4 (
16
25
) ( + =
4. sabiendo que g
dt
dv
= Un cuerpo es lanzado verticalmente hacia arriba, con
una velocidad de 20 m/seg,
a-) Cual ser la altura mxima alcanzada por el cuerpo
b-) El tiempo de vuelo del cuerpo.
Rta: a-) 20,408 metros
b-) 4,081 segundos







UNIVERSIDADNACIONALABIERTAYADISTANCIA UNAD
ESCUELADECIENCIASBSICAS,TECNOLOGAEINGENIERA
CONTENIDODIDCTICODELCURSO:100411ClculoIntegral

Leccin 42: Integrales en la hidrulica: bombeo de lquidos.
Cuando se desea desplazar un lquido, es necesario hacer un trabajo. Debido a
que los recipientes o lugares donde se almacena el lquido no tiene forma regular,
la ecuacin W = F*d no aplica directamente, se requiere una transformacin segn
la forma del recipiente, para as poder aplicar dicha ecuacin. La resolucin se
sigue por las sumas de Riemman.
Con algunos ejemplos modelos podemos analizar problemas de este tipo.
Ejemplo 1:
Un tanque esfrico de 10 metros de radio y lleno de agua, se desea bombear el
agua por la parte superior del tanque. Determinar cuanto trabajo se debe hacer
para bombear toda el agua.
Solucin:
Se debe hallar

=
b
a
dx x F W ) ( La clave est en determinar la funcin F(x) para el
fenmeno en mencin.
Inicialmente sabemos que el intervalo de la variable x est entre 0 y 20 porqu?







Fig. No. 41 Bombeo.


UNIVERSIDADNACIONALABIERTAYADISTANCIA UNAD
ESCUELADECIENCIASBSICAS,TECNOLOGAEINGENIERA
CONTENIDODIDCTICODELCURSO:100411ClculoIntegral

El radio r corresponde a la profundidad de x = c
i
y que es la hipotenusa del
tringulo, su valor es de 10 metros, la altura es 10 c
i
, luego por Pitgoras:
( ) ( )
2 2
2
2 2 2
2
20 10 100 10 10
i i i i i i i
c c r c r r c = = = +

La fuerza para mover el lquido (agua), es la gravedad sobre el mismo.

peso V g V Vg g m F * * = = = =
Pero el peso es de 1.000 Kg/m
3
que
corresponde a la densidad del agua. Luego:
h r h r peso V F
2 2
1000 1000 * * = = =
Donde ya conocemos r
2
Entonces:
( ) x c c F
i i
=
2
20 1000
Aplicando la teora de particin y por la
sumas de Riemman.
( ) ( )
i i i i
c c x c c d F W = = 20 20 1000 *
2

Por consiguiente:
( ) x c c d F W
i i
= =
2
20 1000 *
El trabajo total ser el realizado en cada
capa.
( )

=
=
n
i
i i
x c c W
1
2
20 1000
Si aplicamos lmite cuando n tiende a infinito:
( ) ( )

+ = =
20
0
20
0
3 2
2
40 400 1000 20 1000 dx x x x dx x x W
Integrando:
( ) 33 , 333 . 13 1000
4
1
3
40
200 1000
20
0
4 3 2
=

+ = x x x W
Finalmente:
( ) 33 , 333 . 333 ' 13 33 , 333 . 13 1000 = = W
Julios.

UNIVERSIDADNACIONALABIERTAYADISTANCIA UNAD
ESCUELADECIENCIASBSICAS,TECNOLOGAEINGENIERA
CONTENIDODIDCTICODELCURSO:100411ClculoIntegral

Ejemplo 2:
Un depsito en forma de cono circular recto est lleno de agua, si la altura es de
10 pies y el radio de la parte ms ancha es de 4 pies, hallar el trabajo para:
a-) Bombear el agua del tanque por la parte superior
b-) Bombear el agua 10 metros por encima del nivel del tanque.
Solucin:






Fig. No. 42 Bombeo circular.

a-) Segn la grfica, x y
4
10
= , el disco tiene como grosor y y altura y, tiene un
radio y
10
4
, luego el volumen ser: y y

2
10
4
, con peso y v .
La fuerza necesaria para elevar el disco de agua es igual a su peso, entonces el
trabajo requerido para elevar el disco de agua ser:

( ) y y y d f w

= = 10 * )
10
4
( *
2
Por consiguiente:
( ) ( )dy y y dy y y w

= =
10
0
3 2
10
0
2
10
100
16
10
100
16
Resolviendo:

UNIVERSIDADNACIONALABIERTAYADISTANCIA UNAD
ESCUELADECIENCIASBSICAS,TECNOLOGAEINGENIERA
CONTENIDODIDCTICODELCURSO:100411ClculoIntegral

( ) 39 , 137 . 26
4
1
3
10
100
16
10
100
16
10
0
4 3
10
0
3 2
=

= =

y y dy y y w Lb-pie

Se tomo la densidad como 62,4 Lb/Pie
3

b-) El razonamiento es similar a la caso anterior, solo que para este caso la
altura es 20 y, luego:
( ) ( )dy y y dy y y w

= =
10
0
3 2
10
0
2
20
100
16
20
100
16
Los lmites no cambian
porque?
( )
10
0
4 3
10
0
3 2
4
1
3
20
25
4
20
25
4

= =

y y dy y y w Evaluando:
w = 130.687,60 Lb pie

Ejemplo 3:
Mostrar que para un tanque lleno de agua, de forma cilndrica vertical de 5 metros
de radio y 10 metros de altura, se debe hacer un trabajo de 69,3X10
6
Julios para
bombear el agua 4 metros por encima del tanque.
Solucin:
Por un lado: h r V
2
= donde y h = Luego: ( ) y V =
2
5 . Por otro lado, como
el
peso del agua es de 9.800 N/m
3
, entonces: V V F = = 800 . 9 , Pero
y V = 25 .
Luego: y y V F = = = 000 . 245 25 * 800 . 9 .
Ahora ( ) y F d F W = = 14 * * El tanque mide 10 metros de largo y 4 metros
que debe subir dems el lquido hace que la altura sea 14 metros. Ahora si
podemos hallar el trabajo:

UNIVERSIDADNACIONALABIERTAYADISTANCIA UNAD
ESCUELADECIENCIASBSICAS,TECNOLOGAEINGENIERA
CONTENIDODIDCTICODELCURSO:100411ClculoIntegral

( ) ( )
10
0
2
10
0
10
0
2
1
14 000 . 245 14 000 . 245 14 000 . 245

= = =

y y dy y dy y W
Evaluando obtenemos:
W = 69,3X10
6
Julios.
As queda demostrado el problema.

Ejercicios:
1. Un tanque esfrico est lleno de agua, el radio es de 10 pies y se desea
bombear por la parte superior el agua hasta que el tanque quede a la mitad, que
trabajo se realiza en este proceso. Rta:
816.814 pies-libra
2. Un tanque cilndrico vertical tiene 20 metros de altura y 10 metros de radio, Qu
trabajo se realiza para bombear el agua a un nivel de 4 metros por encima del
tanque.
D
agua
= 9.800 N/m
3
Rta: 862055.040 Julios

3. Un tanque de almacenamiento de forma cilndrico vertical tiene Kerosn, cuyo
peso es 51,2 Lb/pie
3
, el tanque tiene 30 pies de alto y 20 pies de dimetro. Qu
trabajo se necesita para bombear el combustible hasta el nivel superior del
tanque.
Rta: 7238.229,48 pies- libra

4. Un tanque tiene forma de cono circular invertido, 10 metros de altura y 4 metros
de radio en la parte ms ancha. Es llenado con agua hasta 8 metros de altura,
Qu trabajo se requiere para vaciar el tanque hasta la parte superior. Rta:
3,4X10
6
Julios

UNIVERSIDADNACIONALABIERTAYADISTANCIA UNAD
ESCUELADECIENCIASBSICAS,TECNOLOGAEINGENIERA
CONTENIDODIDCTICODELCURSO:100411ClculoIntegral

Leccin 43: Integrales en la estadstica: funcin de distribucin.

( ) ( )

=
b
a
dx x f b x a P

En Estadstica las integrales son una herramienta para hallar probabilidades de
ocurrencia de sucesos de variables aleatorias tipo continuo. Estudiaremos dos
casos de los muchos que se presentan, como ilustracin de las integrales a la
ciencia de la estadstica.
Funcin de distribucin:
Es la probabilidad de que una variable aleatoria con funcin de densidad de
probabilidad f(x) tome un valor menor o igual que x.
) ( ) ( x F x X P =

Donde: F(x) es la funcin de distribucin y x la variable aleatoria. Este tipo de
funcin no puede ser negativa, ya que corresponde a una funcin de probabilidad,
tampoco puede ser decreciente debido a que es acumulativa; adems, es
acumulativa. 1 ) ( 0 x F . Entonces para cualquier x, ) ( ) ( x X P x F = , que
significa el rea bajo la funcin de densidad de probabilidad sobre el intervalo (-,
x]. Por la notacin de integrales:

( ) ( )


=
x
dt t f x F

De la funcin de distribucin se puede resaltar:
-) F(-) = 0 y F() = 1
-) p( x
1
< X x
2
) = F ( x
2
) F ( x
1
)
-) p( x
1
< X x
2
) = P ( X x
2
) P ( X x
1
) 0


UNIVERSIDADNACIONALABIERTAYADISTANCIA UNAD
ESCUELADECIENCIASBSICAS,TECNOLOGAEINGENIERA
CONTENIDODIDCTICODELCURSO:100411ClculoIntegral

Funcin de Densidad de Probabilidad:
Tambin se le llama Densidad de Probabilidad. Sea f(x) una funcin llamada
como funcin de densidad de probabilidad; entonces:

( ) ( )

=
b
a
dx x f b x a P

rea bajo la curva:
Donde ) ( b X a P significa la probabilidad que la variable aleatoria tome un
valor entre a y b, la funcin f(x) obviamente debe ser integrable en el intervalo
establecido. Al elemento f(x)dx se le conoce como probabilidad elemental o
elemento diferencial de probabilidad.
Algunas propiedades de esta funcin:
-) f(x) 0 ya que p(x) 0
-) 1 ) ( ) ( = = < <


dx x f x p
-) 1 ) ( 0 b x a p

Ejemplo 1:
Dada una funcin de distribucin F(x) = 2x x
2
en [0, 1]. Hallar la funcin de
densidad de probabilidad
Solucin:
La funcin de densidad se obtiene derivando la funcin de distribucin
Como f(x) = F(x) entonces:

x
dx
dF
x f 2 2 ) ( = =

UNIVERSIDADNACIONALABIERTAYADISTANCIA UNAD
ESCUELADECIENCIASBSICAS,TECNOLOGAEINGENIERA
CONTENIDODIDCTICODELCURSO:100411ClculoIntegral

Ejemplo 2:
Para el ejemplo anterior, hallar la probabilidad de que un evento aleatorio sea
{ } 2 , 0 X
Solucin:
( ) ( ) 36 , 0 04 , 0 4 , 0 2 2 2 ) 2 , 0 (
2 , 0
0
2
2 , 0
0
= = = =

x x dx x X P Por consiguiente:
36 , 0 ) 2 , 0 ( = X P
Ejemplo 3:
Una variable aleatoria tiene como funcin de densidad de probabilidad:

>
=

otros para
x para e
x f
x
0
0 2
) (
2


Cual ser la probabilidad de que la variable tome un valor entre 1 y 3. 3 1 X

Solucin:
Por definicin:

=
b
a
dx x f b X a P ) ( ) (
Como X est en la condicin para que
x
e x f
2
2 ) (

= entonces:


= =
3
1
2
3
1
2
2 2 ) 3 1 ( dx e dx e X P
x x
Operando:
( ) 1328 , 0
2
2 ) 3 1 (
6 2 2 6
3
1
2
= =

e e e e
e
X P
x


UNIVERSIDADNACIONALABIERTAYADISTANCIA UNAD
ESCUELADECIENCIASBSICAS,TECNOLOGAEINGENIERA
CONTENIDODIDCTICODELCURSO:100411ClculoIntegral

Ejemplo 4:
Para el ejemplo anterior, cual es la probabilidad de que la variable tome un valor
mayor que .
Solucin:
Siguiendo el procedimiento de la definicin:

= =

2
1
2
2
1
2
2
1
2
2
2 2 )
2
1
(
x
x
x
e
e
dx e X P

( ) 3678 , 0 )
2
1
(
1 1
= =

e e e e X P

Existen muchas funciones de densidad de probabilidad, utilizadas en el mundo de
la Estadstica, tales como: La Normal, Log normal, x
2
de Pearson, otras. Estas se
pueden explorar en el curso de Estadstica y de Probabilidad.
Ejercicios:
1. La densidad de probabilidad de una variable aleatoria est dada por la funcin:

<
< <
=
otroscasos para
x si x
x si x
x f
0
2 1 2
1 0
) (

a-) Hallar la funcin de distribucin
b-) Determinar la probabilidad de que una variable aleatoria con esta funcin
de
distribucin tome un valor mayor a 1,8
Rta: a-)
2
2
1
2 ) ( x x x F = b-) 02 , 0 ) 8 , 1 ( = > X P
2. Para el ejercicio numero 1, determinar la probabilidad de que una variable
aleatoria con esta funcin de distribucin, tome un valor entre 0,2 y 0,6
Rta: 16 , 0 ) 6 , 0 2 , 0 ( = X P

UNIVERSIDADNACIONALABIERTAYADISTANCIA UNAD
ESCUELADECIENCIASBSICAS,TECNOLOGAEINGENIERA
CONTENIDODIDCTICODELCURSO:100411ClculoIntegral

3. La funcin de distribucin de una variable aleatoria est dada por la expresin:

>
=
2 0
2
4
1
) (
2
x para
x para
x
x F
Cual ser la probabilidad de que la variable aleatoria:
a-) Tome un valor menor que 3
b-) Tome un valor entre 4 y 5
Rta: a-) 555 , 0 ) 3 ( = < x P b-) 09 , 0 ) 5 4 ( = < < x P

4. El consumo de energa de cierta planta es una variable aleatoria, cuya funcin
de
densidad de probabilidad es:

3
9
1
) (
x
xe x f

=
Para x > 0
La planta tiene una capacidad diaria de 12 millones de Kw/hr. Cual ser la
probabilidad de que el suministro de energa sea inadecuado en un da dado.
Rta: 0916 , 0 ) 12 0 ( < < x P
5. la vida til de un artculo electrnico es una variable aleatoria, con funcin de
densidad de probabilidad:
x
e x f
6
6 ) (

= Cul es la probabilidad de que el artculo
dure menos de 3 meses?
Rta: 7768 , 0 ) 12 / 3 0 ( = < < x P
6. Para el caso de la vida til del artculo electrnico referenciado en el ejercicio
anterior, Cul ser la probabilidad de que el artculo electrnico dure entre 2 y 4
aos?
Rta: 000006144 , 0 ) 4 2 ( = < < x P

UNIVERSIDADNACIONALABIERTAYADISTANCIA UNAD
ESCUELADECIENCIASBSICAS,TECNOLOGAEINGENIERA
CONTENIDODIDCTICODELCURSO:100411ClculoIntegral

Leccin 44: Integrales en la economa.

( )

=
Q
QP dx x D C E
0
.

En Economa son muy usados los trminos demanda y oferta. La curva de
demanda del consumidor P = D(x), nos da el precio de demanda que el
consumidor est dispuesto a pagar por unidad para x unidades, la curva
generalmente es decreciente, debido a que al vender cantidades mayores, el
precio baja. La curva de oferta del productor P = S(x), nos da el precio por unidad
al cual el vendedor est dispuesto a ofrecer x unidades, la curva es creciente, ya
que a mayores cantidades, el precio de venta sube.




Fig. No. 43 curva oferta demanda.



CURVA DE OFERTA DEMANDA

La grfica muestra la curva de oferta
P = S(x) y la curva de demanda P = D(x).
P(Xc,Yp) corresponde al punto de equilibrio.
Utilidad: Es el concepto asociado con una funcin que describe el grado de
beneficio o satisfaccin, cuando el consumidor recibe x unidades.

UNIVERSIDADNACIONALABIERTAYADISTANCIA UNAD
ESCUELADECIENCIASBSICAS,TECNOLOGAEINGENIERA
CONTENIDODIDCTICODELCURSO:100411ClculoIntegral

EXCEDENTE DEL CONSUMIDOR: (E.C.)
En trminos sencillos, el excedente del consumidor E.C. es la cantidad de dinero
que ahorra un consumidor cuando compra un artculo a P precio, para una
cantidad x de artculos. Lo anterior se traduce en la utilidad del consumidor,
cuando disminuye el precio a razn de aumentar la
compra del artculo.
Para Q artculos el precio es P, luego el gasto total ser
QP. El rea total bajo la curva es la utilidad total U.

=
Q
dx x D U
0
) (

D(x) es la funcin demanda. Fig. No. 44 Excedente.

As, el excedente del consumidor ser entonces la utilidad menos los gastos
totales.
( )

=
Q
QP dx x D C E
0
.
Excedente del Consumidor
EXCEDENTE DEL PRODUCTOR: (E.P.)
Los economistas lo refieren a la utilidad que recibe el productor, cuando se ofrece
mayores cantidades del artculo, a razn del aumento del precio. Esto significa los
ingresos extras que recibe el productor, cuando el consumidor aumenta la compra
del artculo.




Fig. No 45 Excedente del productor.

UNIVERSIDADNACIONALABIERTAYADISTANCIA UNAD
ESCUELADECIENCIASBSICAS,TECNOLOGAEINGENIERA
CONTENIDODIDCTICODELCURSO:100411ClculoIntegral

Como Q es la cantidad de artculos ofrecidos a P precio, la cantidad recaudada
ser de QP. El excedente del productor E.P, ser el recaudo total menos el rea
bajo la curva, que corresponde a la funcin oferta de produccin.
( )

=
Q
dx x S QP P E
0
.
Excedente del productor.
Ejemplo 1:
Dadas las funciones demanda D(x) = (x 5)
2
y de oferta S(x) = x
2
+ x + 3, hallar
a- El punto de equilibrio
b- El E. C. en el punto de equilibrio
c- El E. P. en el punto de equilibrio

Solucin:
a- El punto de equilibrio es donde D(x) = S(x), es decir: (x 5)
2
= x
2
+ x + 3.
Haciendo las operaciones algebraicas: 3 25 10
2 2
+ + = + x x x x , despejamos la
variable, luego: x
E
= 2. Ahora podemos hallar el valor de y, as: y
E
= (2 5)
2
= 9, el
punto de equilibrio ser: P(2, 9)

b- Para calcular el excedente del consumidor, utilizados la ecuacin para E. C.
( ) ( ) ) 9 ( * ) 2 ( 5
3
1
5 . .
2
0
2
0
3 2
= =

x QP dx x C E

Evaluando y simplificando:
( ) 667 , 14 18 667 , 32 18 125 27
3
1
. . = = + = C E

c- De igual manera que en el caso anterior, el excedente del productor se calcula
con la ecuacin para este fin.
( )dx x x dx x S QP P E

+ + = =
2
0
2
2
0
3 18 ) ( .
Desarrollando tenemos:

UNIVERSIDADNACIONALABIERTAYADISTANCIA UNAD
ESCUELADECIENCIASBSICAS,TECNOLOGAEINGENIERA
CONTENIDODIDCTICODELCURSO:100411ClculoIntegral

33 , 7
3
22
3
32
18 3
2
1
3
1
18 .
2
0
2 3
= =

+ + = x x x P E

Ejemplo 2:
La demanda de un producto est gobernada por la funcin:
2
0001 . 0 2 . 0 200 . 1 ) ( x x x D = Cul ser el excedente del consumidor para un
nivel de ventas de 500 unidades?
Solucin:
Para este caso Q = 500, luego P = 1.200 0,2(500) 0,0001(500) = 1.075,
entonces el gasto total ser de QP = 500*1075 = 537.500
Ahora calculamos el E. C. utilizando la ecuacin correspondiente.
( ) 500 . 537 0001 , 0 2 , 0 200 . 1 . .
500
0
2
=

dx x x C E

( ) 500 . 537 5 , 837 . 570 500 . 537 10 33 , 3 1 , 0 200 . 1 . .
500
0
5 2
= =

X x x C E

5 , 337 . 33 500 . 537 5 , 837 . 570 . . = = C E
Ejemplo 3:

Determinar el E. P. Para un producto cuya funcin oferta es: x x x S 2
2
1
) ( + = ,
para x = 20.
Solucin:
Para este caso Q = 20, luego P = 20/2 + 2(20) = 50. Entonces: QP = 20*50 =
1.000
A continuacin se calcula el E: P.

UNIVERSIDADNACIONALABIERTAYADISTANCIA UNAD
ESCUELADECIENCIASBSICAS,TECNOLOGAEINGENIERA
CONTENIDODIDCTICODELCURSO:100411ClculoIntegral

( ) 400 100 000 . 1
4
1
000 . 1 2
2
1
000 . 1 . .
20
0
2 2
20
0
+ =

+ =

+ =

x x dx x x P E

( ) 500 400 100 000 . 1 . . = + = P E

EJERCICIOS:
1. La funcin oferta de cierto artculo est dada por: 5
10
) ( + =
x
x s . Para un precio
de venta de $10. Calcular el excedente del productor cuando el precio de venta es
de $10.
Rta: E.P.=$4.166,67

2. La funcin demanda para un producto es de la forma
8
450
) (
+
=
x
x D .
a-) Cual ser el nivel de venta para un precio de $10
b-) encontrar el excedente del consumidor para el nivel de ventas de la parte a.
Rta: a-) Q = 37
b-)E.C.= 407,15

3. En un anlisis econmico, la funcin demanda y oferta son respectivamente:
( )
2
4 ) ( = x x D y 6 2 ) (
2
+ + = x x x S . Calcular el excedente del productor en el
punto de equilibrio.
Rta: E. P. = $1,67
4. Para el caso el problema 3, calcular el excedente del consumidor, cuando la
venta es de un artculo.
Rta: E. C. = $3,33

UNIVERSIDADNACIONALABIERTAYADISTANCIA UNAD
ESCUELADECIENCIASBSICAS,TECNOLOGAEINGENIERA
CONTENIDODIDCTICODELCURSO:100411ClculoIntegral

COSTO TOTAL:
Siguiendo el estudio de las integrales en la economa, se debe hacer notar otros
trminos que en economa son frecuentes como costo marginal y costo total. El
concepto de Marginal hace referencia al cambio que manifiesta una cantidad,
cuando hay un cambio muy pequeo de una segunda cantidad, en este orden de
ideas si conocemos la funcin costo marginal C(x) o dC/dx, se puede hallar el
costo total. C(x), entendiendo este ltimo como el costo necesario para producir x
unidades de cierto artculo.
El costo marginal ser C(x) siendo x=x
i
para i = 1, 2, 3, Si la derivada existe,
entonces a dicha funcin se le llama funcin costo marginal.
Con el principio de la antiderivada, podemos inferir que a partir del costo marginal
podemos hallar el costo total. Al realizar el proceso de integracin, la constante
arbitraria, se puede evaluar si se conoce el costo general; es decir, el costo sin
producir unidad alguna, entonces:

( ) ( )

= dx x c x C
Costo total de produccin
NOTA: El costo marginal, no puede ser negativo, luego c(x) 0
Ejemplo 1:
Dad la funcin costo marginal . 12 3 = x
dx
dC
la produccin de 4 unidades, origina
un costo de $16. Hallar la funcin costo total.

Solucin:
Como 0 12 3 0 x
dx
dC
Luego 4 x Ahora:
c x x dx x x C + = =

12
2
3
) 12 3 ( ) (
2

Pero C(4) = 16, entonces: c + = ) 4 ( 12 ) 4 (
2
3
16
2
despejando c, se obtiene: c = 40.
Por consiguiente: 40 12
2
3
) (
2
+ = x x x C

UNIVERSIDADNACIONALABIERTAYADISTANCIA UNAD
ESCUELADECIENCIASBSICAS,TECNOLOGAEINGENIERA
CONTENIDODIDCTICODELCURSO:100411ClculoIntegral

Pero la mnima cantidad que se debe producir es de 4 unidades. 4 x
Ejemplo 2:
En un proceso de produccin la funcin costo marginal est dada por:
4 5
3
+
=
x
dx
dC

El costo general es de $10, Cul ser el costo total?

Solucin:

+
=
+
=
4 5
3
4 5
3
) (
x
dx
dx
x
x C Aplicando cambio de variable: u = 5x + 4 entonces
du = 5dx, despejando dx = du/5, ahora reemplazamos en la integral original.


= =
+
du u
u
du
x
dx
2 / 1
5
3
5
3
4 5
3 Integrando se obtiene:
c u c
u
+ = +
5
6
2 / 1
*
5
3
2 / 1
Luego:
c x x C + + = 4 5
5
6
) (
Para hallar el valor de c, tomamos las condiciones dadas: C(0) = 10, entonces:
c + + = 4 ) 0 ( 5
5
6
10 Despejando c se obtiene: c = 38/5. Finalmente:
5
38
4 5
5
6
) ( + + = x x C



UNIVERSIDADNACIONALABIERTAYADISTANCIA UNAD
ESCUELADECIENCIASBSICAS,TECNOLOGAEINGENIERA
CONTENIDODIDCTICODELCURSO:100411ClculoIntegral

INGRESO TOTAL:
Para estudiar el ingreso total, debemos recordar el concepto de ingreso marginal,
denotado por R (x), para x = x
i
con i = 1, 2, 3, La funcin R (x
i
) si existe se
le llama ingreso marginal. Esta funcin puede ser positiva, negativa o cero Se
interpreta como la tasa de cambio del ingreso total cuando se requieren x
unidades.
A partir del ingreso marginal, podemos obtener el ingreso total, por medio de
integrales indefinidas.
Si p es el precio unitario y x las unidades vendidas, entonces el ingreso ser:
R(x) = p*x
Segn la ecuacin anterior, el ingreso total lo podemos obtener a partir del ingreso
marginal.
( ) ( )

= dx x R x R

Ejemplo 1:
Cual ser el ingreso total para la funcin marginal R (x) = 300 x
Solucin:
Por definicin: ( ) c x x dx x x R dx x R x R + = = =

2
2
1
300 300 ) ( ) ( ' ) (
Para hallar el valor de c, partimos de la siguiente premisa: El ingreso es cero,
cuando el nmero de unidades es cero; es decir, R(0) = 0
Reemplazando en la funcin obtenida: 300(0) (0)
2
+c = 0, despejando c se
obtiene que c = 0, por consiguiente:
2
2
1
300 ) ( x x x R =

Recordemos que cuando x = 0, no hay ingresos.

UNIVERSIDADNACIONALABIERTAYADISTANCIA UNAD
ESCUELADECIENCIASBSICAS,TECNOLOGAEINGENIERA
CONTENIDODIDCTICODELCURSO:100411ClculoIntegral

Ejemplo 2:
La utilidad total se le llama P(x) y se define como:
( ) ( ) ( ) x C x R x P =

Una compaa tiene para un artculo el valor de $100 la unidad; precio de venta. Si
produce diariamente x unidades, el valor por produccin marginal es 2x + 20. El
costo general es de $700.
Hallar:
a-) La funcin utilidad total
b-) La utilidad que se obtiene al producir 40 unidades.

Solucin:
a-) C (x) = 2x + 20 Entonces: ( ) c x x dx x x C + + = + =

20 20 2 ) (
2

Para C(0) = 700, luego: 700 = 0
2
+ 20(0) + c, c = 700, la funcin costo total ser:
700 20 ) (
2
+ + = x x x C

La funcin ingreso ser: R(x) = 100x como tenemos la funcin costo total C(x),
entonces podemos calcular la funcin utilidad.
( ) 700 80 700 20 100 ) (
2 2
+ = + + = x x x x x x P

As, la funcin utilidad total ser:
700 80 ) (
2
+ = x x x P

b-) Como conocemos la funcin utilidad, solo reemplazamos para x = 40,
entonces:
700 ) 40 ( 80 ) 40 ( ) 40 (
2
+ = R
Desarrollando:
900 300 . 2 200 . 3 ) 40 ( = = = x R

UNIVERSIDADNACIONALABIERTAYADISTANCIA UNAD
ESCUELADECIENCIASBSICAS,TECNOLOGAEINGENIERA
CONTENIDODIDCTICODELCURSO:100411ClculoIntegral

EJERCICIOS: Leccin No. 4
1. Para cierta mercanca la funcin ingreso marginal est dada por: R (x) = 20
4x, cual ser el ingreso total cuando se requieren 10 unidades de la mercanca.
Rta: R(x =10) = 0

2. La funcin costo marginal para cierto artculo est gobernado por:
4 5
3
) ( '
+
=
x
x C
Si el costo general es de $10, cual ser el costo total en la produccin de 50
artculos.
Rta: C( x = 50 ) = 26, 725

3. En la produccin de una pasta de jabn para tocador, la funcin ingreso
marginal se determin como:
2
2 8 ) ( ' x x x R + = . Cul ser el ingreso total para
12 unidades?
Rta: R(x =12) = 528

4. La fbrica de bombillas El Alumbrador tiene como precio de venta para su
artculo el valor de $700 la unidad. Si produce diariamente x unidades, el valor por
produccin marginal es 5x + 8. El costo general es de $800. Cul ser la
utilidad al producir 50 bombillas?
Rta: P(x) = $27.550




UNIVERSIDADNACIONALABIERTAYADISTANCIA UNAD
ESCUELADECIENCIASBSICAS,TECNOLOGAEINGENIERA
CONTENIDODIDCTICODELCURSO:100411ClculoIntegral

Leccin 45: Integrales en las ciencias sociales.

Vemos los siguientes ejemplos:
Problema No. 1
La rata a la cual est creciendo la poblacin de cierta ciudad, cambia con el
tiempo. Se calcula que dentro de t meses la rata de crecimiento ser de t 4 3+
personas por mes. La poblacin actual es de 25.000 personas.
Cual ser la poblacin dentro de 16 meses?

SOLUCION:

Designemos ( ) t p la poblacin dentro de t meses. La derivada de ( ) t p expresa la
rata de cambio de la poblacin, es decir t
dt
dp
4 3+ =
Se deduce que la funcin poblacin ( ) x p es una integral, por lo tanto:
( ) ( ) c
t
t c
t
t c
t
t dt t t p + + = + + = +
+
+
+ = + =

3
8
3
5 . 1
4
3
1 5 . 0
1 4
3 4 3
2 / 3 2 / 3

Para determinar t usamos la informacin sobre la poblacin inicial o sea 0 = t .
Si ( ) C t P t = = = 25000 0
Por lo tanto: ( ) 25000
3
8
3
5 . 1
+ + =
t
t t P
Dentro de 16 meses la poblacin ser de: ( ) ( )
( )
25216 25000
3
16 8
16 3 16
5 . 1
= + + = P
personas.

UNIVERSIDADNACIONALABIERTAYADISTANCIA UNAD
ESCUELADECIENCIASBSICAS,TECNOLOGAEINGENIERA
CONTENIDODIDCTICODELCURSO:100411ClculoIntegral

Problema No. 2
Dentro de x horas, las bacterias de cierto cultivo estarn creciendo a una rata de
x
e x
2
5 4 + por hora. El nmero de bacterias actual es de 10.000.
Cuantas habr despus de transcurrir 8 horas?
SOLUCION:
La rata de crecimiento se obtuvo de derivar, por lo tanto para hallar la funcin de
crecimiento integramos:
( )

+ + = + c e x dx e x
x x 2 2 2
5 . 2 2 5 4
Para determinar C realizamos x=0 en ( ) x f , entonces: ( ) 10000 5 . 2 0 = + = c f , por lo
tanto: 5 . 9997 = c
( ) 5 . 9997 5 . 2 2
2 2
+ + =
x
e x x f

UNIVERSIDADNACIONALABIERTAYADISTANCIA UNAD
ESCUELADECIENCIASBSICAS,TECNOLOGAEINGENIERA
CONTENIDODIDCTICODELCURSO:100411ClculoIntegral

ACTIVIDADES DE AUTOEVALUACIN DE LA UNIDAD
PREGUNTAS DE SELECCIN MLTIPLE CON NICA RESPUESTA

A continuacin, usted encontrar preguntas que se desarrollan en torno a un
enunciado, problema o contexto, frente al cual, usted debe seleccionar aquella que
responde correctamente a la pregunta planteada entre cuatro opciones
identificadas con las letras A, B, C, D. Una vez la seleccione, mrquela en su hoja
de respuestas rellenando el valo correspondiente.

1. El rea entre las curvas
4 4
2
= +y x
y
1
4
= y x
es:
A.
15
104

B.
53
10

C.
17
4

D.
45
235

2. Cul ser el volumen del solido generado por las curvas


x y =
y
1 = y cuando giran alrededor del eje x:
A. 094 . 2
B. 98 . 1
C. 78 . 4
D. 61 . 3

UNIVERSIDADNACIONALABIERTAYADISTANCIA UNAD
ESCUELADECIENCIASBSICAS,TECNOLOGAEINGENIERA
CONTENIDODIDCTICODELCURSO:100411ClculoIntegral

3. El volumen del solido generado cuando se hace girar alrededor del eje
y
,
la regin por encima de la parbola
2
) ( x x f = y por debajo de la curva
2
2 ) ( x x g =

A. 1416 . 3
B. 287 . 6
C. 416 . 2
D. 652 . 4

4. Si
g
dt
dv
=
.Entonces un cuerpo que es lanzado verticalmente hacia arriba,
con una velocidad de
seg
m 20
, Que altura alcanza?

A. 408 . 20
B. 456 . 13
C. 321 . 54
D. 98 . 37

5. Sea la variable aleatoria X, que tiene como funcin de densidad de


probabilidad

>
=

otros para
x para e
x f
x
0
0 2
) (
2

Cul es la probabilidad de que la variable tome un valor mayor que 5 . 0

UNIVERSIDADNACIONALABIERTAYADISTANCIA UNAD
ESCUELADECIENCIASBSICAS,TECNOLOGAEINGENIERA
CONTENIDODIDCTICODELCURSO:100411ClculoIntegral

A. 3678 . 0
B. 4531 . 0
C. 7865 . 0
D. 1943 . 0

6. La funcin ingreso marginal est dada por ( ) x x R 4 20 = . Cul ser el


ingreso total cuando se requieren 10 unidades de la mercanca?
A. 0
B. 1
C. 2
D. 3
HOJA DE RESPUESTAS.

A B C D
1
2
3
4
5
6

UNIVERSIDADNACIONALABIERTAYADISTANCIA UNAD
ESCUELADECIENCIASBSICAS,TECNOLOGAEINGENIERA
CONTENIDODIDCTICODELCURSO:100411ClculoIntegral

FUENTES DOCUMENTALES DE LA UNIDAD 3

RONDON, J.E (2007) Calculo Integral. Primera edicin, UNAD Ciencias bsicas
PURCELL, E (2001) Clculo, Pearson Education: Prentice hall, Octava Edicin,
Mxico.
THOMAS Y FINNEY (1987). Clculo con Geometra Analtica Vol. 1. Edicin
sexta, Addison Wesley Iberoamericana. Mxico.
STEWART, J. (2001) Clculo de una Variable. Thomsom-Learning. Cuarta edicin,
Bogot.
LARSON, R. Y HOSTETLER, R. (1998) Clculo Vol. 1, Mc Graw Hill, sexta
edicin, Mxico.
SMITH, R. Y MINTON, R. (2002) Clculo Vol. 1. Segunda Edicin, Mc Graw Hill,
Bogot.
BAUM Y MILLES. (1992). Clculo Aplicado. Limusa, Mxico.
LEYTOLD, L. (1987) El Clculo con Geometra Analtica. Harla, Mxico.
PITA, C. (1998) Clculo de una Variable. Pearson educacin, Mxico.
DE BURGOS, J. (2007) Clculo infinitesimal de una Variable. McGraw Hill, Madrid.

FUENTES DOCUMENTALES DE LA INTERNET

http://www.xtec.cat/~jlagares/integral.esp/integral.htm
http://thales.cica.es/rd/Recursos/rd97/Problemas/541pIntegral.html
http://sigma.univalle.edu.co/index_archivos/calculo1y2/formulasdecalculo1y2.pdf
http://www.matematicasbachiller.com/temario/calculin/index.html
http://es.wikipedia.org/wiki/Teorema_fundamental_del_c%C3%A1lculo
http://www.aulafacil.com/matematicasintegrales/curso/Temario.htm

UNIVERSIDADNACIONALABIERTAYADISTANCIA UNAD
ESCUELADECIENCIASBSICAS,TECNOLOGAEINGENIERA
CONTENIDODIDCTICODELCURSO:100411ClculoIntegral
http://www.monografias.com/trabajos10/historix/historix.shtml
http://www.fata.unam.mx/tecnologia/material/sem01/Calculo_I_Historia_1.pdf
http://www.uam.es/personal_pdi/ciencias/barcelo/histmatem/calculo/calculo.html
http://integrals.wolfram.com/index.jsp?expr=x%5E2*%28x4%29%5E0.5&random=false
http://www.dma.fi.upm.es/docencia/primerciclo/calculo/tutoriales/integracion/
http://www.matematicasypoesia.com.es/ProbIntegral/problema110.htm
http://usuarios.iponet.es/ddt/logica1.htm

Das könnte Ihnen auch gefallen